Download as pdf or txt
Download as pdf or txt
You are on page 1of 261

注意:

允許學生個人、非營利性的圖書館或公立學校合理使用
本基金會網站所提供之各項書籍、試題及其解答。可直
接下載而不須申請。

重版、系統地複製或大量重製這些資料的任何部分,必
須獲得財團法人臺北市九章數學教育基金會的授權許
可。

申請此項授權請電郵 ccmp@seed.net.tw

Notice:

Individual students, nonprofit libraries, or schools are


permitted to make fair use of the books, the papers and
its solutions. Republication, systematic copying, or
multiple reproduction of any part of this material is
permitted only under license from the Chiuchang
Mathematics Foundation.
Requests for such permission should be made by
e-mailing Mr. Wen-Hsien SUN ccmp@seed.net.tw
目次

1
第1章基本概念 1
1. 集與子集.................... •• 1
2. 對應和函數 3

3. 函數的圖象 6
4. 初等函數 :............
習題1.......................... 14
15
第2章極限、連續函數、微分導數 15
2G
1. 極限…................................. 23
2. 極限的性質................... 25
n9
習題2 oo
35
3. 函數的連續性 33 3
6
W 3
4. 導數的定義
習題4....... '..................
第3章 微分規律及微分的意義•……
1.微分規則
-1 -
習習5 ” :… 43
48
習応6............ ........ c 51
52
習題7 54
55
微分公式的總結 - 56
58
2. 髙次微分導數 59
62
習題8 ............. 一• .63
67
67-
3. 隱微分....................
習題9 *............. -
4. 微分......................
習題10................. .
5. 可微性與連續牲
第4章導數的應用
1. 遞增函數和遞減函數•…
69
2. 局部的極大和極小
79
3. 第二微分二數 •••…
習題11.............*•........ •
89
90
4. 導數在商業上的應用•… •
95
96
習題 12..... :•...... ........ .
100
101
5. 不定式的計算… •
04
105
1
習題i3......... :.......... 1
0

6. 中値定理...... ......... . 1
.習題14...... ............... 1

7. 孤長 1

習題15・、

-2 -
8. 質點在曲綫上的運動.速度的矢量性 -111
習題16 -114
9. 加速度的矢量性十••…• -115
習題17•: •120
第5章積分概念的引爆••… ■122
1. 反導數 •122
2. 代換法. ..... ........... •126
3. 代換法的進一步練習… •129
習題18 •133
4. 微分方程… •135
習題19・・・...................... ■140
5. 微分方程的應用 •141
習題20..................... •146
第6章積分的技巧 148
1. 一般較困難一點的例・• 149
習題21..................... 153
2. 部份積分法............. 154
3. 遞推公式 ...... 157
習題22....................... 159
4. •有理函數的積分 161
5. 部份分數(分項分數) 165
習題23 .............. 170
6. 一些包含三角函數和雙曲函數的代換 •170
習題24 ......... ............. 176
• 3 •
第7章、定積分 ••178
1. 用面積來表達一定積分…••…••… ••178
2. 定積分的分析定義… ••181
3. 定積分的基本性質 ••183
4. 計算定積分的方法 ••187
習題25 ............ •190
5. 定積分的代換........... :… ••191
習題26 ............................ .…… ••192
6. 用積分來計算面積 ••193
習題27 .................... ••196
7. 旋轉體的體積計算 -197
習題28 ••'200
第8章無窮級數 ................. ••202
1. 級數的槪念 ........... ••202
2. 對數函數和反正切函數的級數… ••206
3. 重要的定理 ••211
4. 怎樣判別正項級數的收斂或發散
習題29 .... .... •227
習題答案…............ • 229

• 4 •
第1章基本槪念
1.集與子集
集(Set)是數學裏最原始的槪念之一,所以不需用更簡單的
槪念去給它定義。舉例來說,課室裏的學生組成一個集,銀行裏
的人員組成一個集,太陽系的行星組成一個集,下列的英文字母
A、B、C、D也組成一個集。我們也可以用下面的同義字去代替''集"
這一術語:“全體”、“聚合"(Aggregate), “系”、“複元”等等。
組成集的事物我們稱作集的元素(Elements)。任何集的一部份我
們稱它爲子集(Subset)o一個沒有元素的集叫作空集(Empty set)o
在上面舉出的第四個例子中,英文字母A、B. C、D是這個集的
元素,而ABC,或BCD,或AB則是組成這個集的子集。

2.對應和函數
對應(Correspondence)也是最簡單槪念之一。現在我們也
舉例解釋。我們可使正方形的邊長與它的面積對應,或半徑的長
• 1・
與圖的面積對應。意思就是說。給出正方形的邊長或圓形的半
徑,我們有一定的規則來計算它們的面積。所以每一個邊長或半
徑的値都對應於從這個値計算出來的面積。如果稱全部邊長値組
成的集爲M,計算出來的面積所組成的集爲N,則對於第一個
集的每一個元素x我們有一個規律使它對應於第二個集的一個元
素y。在數學裏,我們說這個規律建立了一個從M集到N集的
函數(fimction)o通常我們用下面的記號代表函數:
y = f(x)。其中y是N的元素,x是M的元素。第一個集M
叫做函數的定義域(Domain of definition),它的元素叫做變數
値。第二個集N我們叫做値域,它的元素y我們叫做函數値。
從此可見,一個函數可從下面三點確定:、
X).變數値的集合(卽定義域)M。'
• 2).函數値的集合(卽値域)N。
3).定義域和値域之間的對應規律。例如給出=個正方形南
邊長(一個M集的元素),我們計算它的平方便可得到它所對應
的面積(一個N集的元素)。這個命令“計算它的平方”,便是這
個函數的對應規律了。在一般情形下,M集和N集都是實敷集,
.卽函數是定義和取値於實數集上(Set of Real Numbers)。道在
下面討論初等函數時便會遇到了。
每一個變數値在習慣上又叫原像,對應的函數値則叫像
(Image)。所以,集M的每一個元素對應於一個像含於集合N。
所有函數値的集合叫作實數値集合的像。我們通常用''寫像"
(Mapping)這個同意義的術語去代替“函數”這一名詞。同時討論
兩個或多個函蛟的時候我們就用不同的記號去代表不同的函數。
所以,除了用f(x)外,我們還可利用下面的記號。
<P (x), #(x),或 fi(x), f2(x), ,仞 1,(X)0 2(X)
等等。自然我們也可用任意別的記號來代表函數,如(X),
a(x), ^(x)..... 等等。
在研究別的自然科學時,函數的槪念也很重要。例如,自由
落體所經過的距離依附於運動開始所需的時間。更確切一點,我
們說,自由落體所經的距離是運動時間的函數;擺(Pendulum)
的振動的週期是擺長的函數;或者說付與某一已知貨物的價値是
該貨物的量的函收。

3.函數的圖象
現在我們討論一下上面提過的特殊情形,那就是函數域和定
義域都是實變數的情形。這時候,假若給出一個實變數的集合
M,又對於M的每一個元素X,'我們都能使其對應於一個確定
的實變數y,那y就是X的函數。由上述我們知道這種函數可由
.下面二要點確定:-
1) ・變數値的集合。
2) ・函數的對應規律。
上面所討論的例子中,每一個變數値只對應一個函數値。這
種函數我們稱爲單値函數(Single valued functions)o在複雜一
點的情形下,一個變數値可能對應很多個函數値,這種函數我們
稱爲多値函數(Multi-valued functions)o
【例】 若M集是0與1之間所有的實數,則y=x2是一單
値函數,它的定義域和値域都是M集本身。若M集是1與負1
之間所有的實數,則y=arcsinx是一多値函數,它的定義域是
M而它的値域則是一個更大的集。
對於定義和取値於實數的函數我們有一個很簡單的方法來表
示,那就是笛卡兒座標(Cartesian coordinates)的圖象法。我們
先取兩個數軸X和Y來表示函數的定義域和値域。一般來說,
這兩數軸的方位是可以隨意的,但最常用的方法是使這兩軸相互
垂直。横的一條我們就稱爲X-軸或横軸,通常是用來表示函蚊的
定義域。縱的一條稱作丫-軸或縱軸,通常是用表示函數的値
y

-4
-3
2 a

f(a)

M.马加12 3 4
x

-2
-3
-4
-5

• 4 •
域。X-軸和Y-軸交於原點。選軸之後,我們便用一個適當的長
度作單位在X-軸上表示出實數的位置,正的在原點之右,負的
在原點之在。同樣,我們在Y-軸上表示實數的位置,正的在原
點之上方,負的在原點之下方。當然,丫-馴和X-軸上的長度
單位不需相同,如圖1所示。
我們把一個變蛟値(當然,它是屬於M集或函數定義域的)
a及其對應的函數値f(a)表示在平面上如上圖示。把同樣的手
續重程,直至對有足夠的點使我們可畫出一連續光滑的曲綫如圖
2所示。
y

圖2. f(c)
f(b)
f( a)

0 a b c x

-5 •
4.初等函數(Elementary Functions)

在進行討論一些比較抽象的槪念之前,我們先來認識一下一
些常見的初等函數(Elementary functions)o
a).常數函數(Constant functions)
這個函數的定義域是一個實數集M,它的値域只有一個實
數,所以M集裏的每一元素都對應於這個實數。在笛卡兒座標圖
中,這種函數是用一直綫平行於X軸來表示,例如f(x)=2, x
是任意一個實數。
b>.綫性函數(Linear functions)
這種函數的對應規律如下:•
y = mx+a
y

xi.yi)

圖3

• 6 •
其中x是定義域內的實數,與a是給出的實數。在笛卡兒座標圖
中,這種函數的表示如圖3。
常數m叫函數的斜率(Slope),如果它是零的話,這個函數
便變成常數函數了。它可用下面的公式求出:
m- y2-yi
X2 —Xi
其中(y2, X2)與(yi, X。是綫上的任何二點。
c).慕函數(Power functions)
'這種函數的對應規律是:
• y=x"
其中x是定義域中的實數。n = 2, 3, o在笛卡兒座標
圖中,這種函數作如下的表達(見圖4):

* 7 -
n若是0.1,這個函數便成爲曾討論過的常數函數和綫性函數了。
d).三角函數(Trigonometric functions)
在所肴的高等分析研究中,我們都用半徑角來測量角的大
小。我們把要量的角的頂點放在一個半徑等於1的圖心內,角的
大小便取决於被角的二邊從圓周上所截出的弧的長度,如圖5所
示。所以,一個180。的角是和一個V半徑角的角—樣,一個90。
曰6 AXI兀•一 一」…- …一一.一 一….. 亠一一•

的角是和一個幸半徑角的角一樣。反過來說,一個等於一半徑角的
角用度數來表示是等於,180°
it = 57°, 17', 45〃。從今開始,每當

B D

tanx
x

圖5 0
-—COSX

• 8 •
我們提及角x時,就是說這是一個半徑角爲x的角。
簡單的三角函數sinx、cosx. tanx、cotx在笛卡兒座標圖
中它所作的表示如圖6及圖7所示。
y

-x
2k
.2

圖6
y
3
y=tanx \
2
--t —

0 3 \

2n
圖7 岩'、 2K \

\ y=ctgx

e)・反函數(Inverse functions)
在上面對函數的討論我們很小心的把變數値和函數値分別開
來,二者藉函败的對應規律聯繫着。其實這種分別是不必要的,
• 9 -
因爲在某種情形下,若給出函數値,我們也可用對應規律把變數
値計算出來,所以變數値也是函數値的函數。例如在考慮綫性函
數時,我們把變數値和函數値的關係寫成y=mx+a。現在設
m尹。,則我們可把這個式子寫成
x_ (y—a)
m
這樣便很清楚的表明了變數値x對函數値的函數依附了。所以,
給出一個任意的函數y=f(x)時,我們可以研究在什麽情形之下
可以决定X對y的函數依附,或者說我們把原來的函數y=f(x)
換成它的反函數x= p(y)。
在笛卡兒座標圖中,我們可把上面所解釋的理解如下!
1) . •我們首先把y=f(x)的圖像描出來。
2) .把正X-軸和正y-軸間的角平分。
3) .則y=f(x)的反函敷x=c(y)的圖就是原來的曲綾在
這分角綫上的倒映如圖8所示。
y

y=f(x)
9(y)

1
LT"

I
|
y
<

y1 z I

o y

8
1o
f) •寒函數的反函數(Inverse functions of power funct-
ions)
幕函數的對應規律是y=x\若n爲偶數時,它的反函數表
示式是x = *亍=y"o在初等數學裏,我們說x是y的正n方
根。若口爲奇數時.則不論x爲正或負,這個反函數都存在而
V玉■是依附x爲正負。
g) .反三角函數(Inverse trigonomtric functions)
y

I 2乒

12

圖9 I &
I 2

0 -x

// L2L
2

-11 -
1) . y=sin x的反函數的表示式是x=arc siny或x=sin-iy。
它在笛卡兒座標圖中的表示如圖9所示,當- 7T<XO 時,這
個函數是單値的。
2) . y=cosx的反函數的表示式是x=arccosy或x=cosTy,

它在笛卡兒座標圖中的表示如圖10所示,當孝<x<0時,這
y

圖10
x
-1 0 Z1
/

個函數是單値的。
3).用同樣方法,我們可考慮y=tanx和y=cot x是反函
數並得出在什麼情形之下這些反函數是單値函數。
h).指數函數和對數函數(The Exponential function and
logarithmic functions)©
指數函數的表示式是
y=ax
其中a是一個正的實常數而x是函數定義域內的實數。a又叫指
-12 -
數函數的基底(Base)。指數函數的反函數叫對數函數,它的表
示式爲:
X = loga yo
在初等蚊學裏,我們常常不討論在定義這些函數時所遇到的困
難,在這裏我們也不對這些小節作詳盡的討論| •只是簡畧地去了
解這些函數的定義。
當x=^時,(P與q是正整數),假若a也是正;刖ax可

了解爲丿宁 al,卽a的p幕的正q方根。很自然我們可把這
p

槪念推廣到當x取其它値的時候,例如當x=x/2-, X=7T或x取
任何的實數値時,使這個函數也定義,推廣至包括下述情形:若
x = W,函政的値也如上述一樣。這樣我們便得到了指數函數
y
y y=az(0<a<l) y=ax(a>i)

y=logq(a>l)

-X
0 (0,1)
(1.0)
0
y=logax
(0<a<l)
圖11

-13 -
y=axo對數函數x=loga y,當y>0時是指數函數的反函數。
在笛卡兒座標中,這兩個函數的表示如圖11。

習題1
1).描述y=x3的圖象,並用這個圖象,不再作別的計算,描述y= / x
的圖O
2).描述下面各函蚊的圖:
a) . y=sin2x e) . y=sin (x+兀)
b) . y=5 cos x f) . y=3 cos (x+0.6Q
c) . y=sin x+cos x g) . y=5 tan x—7x
d) . y=2 sin x+sin 2x
3)・一自由落體在t秒內跌落了 16t2尺。現有一皮球,從一25尺高的窗
跌下,把這球和地面的距離作爲t的函數描述出來。
4).現有一函數定義於一實數集2<x<pjLo它的對應規律是t(x) =
(x-2)(8-x)。a).求 6 與一 1 的象(函數値)。b).求f〔f(3)),
並把這函數的圖象描述出來。
5) .現有一函數定義於全部實數王 。它的對應規律是
f(x) = l,當X可寫成专;p與q是正整數,
=0,當x不可寫成史-時。
q
求著,一5, 1.41423, /歹的象,b)描述這函數的圖象。
6) . £).作出在習題4中的函數的反函數。b)・找出這個反函數的表
示式,並說明這反函數爲何不是單値的。
• 14 •
第2章 極限,連續函數、微分導數
在第1章裏,我們簡單的解釋了集和函數的槪念。在這一章
裏,我們開始涉及較困難一點的問題,那就是極限(Limits)的
槪念和函數的連續性(Continuity)、可微性(Differentiability)。
在章末,我們引進微分系數(Differential coefficients)的定義並
舉實例加以說明。在下一章,我們將詳述微分法的各種規則和它
們在其它自然科學和商業上的應用。

1.極限(Limits)

極限是數學裏核困難的槪念之一。直觀上來說,我們很難了
解怎樣無限的接近一點或一個數値。事實上,'額限”却常常出現
於日常生活的非數學思維中。舉例來說,我們說及一個機器的最
大生產率,或一間工厳的最大爭貨率。這些都含有極限的意思。
因爲最大的生產率和出貨率都是理想的意念,只可能無窮的接近
而永遠不能達到。同樣,我們計算一個電子管的最佳操作條件,
或怎樣才是最佳的市塲從而獲得最大的利潤等等,也含有極限的
-15 -
意思。
在數學裏,要了解微分法,我們就得先了解極限的槪念。在
下面,我們將嘗試作詳細的討論。
考慮-函败f(x),使其變數値在一給出的常教a附近取値,
函數的對應規律便給出一相對的函數値集。設當x接近a時,對應
的函數値f(a)是接近某一常數A。並且,我們假設此函數値f(a)
和常數A之差可無限的減少,假若我們能使x足夠的接近a,並
且,上述的情形是對於x與a之間的所有數値都是眞的(x=a可
能除外)o這樣,我們說當x趨近a時,這個函數f(x)取A爲
極限。更準確的來說,我們可把這個定義叙述如下:
我們說一個變數x趨近於一常數a爲極限,如果當x的値變
動時而|x—a|是小於任何給出的正數(這裏| x-a |表示x與
a間之差的絕對値),不論這個正數如何小,我們用下面的符號
來表示:
min x = a 或 x—a
【例】 如果x取下面的一列系値:
J_ J_ _7_ 15 2n-l
2,4,8,~16~, ........ 2n * ........
則x-l (卽X越來越接近1 )。但如X取下面的數値時:
丄 3 7. 15 z
•••(一1)〜
2, _石,百
8,, _~IF,‘(T) .
則x不趨於任何極限。
若變數x趨於a而又不等於a時,函數f(x)接近常數A不
論x怎樣迫近a,我們說當x無限的接近於a函數f(x)取A爲
極限用符號來表示:
-16 -
lim f(x)=A,或 f(x)->A 當 x->ao
x —> a

例如,f(x)=2x + 5,則 lim f(x)=5,因爲

f(I)=2(l)+5 = 7 般一!)=2( — 1)+5 = —2 + 5


=3
=6 f(-+)=4

仙= 5 土 ,(-土)= 4t
49
f(而) =510 f(-焉) 4 50
490
f (扁)=5-500 500

下面的定義關於函數趨近於a爲極限是相當於上面所給出過的定
義。
我們說當X接近a時,函數f(x)取A爲極限,如果當x足
夠地接近a而又不等於它時,f(x)和A之差的絕對値能少於任
何給定的正敏。或當x接近a時,函败f(x)取A爲極限如果每
當給出一個正數£時,我們能找到另一正數6滿足下面的條件。
0<|x — a|< d , |f(x) —A|< €
在上面討論過的極限不一定是有限大的(雖然無限大這槪念
是不容易下定義的)。變收x和函數f(x)也可以變得任意大或任
意小。這種極限我們在下面定義:
如果當變數x的値變得充分的大時,函數f(x)和常數A的
-17 •
差的絕對福是當少於任何給出的正數,我們說當x趨於無限大
時,函數f(x)取常數A爲極限。用符號來表示:
lim f(x)=A 或當 x—8,
X ―> OO

【例】 如果f(x) = l —十時



,則Xlim
—>oo
f(x) = l,因
f⑴=0
f(5)=4/5
f(20) = 19/20
f(100) =99/100
f(1000) =999/1000
f(10000) =9999/10000

同樣地,當變數X接近負無限大時,函數f(x)也可能取一
定的極限。用符號來表示,我們有
lim f(x)=A'或當 x—— 8, f(x)->A'
x—>-oo

【例】如果f(x) = l—十,則 lim


X—►- OO
f (x) = 1

又如當變數是很接近a而又不等於a,函數f(x)的値是大
於任何給出的正數時,我們則說當x迫近a時函數f(x)趨向於正
無限大(或沒有有限的極限)。用符號來表示:
lim f(x) = 8 或當 x->a, f(x)-*8

同樣地,當f(x)的函數値小於任何負數時,我們說f(x)趨向於
負無限大。用符號來表示:
-18 -
lim f(x)=-8 或當 X—a, f(x)—8
I
【例1 若f(x) 7^=2) 則临 f(x) = 8

其它幾個可能性,我們不作詳細的討論,只在下面把它們列
出來;
lim f(x) 或當X->oo ,f(X)~>8
X—> 8

lim f(x) 或當 X-*8, f(x)f — 8


x—> oo

lim f (x) 或當X—— 8 ,f(X)->8

lim f (x) 或當 ,f(X)—_8

【例】若f(x)=x< — 4,則
lim f (x) = 8
x—► oo
與 lim f (x) = oo

若 f(x)=xS — 8,則
lim f (x) = 8
x—>oo
與 Um f(x)

在某種情况之下,一個函數可能取兩個不同的極限,取决於
變數是否從實數軸的左方或右方接近它的極根;在這種情形之
下,函效f(x)是沒有意義的極根,或它的極限並不存在。可是
我們說它的左極限或右極限是存在的。
如果當變數X是從實數軸的右方接近它的極限時,函數f(x)
取接近一常數AL我們說這個函數的右極限存在,用符號表示:
-19 •
lim f(x)=A* 或當 x-»a+, f(x)—A*

左極限的定義也是一樣。爲極限的存在是不能取决於變败
怎樣趨近於它的極限,所以我們說,一個函孜的極限存在如果它
的左右極限都存在並且相等,反之亦然。
【例】若f(t) 則
lim f(t) > 及 lim=-oo
t—*o+ t —o-

2.極限的性質
下面的幾點對於在實際計算函數的極限時是非常有用的:
如果K是一常數及lim f(x)=b與Km g(x)=c,則
x— > a : —> a

1). lim K = K

這就是說,常數的舟限就是那個常數本身。
2). lim [f(x) 士g&)] = lim f(x)± lim g(x)=b±c
x —> a X —► a x—> a

卽是說,二函數的和或差的極限是等於函數的極限的和或
差。這一點可推廣至任何有限個函數的和或差,卽
如果 Um fj(x)=a, i=l, 2, 則
lim (S. fi(x)〕= 2 Hm f〈x)=苴街
X—>a i=l i=l X—i=l

-20・
3). lim [f(x)・g(x)] = [ lim f(x)][lim g(x)]=b・c
x— a x—► a x—> a

卽是說,二函數之讀的極限是它們極限的積。這一點也可推
廣至任何有限個函敎的積。
如果 lim fi(x)=aj; i=l, 2, n,則
lim (ft tj(x)) = |T lim f\(x)=f[ 为
X-* a j=l i=l x—*■ i=l

lim f(x)
4). lim J-W
X—> I
這裏c乂°
limg(x)=§;這裏 0,,
x —> a

就是說,如果分母函數的極限不等於0的話,二函數的商的
極限是它們極限的商。
5). lim [f(x)]n = [ lim f(x)]n = bn
X—>a x—>a

就是說,任何函數的n次方的極限是等於它的極限的n次
方。
6). lim
X —> B
f(x) =n|/T

就是說,一個函數的正n方根的極限等於它的極限的正n方
根。
【例】 (理由)
1). lim (x3 -3x4-5)= lim x3— lim 3x + lim 5
x—>2 x—>2 x—>2 x—>2
(2)
= (lim x)3 — 3( lim x)+5 (1,3,9)
X —2 X—>2

-21 -
=2,—3・(2)+5 = 7
些K普(x+2)
2). xUm
—>2\
f次2讨亍+2)= lim (x2+l) (1,2,345,6)

_22・讨习=鱼
4+1 _ 5
3). lim (-^+ 1)=2 (1, 2, 4, 5)

4) . Jii%( 1 +~^~) = 8 (1, 2, 4, 5)


X2+1
5) ・地(芸芸「)=2 X? + X+1
Cl, 2, 4, 5)

6). lim ( 4 亠' =4 (1, 2, 4)


X + 1丿
兩函數的極限可能是关。這是所謂不定式
當我們用4)時,兩函數的極限可能是导
(Indeterminate form),在下面幾節,我們將會詳細的討論。但常
常我們可以把分子和分母的一個公因子約去,餘下的式子便有有
意義的極限了。
—4
【例】若f(x) = Xx2— 0
2 '則蜩g) 石‘ 是個不定

式。但若X尹2時,我們可用X-2去除分子及分母:
%)(狩2丄,則心=岂
[X
則 f(x)
乙) 入 厶
我們應記住Hm f(x)不是函數在x = 2的値,而是當x充分地接
X—2

• 22 •
近2而又不等2時函數f(x)所取的値。當x充分地接近2時,
lim f(x)=lim
X—>2 X—
(x + 2)=4。所以,雖然f(x)在x=2時是不定義

的,但它是有一定的極限。
同樣地,不定式爰將在下面幾節詳細的討論,可是,同樣的
用上述的方法,我們也可把有意義的極限求出來。
【例】若f(x) = 2xxS34-x2-3 ,則lim f(x)是不定式
+x+2 X—>OO

爰,怛是我們若用对去除分子和分母則得出
3
f(x) = 2对+入2—3
2+土
K+x + 214 2

所以,
1
Hm f(x)=2, 因爲 x->oo 時,京——°
2
X—>oo

與令3

習題2

試計算下列極限:
1). lim 5x4-2
x-1
— a。
15). lim Xx11— a

-23 -
2) lim -[xn —an
16). lim
談a xm—am
x2-i x2—2x—8
3). lim 17). lim
x-*l x-l x2+6x+8

X-+1
4). lim
x- >CO X2-l 18)・ lim X,一 3x+2

x2-4 (x-l)4—l+4x
5). liia 19). lim
x->2 M2—2x X2

6). lim X。-9 20)・ lim I


x—>-3x+3 X->-l 2^5

7). lim x3—8 21)・ lim 一2


x->2 4-x
IQx
8). lim
x2+2x 22) lim !§
x—*o x->i x+1
x3+l (3x-1)(2x+3)
9). lim x 2—1 23). lim
->-i X—8 C5x-3X4x+5)

10). lim x5—x4 24). lim 3x • 2x


x— i x—・8 x^I^x+T
HZ lim
x->2 岩 25). lim e*— I
若 x—*0- x L試證明
x3—a3
12). (a), lim . =1昭。V
V—n
x—o x
x3—a3 —e-b*
13). lim (b). lim =b—a
x—, x2-a2
xn—1
14). 這孩,a, b>0

• 24 -
3.函數的連續性
當我們定義Um f(x)時,函數般乂)在乂="的値是不給出的,
卽是說,這個極限决定於當x接近a時f(x)所取的値而不是决定
於x = a時的函數値。所以,lim f(x)不一定等於f(a),因爲後

者不一定存在。在特殊的情况下,Um f(x)也存在與及f(a)也存
在並且等於lim f(x)。這個情形在高等數學中有特別的重要性。
X—> a

我們說,函數f(x)在x = a點是連續的,如果
1) .f(a)存在,
2) . lim f(x)存在,
3). lim f(x)=f(a)。

注意,每當我們說一個極限存在,我們總了解這個極限是有
限的。雖然我們常常寫下列的式子,
lim A = oo
x— a

這並不等於說A趨近一實數8,式子只是說明當X接近a時,A變
得任意的大(卽大過任何給出的正數)。我們應記住,8並不是
一個責败,也不應被理解成爲實數軸上的一點,但是爲了方便起
見,上面的式子還是會時常出現的。
我們說一個函數f(x)在區間b< x <c上連續(或b< x VC
如果這個函數在虛間上任何一點都是連紐的。如果函數fO)在
-25 -
*=a點在不是連續的,則f(x)在x=a點上是非連續的。在座標
圖上,在某區間中連續的函數的圖象是不斷的曲綫。
非連續性可在下面三個情形出現:
1) .若當x-a時,函數f(x)變成無限大(正或負),我們
說f(x)在x=a時有無限的非連績性。
2) .若當x-a時,f(x)的函數櫃突然改變但仍是有限的,
我們說f(x)有有限的非連續性。卽是說f(a)是定義的但lim
f(x)是不存在的。
3).如果當X-a時,f(a)是不定義的,可是limf(x)是存
X—> a

在的,我們說f(x)在X=a點上有一個失點非連續性(Missing
point discontinuity) o

圖12

• 26 •
【例】,若f(x) (x—2)2 ,刖在x=2時,函數f(x)有無
限的非連績性如圖12所示。
1 則f(x)有有限的非連續性在
【例】若f(x) = (2*+1)
x=0點上,如圖13所示o.
y

圖13

x3—2x2—3x + 6-»則當x = 2時,f(x)是


【例】若f(x)=d: x-2 ,
不定義的並且是非連續的。但是,若X尹2時,
(x2—3). (x-2)
f(x) =
(x — 2)
及lim f(x) = l,所以,如果我們定義f(2)= limf(x) = l的話,
x->2 x—2

• 27 •
則f(x)變成一連繽的函數而它的圖象便是抛物綫y=x2—3,如
圖14示。

(2, 1)

l®14
(W,0)
CO,-3)1
f(x.)=x2-3

連續函數的下面幾個性質是很重要的 ;
如果f(x)及g(x)都在x=a上連續,則
F(x)=f(Q+g(x)
G(x)=f(x) — gjQQ
H(x)=f(x)fx)
.28、
也在x=a點上連續。如果g(a)尹0,則
f(x)
I(x)
g(x)
在X=a點上連續。
如果f(x)在區間b<x<c上連續,則
1) . f(x)在這區間上有一最大和一最小的値°
2) . f(x)從一値變到更一値時,它一定取此二値中的每一値
最少一次。
3) .如果f(b)和f(c)是異號的,則f(x)最少在一點上等
於0。

習題3
1)・在定義域內,x取什麽値才使下列各函數連續:
(a) f(x) = 4/ 101 +厂
1.
(b) f(x) = 10 "<*-3)2

(c) f (x) X2—1


(d) f(x)
1+cos X
3+sin x
1
(e) f(x) 10 - (x-3)2.
(x-3)2 > 若 xW3
0, ,若 x = 3
2).試說明f(x)=x2在x=2爲連續。
• 29 -
2x4-6x3+x2+3
3) •考慮函數f(x) = ' X—1
,它在X = 1是否連續?
4) .函蛟f(x)=x是否在任何一點X=xo連績?
X
5).考慮函蛟z O試說明這函數在(0, 0)爲非連續。
x-y

4・導數的定義

當我們討論綫性函數的時候,斜率被定義爲
m=.y2 二臣 △y
X2 — X1 △x
卽當一點在綫上運動的時候,斜率是這點的垂直距離(卽和X-軸
y

Ay
圖]5

-30 -
的距離)的改變和它的水平距離(卽點和Y-軸的距離)的改變
的比例。又從圖15看出,斜率m也是等於此直綫的傾斜角(卽
直綫和正X-軸所成的角度)的正切函數,
m = tan 0
任何直綫的斜率是一常蛟,卽随着x的改變,y的啓變率是一
常數,這情况是不論點的位置在直綫的哪一部分。但遍複雜一點
的曲綫來說,斜率是决定於點在曲綫上的位置。
切在曲綫上y=f(x),有二點,它們的笛卡兒座標爲(x】,yD,
(乂2, y2)o則連起這二點的直綫(叫弦)有一斜率爲
m sec Y2-yi .Ay
X2 —X1

若點(Xi, yi)固定不動,而點(X2, y2)却被依着曲綫上移


向第一點,則在一般的情形之下,隨着第二點的位置在變動,連
起這兩點的直綫的飼率也在改變。對於很多的曲綫,當第二點越
來越接近第一點時,弦綫的料率慢慢的改變而最後接近一個有意
義的極限値。當這種情形出現的時候,這個斜率的極限値我們稱
爲曲綫在點(Xi, y。上的正切斜率,或曲綫在點(X】,光)的斜率,
如圖16所示。
更準確的來說,如果當(x2,y2)随着曲綫y = f(x)上向(Xi,yD
接近時,正弦的斜率接近一個常數,則此常數我們稱爲曲綫y=
f(x)在(Xi, y。點的正切斜率,或簡單地稱它爲曲綫在(X】,yj
的斜率。用數學式來表示:
lim m.sec lim _Ay 曲綫在(xi, yi)的斜率
0 Ax
一條曲綫在一點上的正切的斜率又叫做此曲綫在這點上的第
・.31・
y

(xz,y2)或
(x+ ax, y+ Ay> Ay

圖16

-Ax
(X1.y1)或(x,y)

一微分導數(First derivative)o
通常當我們爲曲綫的第一導數下定義時,我們用(x,y)來表
示不動點的坐標〃(x+Ax, y+Ay)來代表被移動的點的坐標。
如此我們得岀:
也=lim
lim 里=lim
lim • f(x+8—f忿
dx - &x—>o
A*—*o \x —>o
axax —>o Nx
爲函M g=f(x)對X的第一導數。上述的極限,對某些變數値X
來說可能不存在。在每點(x, y)上,如果谆個極限存在的話,
我們說函數y=f(x)在這點上是可微分的,或者函數y=f(x)有
一微分導败蓦。從原來給出的函數去計算皂的導數的過程我們
叫作微分。其它使用的符號如下:
-32 -
f'(x), y', £(y),Dxy,Dx(y)
、八」'dxJ' 、”
我們應注意到一函數的専數是一新的函數,可•能和原來的函
數有着完全不同的函數性質。從計算極限的規則,我們可得出計
算微分導數的規則。這點我們以後會詳細討論。在推出這些規則
之前,我們從導數的定義開始,計算幾個簡單函數的導數。它的
步驟如下:
1) .考慮函數y=f(x),
2) .使變效値和函數値從X, y增至x+Ax, y+Ay,則有
y+ △y=f(x+ Ax)
3) .兩式相減,便得函數値的增値。
△ y = f.(x+ Ax)—f(x)
4).計算函數値的增値和變數値的增値的比例:
△y f(x+ A) — f(x)
△x 一 Ax
5).求這比例當Ax趨於0的極限:
Ay lim f(x+ A)—f(x)
lim
Ax —o △x A*—*o △x
【例】求y=4x+i的第一導數。
解: lim
f(x+ △x)—f(x)
a*—>o △ X

4(x+ A) + l —4x—1
lim
AX—>o Ax

lim 4 Ax lim 4 = 4。
ax—o △x A、一>o

• 33 ,
,注意y=4x+i是一綫性函數,故其斜率是一常數,
【例】求y = x,+12x + 3的第一導數。
解: dy lim f(x+ Ax) — f(x)
dx Ax- 0 A~x

lim (x+ 小)3>< 12(X+ △x) + 3—x3—12X — 3


A*—>o

lim 3x2(Ax) + 3x(A)2+(Ax)3 + 12(Ax)


AX->0 ~ ~ Ax 一

lim [3x2 + 3x( Ax) + ( Ax)2+12]

= 3x2X 12

注意:星是個新的函數。
【例】 求y=x2+W的第一導數,若x尹0。
解: dy =]lim
赤 f(x+^x)-f(x)
dx ax—>o
»o △ x

lim
(X+ 3)2 -+
x+ Ax xT
a*—o

2x(Ax) + (^x)2+ x—x— \x


lim x(x+ Ax,
a、—o Ax

lim
a*—o
2x+Ax x(x+ Ax)
]
I
• 34 •
=2x 令若x尹0。
dy
注意:在x = 0時, dx
沒有定義。

習題4
計算下列各函數的第一導數:
1). y=3x2 8).

2). y=10x+3 9). y• = x+F


Y -i-o

3). y=-2x+5 10). y=4^3

4). y=5x2—8 ID. y= ax+b


3 2x+3
5). y=± 12). y=

6). y=ax2+bx+c I
13). y= x2+a2
7). y^2 14). y = /3x—1

• 35 -
第3章微分規律及微分的意義
在這章裏,我們首先詳細地說明微分的規則,然後解釋高次
微分導收和微分(Differential)的物理意義。在下一章裏,我們
將研究微分導數在物理;化學和商業上的應用。再往後,我們便
討論極大和極小的問題了。

1.微分規則
在實際的計算中,公式:
f(x+ △x)+f(x)
f'(x)= lim
AX—>0 △ X

是非常不方便的。所以,我們需要一系列的微分規則來簡化計
算。在這一節裏,我們將討論多項式函數,代數函數,對數函
數,指數函數,三角函數,反函數和複函數的微分規則。
a)多項式函數(Polynomial functions)
把有限個幕函數加起來便得到一多項式函數。如
f(x)=x4 + x' + 2x,

• 36 •
g(x) = 8x3+1
h(x) =4x
i(x)=6
多項式函數的導數可用下面的公式求出:
公式1:常數的微分導數是零:如y=c
則 =0

【例】若y=6, =o; y=o; =0

公式2: 一個常數和一可微函數的積的第一導數是等於此常
數和這可微函數的第一導數的乘積;即,若y=^cu(X),則u(x)
是一可微函數:
c
du
dx
【例】如y=6x\ = 6(4)x3 = 24 x3

如 y=iox, 10 (1) xi* = 10・x0 = 10

公式3:有限個可微函數之和的第一導數是等於這些函數的
導數之和:如y=u+v,及u = f(x)與V=g(x)是X的可微函數:
_ du
+] dv
dx — dx dx
更普遍的來說,如果y= i Ui,其中Ui=fi(x)是X的可微
i=l -
函數,及i=l, 2 n,則
-37 •
n
dui
2
i=l dx
【例】如y = 3x2 + 4x + 2;則
=6x + 4
若 y=x8 + 6x5+x〈+x2 + x,則
dy = 8x7 + 30x4 + 4x3 + 2x+1。
dx
總的來說,一個幕函數ex”是用下面的公式去求得:
(cxn) =cn xn-1
dx
而一多項式函數的導數可用下面的公式求得:
金(豎® )=£i=l
,Ci Bi X ni-l
i=l

b) 代數函數(Algebraic functions)
這一節的公式對乘積、商、專和根都通用。
公共4:二可微函數的積的第一導數,等於第一函數和第二
函數的第一導數的乘積,加上第二函數和第一函數的第一導數的
乘積。卽如y=uv,若u = f(x), v=g(x)都是可微函數,則
dy dv du
=U +- V
dx dx dx

一般來說,如果 y=TT uif其中m=fj(x)都是x的可微函


i=l

數,i = l, 2 則
-38 -
-sr(EiUi) =it[f,Cx)fluj]
j#i

【例】 若 y = (x,+ 4)(x+3),則


dy , 、
*(UV)=U'dy . du
*(uv)=u dF+v~dF
du = 3x2 + 0
及 dx
dv = 1(X1T) = 1
dx
dy = (x3+4)(1) + (x+3)(3x2)
dx
=x3 + 4 + 3x34-9x2
= 4x,+ 9x2 + 4
【 例】 若 y = (x + 3)(2x + 3)(x2 + l)則
dy =(2x+3)(x2+1)+2(*+3)(x2+1)+2x(x+3)(2x+3)
dx
= (4x + 9)(x2 + 1)+2x(2x2 + 9x + 9)
= 4x3 + 9x2 + 4x4-9 + 4x3 + 18x2-t-18x
= 8x,+ 27x2 + 22x + 9
公式5:二可微函數的商的第一導數,等於第二函數和第一
函败的導數的乘積減去第一函數和第二函數.的導數的乘積,然後
全被第二函數的平方去除。卽,若y =号。其中U = f(x)與Vrg(x)
是x的可微函败,則 V
通-u V2
dx

-39 -
x2 —4X+ 1
【例】 若y x—6
,則 u = x2 —4X4-1,

V=x—6,
du dv
= 2x—4;
dx dx
du = (x —6)(2x—4) —(x2—4x+l)(l)
及 "dy (x—6)J
2x2—16x + 24—x2+4x _ i
(x —0)
(x2-12x4-23)
= (x—6)"72
A .
【例】 若 y=y;則 u=4, V=x6

du =0; dv
*=6x5
-j— = 0; —j—
dx ' dx
dy (x6)(o) — 4(6x5)
dx (x6)2
24x5
=_ X12一

24

公式6: 一可微函數的n次幕的第一導繋,等於這函數的第
一導數和它的n — l次幕及(n-1)的乘積。胡如果u=f(x)是一
可微函蛟,及y=un,其中n是一實數,可正可負,可以是整數
或是非整數,則•.
蓑=顽-吧m nu

特別情形:若U=f(x)=x,則un=xn,及
-i

-40 -
dy 二 nu -1 =nx n-1
dx -
其中n可以是任何實數。
【例】 若 y=(x2 + 3)3,則
】3(x2 + 3)2(2x)
= 6x(x2+ 3)2

【例】 若 y=(x+3)T,則
dy = _}(x+3)4
dx

-y(x + 3)4
【例】 若y=(渕 ,刖
辛dx(、
-4^- )丿 nu n~1^-
(uun")=nu -i
dx

在這例中, u=当,故可用公式與得出塞
cdu (x — 1) — (x+1) 2
dx — 丫—1
((X-1) (x-l)2

所以 dy
dx
=2
渭)J
4(x+l)
-2
2

(x-l)3
【例】 若 y=(x+i)2(x2+i)-s
dy - 、 dv du
(uv) —U -dT 土 V
-dT(uv)==u
在這例中,U=(X+1)2, V='(X2+1)・3,故可用公式6來得
• 41 •
dv
出 ar必
及 ar 。
= (x+1)2(-3)(x2+i)-4(2x) + (x2 + i)・3(2)(x + 1)
= (x+1)(x2+i)-4〔一6x(x+I)+2(x2+i)〕
= (x+1)(x2+1)-4( — 4x2-6x + 2)
2(x + 1)(2x2 + 3x-1)
_ 3+1)4 -
【例】若y=(x2-x)-2,則
dy 2(*2—x)"(2x-l)
dx

=一2(2*—1) (x2—X)3
=2(1—2X)
为(x-l)。
【例】若y=x2(x+i)・i,則
dy = 2x(x+l)r + x2( — l)(X+l)-2
dx
= (x+1)・2〔2x(x+1)-x2〕
= (x+l)-2〔2x2 + 2x-x2〕
= (x+1)-2(x2 + 2x)
x(x + 2)

• 42 -
習•題5
求出下列各函數y=f(x)的第一粤數:
1) y=x2—4x+3 14) .y=(x2+3)(2x2-l)
2) y=3x3+7x2—9x+21 15) y=(x-l)(x +1)(x2+1)
5丄j
3) y —+4x
x 16) y=(x+l)(2x+l)(3x+2)
4 2
4)y=7+±・ x2
17) y=(px+q)(ax2+bx+c)
5) y=x(5—x+3x2) 18) /五(2x-l)(x2+x+2)
3x
6) y=8/T~ 19)广 x-1
丄 乂2+丄1 x 4-— x3+l
7) y 2
2 、十5 5 “十7 20) y X3-1

8) y=(X4-—1 )2 X(x+1)
x 21) y:
x2—3x+2
X2
9) y= / x + 22) y
Zx x2—4
10) y = — x +/ x ■-^― 23) y x+l
x+1
11) y=(x2—5x+2)(2x2+7) 24) y

y=(x2+i)(*x+l) 25) y y/~x +1


-1
X2+X + 1
13) y = (2x2-3)(3x2+x-i) 26) y X2 —X+l

-43 -
2x2—x+1 / X + 1
27). y 3x2+x—l 29). y
(✓Go3

28). y a2—x2
c)對函數數
在往後的討論中,我們都了解對數函數的底是
lim (l+4)n,卽我們除了特別聲明外,自然對數是用In
―> 8 U

來表示。
公弍7:若y = toga U,其中u = f(x)是一可微函數,則
dy loga e du
u- dx
x
【例】 若 y=iog 則(這裏對數的底不特別聲明,下
例亦同)
dy =loge( x + l—x
dx x l〈 X 丄1、2
(x+1)
log e
一x(x+l)
【例】 若 y = l/log x ,則
=〒(log x)
dx 2
log e
2xiZ log x
【例】 若 y=(log x2)3,則
dy
=3 (log x2)2
dx

• 44・
=6 (log x2)2 logx e
特殊情形:若對數的底爲e,則loge e=l,所以
_dy ]du
dx u dx
【例】 若y=lnT/5^=T,則
dy
dx
(宀)(现—尸 (2x)
X
3 — 1)
d)指數函數
公式8:若y=a\其中u=f(x)是一可微的函數,則
dy da
~dx au In a dx-
【例】 若y=2~*,則
dy = 2-x(ln 2)( — 1) = 一2" In 2
dx
【例】 若 y=iox2-x,則
dy = 10x2-x(ln 10)(2x-l) = (2x-l)10x2"xlnl0

【例 】若 y=ax xa,刖
dy ax(lna)xa + a x •a
dx
特殊情形:若y=e。其中U = f(x)是X的可微函數,則
dy n dU
dx ~dT

-45 -
【例】若y=x5 e-sin X=X2,則
dy =2x
dx
公式9:若y=u\其中U=f(x), v=g(x)是X的可微函
數,則我們微分m y=v in u,便得出
dv
vu -i du 4-uv In u
dx
【例】若y=x",則
In y = x2 In x
[dy ?=2x lnx+x2 (号)
y ~dx
=x(2 In x + 1)
dy = xx% (i+2 Inx)
dx

【例】若y (2x4-1)2 ,則 In y =21n(2x + l)


(x2 + 2)3
—3 ln(x24-2)
1 dy
y ~dx 2(2x4-1)-3(x22+2)
dy _(2x+1)(8-6x-8x2)
dx 3 + 2)4
e) 三角函數
公式10:若y = sin u,其中u=f(x)是x的可微函數,則
du
~3T
【例】若y = sin(x2+°),則

-46 -
—dy
p—=2x COS(x2 + 0)

若 y = sin2 3X,則
= 2(sin 3x) (cos 3x)(3) =6 sin 3x cos 3x

函败cosine,正切,正弦,餘弦和cotangent的導數都能從
公式10推出。
【例】 若y = cosu,其中u = f(x)是一可微函數,則
y=sin (孕—u),故得裏 =cos (3—u)((-会-)
dy =cos
dx
du
—sin u
~dT
【例】 若y = tan u,其中u=f(x)是一可微函數,因
u du +
(cos2u_^_ + sin-2u_^_
sin"11 )
sin u du
y cos u 及 dx (cos2u)~
du
dy ~dT du
故得 COS2U
sec,2u

同樣的,我們可微分下列的函數,
y=sec u
cos u

y = csc u =
sin u

y=cot u
tan u
總括來說,六個三角函數的導數如下:
• 47 -
du du
(sin u) =cos u (esc a) = —esc u cot uL-dT
dx 顽
du du
(cos u) = —sinu (sec u) =sec u tan u
~dT ~dT
du
一^^ [uui u)
(tan u= =sec
aw2 u
U-’dx" (cot u) = — CSC2 u
dx
【例】若y=(i+cot x)2,刖
-^-=2(14-cot x)(—CSC2 )

=—2 esc2 x(l +cot x)


f)反函數
公弍11:函數的反函數的導數是等於此函數的導數的倒逆。
若y=f(x),及x=g(y)是它的反函數,又設二者都是可微的,則
dy _±_ 或 'df(x)
df(x) __1_
dx dx >,.或
或¥一一 dg(y)
dy dy

【例】若x=y十 dy
+g~y5,求 玄
3
dx dy
14-y2+y4, dx
dy dx l+y2+y4
西
習題6
求下列函效的導數:
1). y=log(x24-a)

• 48 •
x2+2x
2). y=log/ 1 - x2 19). y=7

3) ・ y=iog 20). y=c««F.2

4). y=xcx 3
21). y=】og
T-ex
5). y=(4x2-7)2+/ x2-5 22). x 2)
6). y=log(ax+b) e'T
23). y= 3 + 1
1+X2
7). y=i°g 24)・ y=x? e>ax
1

8). y=log(x3 - 2x+5) 25). y =告每7一


2 '
9). y=logk (2x+x3) ex—c~x
26). y
e^+er
10) . y=x log x 27) . y=xn a*
11) . y=Iog x3 28) . y=xx

12) . y=logs x 29) . y=xv

13) . y=log (x+/ 1+x丁) 30) .y=x】。叱


14) . y=e” 31) . y=(log x)-^
log x
15). y=W5 32). y
b2+x2 33) . y=log (log x)
16). y=e
17). y= ae 34) . y=log (1滅区)

18). y=log(3 — 2x2) 35) . y=iog]/4=T

-49 -
36). y=log 40) . y = jZ sin
Zx2+1 +x
I 41) . y=tan^-
log X
37) . y X
42) . y=qsc 直
38) . y=sin x2
43) . y=sin 3x—cos 3x
39) . y=sin/ x
複合函數(Compositce functions)
如果y是u的函數而u又是x的函數,刖y是x的複合函
數。一複合函數對x的導數是y對u的導數和u對x的導數的乘
積。
公式 12:若 y=e<u), u=F(x),卽 y=(D〔F(x)〕=f(x),則
dy dy du
dx du dx
這又叫微分的鏈式法則- (Chain Rule)。
公式6, 7, 8, 9,及10是公式12的特殊情形。
dy
【例】.若y=u务,U = x2 4-1,求 "dx~
dy 2 2 I
du 3
u~i ~3
3+1)
du
=2x
dx
dy dy du 4x
dx du dx
33+1)
【例】 若y = ln(w3-l),及w=z2—l,求名

• 50 •
dy 3w2 _ 3(z2-i)2
du w3—1 — (z2—I)3—1
dw
=2z
dz
dy dy dw 6z(z2—l)2
dz dw dz Q2—1)3一1

習 題7
試求下列各函數的祟
dx
-:
1). y=sint/^x 10) . y=(畳T
2). y=sin2 x 11) . y=(y—2)/x2-4x+8
12) . y = / sin x2
3). y=sin (兀+nx).
13) . y = tan x sin 2x
4). y=tan(l—x) 14) . y=cot(5x+l)
]
15). y 1 + cos X
5). y=sin(-^~)
1—cos x
16)・ y 1+cos X
6) . y = / sin x
7) . y=a sin nx + b cos nx 17). y 1—tan x
sin2 x
8). y = Z i—X2 18). y 1+sin x
tan x—l
9). y 19). y
/ l+xs ~sec x~

• 51 •
20) . y=xv/-sin^' 23) ・ y = / 1 + (sin x)2

21) . y 24) .y*l—xT tan x


25) . y=(x2+4)4(2x3—3)3
22). y = +/M

微分公式的總結
■多項式函數
dy
公弍1:若y=c dx
=0

du
公艾2:若y=cu,其中U=f(x),刖 ~ar
公式 3:若 y=u+v,其中 U=f(x), v=g(x),則
dy du dv
dx dx dx
BH弋數函數
公式 4:若 y=uv,其中 u=f(x), v=g(x),則
dv
U +- v
dx dx dx

父式5:若 y 号其中 u=f(x), v=g(x),則

du dv
dy dx
—u )

dx V2

・52 •
公式6:若y=u\其中u=f(x),則
享=
dx
nu n-i毁
nu n-1
dx
dy
特殊情形,如果urf(x)=x,則un=xn,與 dx nu -i

=nx
■對數函數
公式7:若y=loga u,其中u = f(x),則
dy loga e du
dx — ~~u~ •"ST
dy 1 du
特殊情形若a=e,則y=ln u與 dx V-dF
■指數函數
云弍8:若y==au,其中u = f(x),則
dy =au In a du
dx ■dT
dy u du
特殊情形,如果a=e,則y=eu與
dx ~dF
公弍9:若y=uv其中u=f(x)與v=g(x),則
必=心-1.业
dx
vu -1
dx
4-uv Inu dv

今式8是公式9的特殊情形。
■三角函數•
父弍10:若y = sin u,其中u = f(x),則
dy
人=cosu
= COS U
du—
dx dx

• 53 •
從公式10,可導出若y = cos U,其中u=f(x),則
dy du
―—=—sm u-
dx
若 y=tan u 其中 u = f(x)
dy du
sec2 u-
dx l"dT
■反函數
公式11:若y=f(x)與x=g(y)是函數與反函數,則
dy_=_i_
dx dx
dy
■複合函數
公式 12:若 y=(P (u)與 u = F(x),則
dy dy du
dx du dx

2.高次微分導數(Higher Derivatives)

一般的來說,y=f(x)對x的導;數是一新的函數,所以我們
也可計算這新函數對x的微分。第一専數的微分導數是原有的函
數的第二微分珥數。如此類推,我們可計算原來函收的第三、
四 …次微分導收。
一般的來說,要計算函數y=f(x)的第n次微分導數我們可
• 54 -
把原來的函數微分n次。微分的結果可用下面的符號來表示:
dny f(n)(x) V(n) _dn
,f(n)(x), y< (y), D、ny, D、n(y)
dxn ' y 9 dxn
【例】若y = x3-3x2 + 2,則
= 3x2-6x

=6x—6

=6

=0

高次導數在某些败學和統計的理論問題的硏究中是非常有用
的,但在應用的問題方面,它們的用處却不很大。在研究比較複
雜的函數時,對於怎樣不用實際的計算才可得出它們的圖象的問
題,第二導數是非常有幫助的;這些在下面我們將會詳細討論。

習題8

1). y=4x3—6x2+4x + 7,求 d2y

2).y=x%求票
dx6
3) ・y x3 d4y
T^x ,求 dx4

-55 -
4)・ y=ax2+bx+c,求

5)・ r=sin a5,求—vrr


d3y
6)- y=x2 (x-l),求
~dx^
d3y
7)・ y=10x5—4x3+5x—2,求
日X,
d2y d3y
8). y
X
求 dx2 和 dx3
d^y d 3y
9). y = 求-s~2
dx2 和 无3
d2y d3y
10).y = ETT,求彩和黯
-dx2 dx3
d3r,
ll).r=tan0,求
d03
12).y=Iog(x + l),求 d4y
dx4

3•隱微分
對於像y=f(x)的函數,我們可直接用上面的微分公式來.
算 dy
dx
但是我們常常遇到的情形是:y和x的關係不是正接地
'o

表示出來,而是用一含有一變數的方程式如
f(x, y)=o
來表示。在這種情形之下,通常是不可能把方程式求解而把X與
• 56 -
y的關係寫成y=g(x)。這種方程式把y定義寫成X的函數,因爲
對於變數的每一種,y亦取一値使方程式得到滿足;所以我們說
這方程式把y定義成一 x的隱函數。從這樣的方程式中,我們可
用所設隱微分法來計算票。在這個方法裏,我們把y看作是一個
未知的x的可微函收,所以,上面推導出來的微分公式都可應用。
一般的來說,從隱微分法得出的 dy dx
是X和y的函數。高次導數
也可用此方法來計算。
通過方程式f(x, y)=0, y是被定義成x的隱函數,在計算
塞時,我們把方程式f(x, y)=0逐項微分並把y理解成x的
函數,微分後我們把票解出
dx 。
【例1 若xy2-x2+y = 0,把方程式逐項微分,得
2xy-^-+y
2xy-^-+y
dx
-2x 4+ 孚
22-2x
dx
=0

解此代數方程式,
dy =(2x—y2)
得 演 _(2xy + D
【例】 若x34-y3-3axy = 0o則逐項微分,得出
3x2+3y2! dy 3ax 3ay = 0
dx

卽 dy
dx
(ay-x2)
(y2—ax)

• 57 -
【例1 若x2 + xy+y2=l,求 dy
dx_,
dy =0
2x + x dy 4-y + 2y
dx dx
dy (2x+y)
則 dx x+2y
再求其導數,則得
(Py
dx2 〔0+祟~)s+2y盘翁 y)(i± 2喪)〕
用原來的方程式,可把上面結果化簡成
d— _________ 6
dx2 (x + 2y)' 'o

習題9
dy_.
用隱微分法,對下列函數求哥:
dx
1) .x2+y2=r?
2) . b2 x2+a2 y2=a2 b2
7)
•(w)y=】
8) . y2—2xy + b2=0
3) . y2=4ax
9) . x2—y2+3x=5y
4) .y3—3y+2ax=0
10) .x2+xy+y2=4
5) . x* + y* = a*
11) .3x2+7xy+9y2=6
12) ・(X2 + y2)2_ (x2 — y2)2 = Q
6).x才+ y

• 58 -
dO du
13).o=8s(e+e),求 14).cos (uv)=kv,求
dO

4.微分(Differential)

dy
在上面幾節,當我們討函數的導數時,我們並沒有把岩看
dy 成一符號的表示,它是當Ax趨近
作成一分數,我們只了解 "dT
於°時舟的極限
- :裝的極限。在某些問題中,我們往往可把dx及dy解釋
成二個不同的微量。這時,我們把dx稱作x的微分,dy稱作y
的微分,在討論積分法和函數的微最改變時,我們便會應用到這
種解釋了。
給出一函數f(x),設在變數的某一値上給出它的第一導數
f'(x),並設Ax是x的任意增量,則函數f(x)的微分df(x),是
用下面方程式來定義;
df (x) = f'(x) △ x = dy △ x
dx
若f(x)=x,則f,(x) = l及dx=Axo故當X是變數時,X
的微分dx是等於變數的增量本身Ax。
若 y=f(x),則
dy = f,(x) dx = dy
dx
卽一函數的微分是等於這函數的導數和變數値的微分的乘積 。
-59 -
在坐標圖中,考慮y=f(x)及設f'(x)是這曲綫在P的導數
則dx=PQ及
dy = f,(x) = (tan。)(PQ) =^-PQ=QT,如圖 17所示。

所以,dy或df(x)是相對於x的y軸增最。
y

圖17
}dy
P、 > Q
dx
0
0

a)微値法(Approximation)
在上圖我們看出,當Axh?。是十分地小時,是
近似地等於dy = QT。事實上,當變數的改變是很小的時候,函
數的微分是和函數的増値差不多相等。所以,我們往往可把這函
數的微分的値看作爲這函數的微增虽。
【例】 如y = x4 — 3x3 + 9x + 7,用微分來求5當x=1.997
時y的値。
-60 •
解:把1.997考慮成對變數x加微增量八x=dx=-0.003,
使其從2變至1.997。故
dy =(4x3 — 6x2 _|_ 9)dx
= (32—24 + 9)(-0.003)
= -0.051
旣然當x = 2時,y = 25及當x從2變至1.997時y的改變的近
似値是一0.051,故
y+dy = 25-0.051=24.949。
所以,當x= 1.997時,y的近似値是24.949o
要計算這個近似値的準確性,我們只需計算Ay-dy。
△ y = f(x+ Ax)—f(x)
f(x+ = Ax)4 —2(x+ △x),+ 9(x+ Ax)+7
=x4 — * + 9x + 7 +4x3( △ X)+ 6x2( △ x)2
4-4x( Ax)3 + ( Ax)4-6x2( Ax) -6x( Ax)2
-2(Ax)« + 9Ax
故 △y=4x3(Ax)+6x2(Ax)2+4x(^x)3 + (Ax)4—6x2(Ax)
-6x(Ax)2-2(Ax)3 + 9(Ax)
因 dy= (4x3 —6x2 + 9) Z\x,刖
dy— Ax=6x2( Ax)2+4x( Ax)s4-( Ax)4—6x( Ax)2
—2( Ax)3
所以,當x=2時
dy — △乂=(24—12)( — 0.003)2+(8—2)( — 0.003)2十
(-0.003)4 = 0.000107838081
所以 y+Ay=y + dy + (Ay-dy) =24.949107838081,而近似値

• 61 •
y = 24.949是準確至三位小數。
【例】.用微分的辦法,計算讨元的値。
數字98和100很接近而後者又是整平方。故當x從100改
變至98時,y=-|/Y的對應改變使y從1/ 100 =10變至l/98o
當 x=100, Ax= —2,
dx 2
dy = 0.1
21/100
所以 i/-98~ = i/I00_-0.1=9.9

習題10
1) ・設有函數y=x3+2xo
若 Ax=l, Ax=0.1, A'X=0.01。試求 Ay 和 dy
.試求下列各函數的微分:
2) . y=(5—2x+x5)<
3) ・)=5一#+ 2 —5x2

利用微分試求下列各式的近似値:
4) . sin 29°
5) . cos 151°
6) .,匸矽
7) .:log 11
8) . arc tan 1.02
9) . sin 60°3,'

• 62 -
10) . cOA
11) .試證明近似公式
Z a2+x (a>0),其中
|x|<a利用這公式計算下列各式的近似値
a) b) iZ 34 c) 1<120
12) ・證明近似公式:
' (a〉0)
n/an + X △: a + nan-1,

其中|x|<a,利用這公式計算
a) , b) 80 » c) 7Z 100» d) WZ1OOO
的近似値。

5.可微性與連續性
(Differentiability and Continuity)

可以證明出若函數y=f(x)在x=c有有限的導數:
f(c+ Z\x)(c)
f'(c) lim
A、—o ~ Ax
則f'(x)在X=C是連續的。
反過來說,函數在一點上的連續性並不保證函數在此點上有
有限的導蚊。卽
可微性其連續性。

• 63 •
就是說:一個函數在某區間上若是可微,在該區間上必是連續
的,但其逆不眞。
【例】一 f(x) = i/ x2在x=0點上是連續的,可是在這點
上,函數的導數並不存在,因
f,(x)=l 當 x>0
-1 當 xVO
如圖18所示。
f(x)

圖18

I例】f(x)=x*在x=o點上是連續的0,可是在這點上,
它沒有有限的導數,如圖19所示。
平滑函蛟(Smooth function)o如果一函數f(x)本身和其第
一導數都是連續的,我們稱此函數f(x)爲一平滑函败。一平滑
函數的坐標圖是一平滑的曲綫,此曲綫不但是連續和不間斷的,
它並有一連續的切綫。在下面我們將說明,平滑函數具有一些很
• 64 •
f(x)

圖19 x
0

有用的性質。並且這些性質對於連續的但第一導數是非連續的函
數是不存在的。
【例】y = i-x2是一平滑函數,因爲y與y'=-2x都是
處處連續的。(如果一函數在它的定義域上的每一點都是連績的,
我們稱此函數爲處處連續)。
【例】 函數y=(x—i)务+ 1不是一平滑函數,因在 x=l
dy _2 --------- 一 .— 一一
點上,它的專數 M y亡女二!是非連續的,如圖20所示。

-65 -
y

y=(x・i)/

(0,2)

I (1,1)

I
x

圖20

,66 •
第4章導數的應用
在這章裏,我們將討論函數微分導數的應用。首先我們該討
論它在數學上的應用,然後我們再說明它在商業和經濟學上的用
途。最後,我們將談到它的物理意義,並舉出一些在物理和化學
上應用的實例。在這一章完結之後,我們便開始討論積分法了。

1.遞增函數和遞減函數
(Increasing and Decreasing Function)

考慮一函數y = f(x)在點x=a上,因函數的f(x)對x的第
一導數是座標圖中代表y = f(x)的曲綫在x點的斜率,故"(a)是
這曲綫在x=a的斜率。
如果f'(a)是正的話,y = f(x)在x = a時是遞增的(卽當x
向正X-軸經過x = a點時,函數f(x)是在增加的);如果導數
f'(a)是負的話,y = f(a)在x = a點時是遞減的(卽當x向正X-
軸經過x = a點時,y = f(a)是減少的)。如圖21所示。
【例】f(x)=2x2 + 10, f,(x) =4x,旣然當 x>0 時,
• 67 •
f(x)

斜率爲負
函數逸減
斜率為正
函數避听
mi

f(x)>Q 與當 xVO 時,f'(x)VO,所以,.當 x>0 時,f(x)是遍


增函數,當XVO時,f(x)是遞減函數。
【例】f(x) = 3x3, f,(x)=9x2。因仃(x)是永遠是正的,
所以f對於任何變數X的値,f(x)都是遞增函數。
【例】f(x)=x3 + 6x2+i5, f'(x) = 3x2+i2x = 3x(x + 4)。
旣然當 x>0 或 xV-4時,〃(x)>0;當一4VXV0 時 f'(x)VO;
所以,當x處於區間一4ZxZ0時。f(x)是一遞減函數;當XV-4
或x>0時,f(x)是一遞增函數。
在上面簡單的例子中,f'(x)= 0都在我們討論範圍之外。在
下面,我們詳細地討論這點。

-68 -
2.局部的極大和極小
(Local or Relative Maxima and Minima)

設y=f(x)是一可微函數。現有一包含x=a點的區間;如果
f(a)是大於f(x)在區間內任何一點所取的値,則我們說y=f(x)
在此點上有一局部(local)的或相對的(relative)最大値:•相反
來說划果f(a)是小於f(x)在區間內任何一點所取的値,則在x=a
點上,f(x)有一局部的極小値。値得注意的這一函數的局部極大
値和極小値是它在一給出的區間內的最大値和最小値,•和函數在
y

區間aMXMb內
的絕對股大値
局部極小値
局部極大値

局部極小

■X
x= a 0 b

翻22

. 69 -
一更大的區間內的絕對極大値和絕對極小値是沒有任何關係的。
有時候,局部極大値可能是小於局部極小値的。如圖22所示。
設一函數f(x)在x=a點上取局部極大値或局部極小値,並
設其第一導數是連繽的,則f'(a)=0。在下面將討論到f'(x)是
非連績的情形。還有兩點値得我們注意的是:
1) 只有當f(x)和f,(x)在x=a點上是連續的時候,,我們
才可斷論f'(a)=0,如果在x a時,此函數取局部最大値、或局
部最小値。
2) 反過來說,就是f(x)和f,(x)在x = a點上是連續的,
我們也不能從f'(a)=0才斷論在這點上,f(x)取局部極大(小).
値。卽是說,我們需要f'(a)=0來使函數a在點上取局部極大
(小)値,但這個條件並不是充分的。

f(x)
X 1

f(x)=(x-l) +1
圖23

-x
0

-70 -
【例】設f(x) = (x-l)言+ 1,則
2
f,(x)=§ • T
(X— 1)3
所以,f'(x)在x=l點主是無限地非連續的。所以雖然函數
在X=1點上取局部極小値,但f'(l)尹。如圖23所示。
這例子幫助了我們了解上面所提出的第一點。
【例] 若 f(x)=x3,則 f,(x)=3x2 及 f,(0)=0。可是這
函數在原點上並不取局部最大•(小)値。如圖24所示。
這例子是針對上面所提由的第二點。
f(x)

圖24

-71 -
現考慮一函數f(x)在X = a點時,f(x)和f'(x)都是連續的。
很顯明的,如果f(a)是一局部極大値,當x向正軸通過a點時,
f(x)的斜率從正變到負;同樣地,如果f3)是一局部最小値,
斜率就會從負變到正了,這因爲遞增函數的斜率是正的;遞減函
數的斜率是負的。如圖25所示。
所以,如果要計算一函數的局部取大(小)値時,我們必須
採取下列的步驟:
1) 求方程式f(x)= 0的根,又稱臨界値(Critical values)o
2) 對於上面的每個根a,决定當變數向正X-軸通過a點
f(x)

斜率=0,局部股大値
負斜率,
f \函數遊減
正斜率, \
函數避地 \
負制不,
函敏遞減
斜率=0,
圖25 局部效小値
o

時f'(x)的符號的改變。
在x=a點上,f'(x)從+變到一。表示在這點上函數取局部

-72 -
最大値;f'(x)從一變到+表示f(a)是一局部最小値;若f(a)
的符號不改嚳的話,則在這點上沒有局部最大(小)値。
【例】 計算函數y = 2x3-3x2_i2x+13的局部最大(小)
値。
解:我們只需依上述的辦法來討論f'(x)=0的臨界値便可
以了。
= 6x2-6x-12

=0,如果 x2—x-2 = 0
(x—2)(x+l) = 0, x = 2,或一1
dy
如果一1VXV2, <0
dx 表示在x = 2時,函數取局部
如果x>2, dy 最小値。
dx >0
dy
如果xV—1,-^厂>0
,dx 表示函數在x=-l時取局
如果一 1VXV2時, dy <0 部最大値。
~dF
如圖26所示。
(例】依上例,討論函數y 讨(X2+1)
(如圖27所示)

dy L 3+1)切 身x(x2 + i)-£(2x)1


M -------- 0#TTj---------- J
(X2+1-X2)
(X2+1)奇

-73 •
f(x)

y=2x3-3x2-12x+13

(-1,20;

x
0

(2.-7)

圖26

1 矣0
3+1) 23
卽沒有臨界値,故沒有局部極大(小)値。
在上面討論的例子中,對於任何的變數値,函數的導數都是
連續的。所以,只有當f'(a)= 0時,在x=a點上函數才可能取局
・74 -
y=f(x)

y
x2+ i

@327

部極大(小)値。上面提過,當f'(a)是非連續時,在x = a點上,
函數也可能取局部極大(小)値。卽是說,如果函數f(x)在x=a
點上是連續的,它的第一導數在此點上是非連續(不論它的非連
續性是有限的或無限的),則就算f'(a)尹0, f(a)仍可能是一局
部最大(小)値。當這情形發生時,我們亦需討論當變數通過
x = a點時。f《x)的符號的改變。總的來說,對於這種函數,計、
算局部極大(小)値的步驟如下:
1)、求出f'(x)在甚麽地方是非連續而f(x)是連續的。這些
變數値,我們稱爲廣義的臨界値(Generalized critical values)o

-75 -
2)對於每一個廣義臨界値a, ••求上當變數向正X-軸通過
這點時f'(x)的符號改變;若f'(x)從正變到負,則x=a點上,
函數取局部最大値;.若f'(x)從負變到正,則在x=a時,函數取
局部最小値;若f'(x)的符號不改變,我們不能作任何斷論。
【例1 討論函數y=x§的局部最大(小)値。
袈=小
dx 3
尹0 (對於所有變數値)。
•孚
dx

—8當 X-0-;孚 8當x-0+;所以在原點上,第
x->0~;
田",dx

一導數的非連續性是無限的,如果XVO, 急V0;如果x>0,
-^->0;所以,在原點上,函數取局部最小値。如圖28所示。
y

-76 -
【例】 依上例,討論函數y=(i — x)言(2+x)§
dy 一2 (1—x)~§ (2 + x)i + (1 — x)j
dx 3 (T)
(2 + x)-身
一 (x+1)
(1—x)§(.2+x)言
=0,若 x= —1
dy
dx
8 當x—r和 OO 當 x—►!*,

並在x= 1處,函數有無限的非連續性,° dy
dx
-8若x接近一2,
所以在x=2處,函數亦有無限的非連績性。
y

y=(l.x 产(2+x)*
(T,声)
圖29

-x
0 (1,0)

-77 •
若一2VXV-1,
dy >0
dx. 表示函數在x= — 1處取局部最大値。
dy
若一IVxC-l, dx <0

dy
若 一IVxVl, dx <0
表不函數在X== 1處取局部最小値o
若 X>1, dy
dx
.>0

dy
若 XV-2, 呈 dx >0 故在x=— 2時,我們不能對函蛛
dy
若一2VxV — 1, -^->0
下斷論。
dx
【例】 依上例,討論函數y=(x—i)*(x+i)*。
總結關於第一導數能給我們的知識。考慮函數y=f(x)及一
點x=a在其定義域內,
f'(a)>0=》f(x)在x=a時是一 x的遞增函數。
f,(a)V0=〉f(x)在x=a時是一 x的遞減函數。
f(x)在x=a時是連續的,及
f,(a) = 0
若 或
f,(a)在x=a點時是非連續的
f'(x)在a的附近從+變到一表示f(x)在這點上取局部最大
値。
f'(x)在a的附近從一變到+表示f(x)在這點上取局部最
則 小値。
• 78 •
f'(x)在a的附近不改變符號表示函數在這點不取局部最大
或最小値。

3.第二微分導數
在這節裏,我們指出一函數的第二微分導數能用來决定此函
數在那裏是上凹,(Concave upward),在那裏是下凹的(Concave
downward)並找出它的拐點(Point of inflection)o
a) 函數的凹性(Concavity)
考慮一函數在x = a點上。在x點上y對x的第二導數是代
表y'=f,(x)的曲綫在這點上的斜率;所以,f〃(a)是曲綫y,=f,(x)
在x=a點上的斜率。
如果一函數y = f(x)的第二導數俨。)是正的話,y'=f'(x)
在x=a點是一遞增函數;曲綫y=f(x)在x=a上則被稱作上凹
(Concave upward)o反之,若第二導數f'(a)是負的話,刖 寸=
f《x)在x=a點上是遞減的,而函數y=f(x)在這點上是下凹
的。
考慮一函數f(x)並設函數本身及其第一導數f'(x)在x=a
點上是連續的。很明顯的,若fz(a) =。及f(x)在這點是下凹的
話,則f(x)在這點上取局部最大値;同樣地如果f'(a) = O及f(x)
是上凹的話,則f(x)在此處取局部最小値。値得注意的是對於
局部最大(小)値的存在,這個第二導數的測驗是一充分的、但
-79 -
不是需要的條件,因爲如果f〃(a)= 0的話,函數f(x)在x=a處
可能是上凹或下凹。卽是說,如果f'(a) = 0,則
y

斜率避圮・上凹
¥>。)

f〃(a)>0己上凹

o
岡30

斜率遮減,上凹
電vo)

f〃(a)vog 下凹

-X
0
圖31
• 80・
從上面的討論,我們得出一個很有用 雖然並不是常常都
通用的測驗,卽
函數f(x)和f'(x)在x=a點上是連續的
如果 與
f《a) = 0
f〃(a)>0=。在x=a處有局部最小値。
則 f〃(a) V0 —在x = a處有局部最大値°
/〃(a) = 0=>這個測驗不能用。
丄- C ..
【例】 討論函數y=W~x3-2x2
3
+ 3x+l的局部最大値和局
部最小値。
dy =x2-4x+3
dx
=0,如 x2 —4x + 3 = 0

y = -i-3--2xa4-3x4-1
(1,2斗) / /
圖32
(0,1)
(3,1)

-x
0

• 81 -
卽x=3, 1爲臨界値。
痒■=2x—4,所以,如果x=l, V0,卽在 x= 1.處,

函數取局部最大値。如果x=3 ,靂->0,卽在x=3處,函數取
局部最小値,如圖32所示。
【例】依上例,討論函數y=x」。
【例】依上例,討論函數y=x%
在上面的例子裏,當f〃(a) = 0時,我們必須找出當變數向
正X-軸通過a點時,第一導數的符號的改變。可是,看時候下
面列出的規則是比較方便的測驗:
若 f,(a)=f〃(a) =…=f《n-i) (a) = 0而f3)(a)尹0,則對於
偶的n, f5)(a)V0表示在x=a處函數取局部最大値;f<n)(a)>0
表示在x=a處函數取局部最小値。對於奇的n, f(a)不是局部
最大値,亦不是局部最小値。
[例】 討論函數y=3x4—x3+2的局部最大和最小値。
dy 12x3 — 3x2
dx
=0 若 12x3-3x2=0

3x2(4x—1) = 0; x=0,

= 36x2—6x = 6x(6x—1)

當乂=言時,溫
dx2 >0,所以,函數取局部最小値。
當x=0時, =0,所以,在此處,函數可能不取局部
• 82 •
最大或最小値。但,
d3y
dx3 = 72x-6
= 6(12x—1),
d3y
所以,x=0時,忐"VO,故此,函數在此處沒有局部最大
或最小値,如圖33所示。
y

y 3x4-x3+2

圖33 (0,2)

-x
0

b)拐點(Points of inflection)
我們說一函數y = f(x)有一拐點在x=a,如果在此點上它的
凹性改變。在座標圖中,在拐點上切綫不是和曲綫相接而是穿過
它。因爲第二導數的符號是指示曲綫的凹性的,故此第二導數的
符號改變時曲綫.的凹性也改變(所以,我們稱它爲拐點)。
-83 •
如果一函數在x=a處有一拐點而它的第二導數在這點是連
續的,則f〃(a)=Oc f〃(x)不是連續的情形將在下面討論。
我們應注意到下面二點:
1) 只有當f(x)及f〃(x)在x=a處都是連續的時候我們才
可斷論。如果x=a是函數f(x)的拐點,則f〃(a) = O。
2) 就算f(x)及f〃(x)在x=a處都是連續的,f〃(a)也不
表示在x=a處函數有一拐點;卽是說,如果f(x)及f〃(x)都在
x=a處連續,則f〃(a)=O是一需要的,但不是充分的條件使x = a
點是一拐點。
現考慮一函數f(x),設在x = a處f(x)及f〃(x)都是連續的。
當變數x通過x=a點時,如果f〃(x)改變符號的話,則在座標
圖中,代表這函數的曲綫在此點上有一拐點。這時,f〃(a) = O。
拐點亦可位於f(x)是連續,但任。)是非連續的地方。如上
述一樣,如果拐點存在於x=a處的話,貝」變數在向正X-軸移動
而通過a點時,第二導數的符號改變。
所以,如果要决定函數的拐點的話,我們可採取下面的兩個
步驟:
1) 决定在甚麽地方f〃(x)是零或非連續的,但f(x)是連
續的。
2) 對於每一個這樣的點,决定當變數通過a點時f〃(x)的
符號改變。.
f〃(x)在x=a點改變符號=> 這點是函數的拐點。
f〃(x)在x=a點不改變符號=)這點並不是函數的拐點。
[例】 討論函數y=x*的臨界値和拐點。
• 84 -
dy 1
-rz-=-z-x
dx 一 3
*0,對於所有的變數値來說,
dy
dx 在原點上是非連續的。
dy
若 'VO, -6
dx
->0
故在原點處,函數不取局部最大(小)値,
若 x>o, »0
dx
<1纣 =—
2 5
^-X~3
—§x-3
dx2
为0,對於所有的變數値。
dx2
是在原點上不連續的。
若 XV0, >0
dx2
故原點爲函數的拐點。
x>0, 骂
dx2
V0
如圖34所示。,
【例】 依上例,討論函數y=x++的拐點,和局部最大
(小)値。
【例 】 依上例,討論函數y=;(x3 — 6x2+9x + 6)
總結我們可從第二導數獲得的知識,考慮一函數y=f(x),
並設X=a爲其定義域內的一點,則
1) f〃(a)>0=>f(x)在 x=a 處是上凹的;

-85 -
圖34

2) f〃(a)VO=〉f(x)在 x=a 處是下凹的。


.3)如果f(x)及f,(x)在x=a處是連續的,並且f'(a) = O;
則f”(a)VO表示在此點函數取局部最小値;f〃(a)>0=>函败
在此點取局部最大値;f〃(a) = O則表示我們不能下斷論。
4)如果在x = a時函數f(x)是連續的及f〃(a) = O或f"(x)
在x = a點是非連績的,則f〃(x)在x=a處變號=》x=a是一拐
點。
5) f〃(x)在這點不變號則表示這點不是拐點。
利用第一和第二導數給我們的知識,我們可在座標圖中描出
一函數的圖象,這是不需要詳細的逐點計算。步驟如下:
1)計算殺及■&
1) 計算土=及苗
• 86 •
2)决定當變數取何値時■是正及何時是負。
(1纣
3)計算y和 dx2
在臨界値上的函數値,測驗這些値看是否
局部最大或最小値。
4)測驗在段
測驗在咨變成非連續的地方,函數是否取局部最大或
最小値。
5).决定當變數取何値時 dx2 是正及何時是負。
6)計算y及 =0時所取的函數値。看這些點是否是函
數的拐點。
d%
7) 測驗在 dx 2 非連續的地方是否有拐點出現。
8) 算出所有截距(Intercepts)。
9) 决定當|x|變得很大時曲綫的性質。
10)依照 及 黯的符號(前者指示出函數是遞增或遞
減,而後者指示出函數是上凹或下凹),所求出的截距,漸近的
性質(Asymptotic properties)和非連續性等把曲綫描岀來。
【例】 描出函數x=y—2y2 + 2的座標圖。
巫=1=4尸也
dx = l=4y
dy — 4y&
丄 qy 3-^--4y
dx 穷 dx
dx

4y(y2—i)

• 87 -
所以,二垂直地綫的切點在(2, 0)(1, 一 1)和(1, 1)
dx2 -4(3y2-i)
(1纣 〔4y(y2—i)〕2
如果yV—l,湍~>0
dx2
(上凹)
(1纣 <0 (下凹)
如果一ivyv- dx2
d 2y
dx2 >0 (上凹)

y x 1 x= 2

),y = y4-2y2+2

(1, 1)

[圖35

-x
0 ](2.0)

(1,-1)

• 88 •
如果ovy 混
dx2
VO (下凹)
如果 /y<y<1. ■&>0 (上凹)
如果y>i,4S-<o (下凹
dx2 7 \ I 丿)

依上述所得出的數據(data),可把曲綫描出如圖35。
4
【例】.依上例,討論函數y= x +—
[例】 依上例,討論函數y = 4 + 3x-xs

習題11
1).若y=x3-6x2+9x+l,求y的極大値和極小値。
求下列各式的極大値〜極小値。並指出x的相對値:
2)・ x2+x+l
4
7). x+ x+2

3). x3—3x+l 16.


8). x2+T-
x
4). x2(3 — X、 9).-J-X3-yX2-6x+y

5). (x+3)(7—x) 10).x(x + l)(x-2)


6-、
).1 --- 丄
“ *

試求出下開各式的極大値、極小値的座標。並描繪各函數的圖象。

-89 -
11).脾一3x2—6 14) . (x—a)2(b—乂)
12)・ 2xs+3x2-12x 15) . x+#
13). 20+3X-4X2-x3
16).證明(i) x/4x-*2在x=3時爲極大。
(ii) x2/ 9—x2 在 x = i^~6 時爲極大。

4.導數在商業上的應用
成本(Cost)、平均成本(Average cost)和邊界成本(Marginal
cost):如果出產x單位商品的總成本y只是x的函數,則總成本
函數可作如下的表示:
y = f(x)
許多不同的函數可用來表示總成本的曲綫,一般的來說,成
本曲綫有下面的性質;
1) 沒有商品產生時,總成本是零或正的 就是說,f(o)
>0。如果f(0)>0,則f(0)是生產的固定成本(Fixed cost)
2) 總成本永遠是x的遞增函數,gp r(x)>o
3) 出產極大量的任何商品的成本的増加率都是随着x的增
加而增加;所以,當x是很大的時候,總成本函數永遠都是上凹
的,卽是f〃(x)>0。但是在一有限的區間內,總成本曲綫常常是
下凹的,因爲邊界成本可以是負的。
如果說成本函數是:y = f(x)
刖平均成本或每單位商品的成本是:
-90 •
y_ 3
y = X X
及邊界成本(Maginal cost)是

= f'(x)

平均成本的第一導數是
应 _ xf'(x)-f(x)
dx x5 =0,只有當 xf,(x)-f(x) = 0
卽是 f(x)=xf《x),反之亦然。
所以,當X能使平均成本等於邊界成本時,平均成本就最低
了;卽是說,平均成本的曲綫和邊界成本的曲綫相交於最低平均
•* • ■ • • , , - •-*— — ,
成本的地方 ~
。因爲成本曲綫的第三個性質 ,—所以
— — -- ‘、dy °
,-我們設在豈=0,
可取局部最小値;當然對於一給定的總成本曲綫,我們可用普通
的辦法來檢測這個局部最小値的存在。
[例】 綫性的成本函數:
總成本:y = ax + b,其中a>0, b>0
・‘ 一 v b
平均成本:y =T=a+v
邊界成本: a

6
邊界平均成本: ,如圖36所示。
[例】二次成本函數:
總成本:y = ax24-bx + c 其中 a>0, b>0, c>0

-91 •
總成本平均成本

3 b
6 a + -
x

X
商品的數鱼

平均成本:y = ~^ = ax+b+与
J A

M成本: 2ax+b

M平均成本: a-■;如圖37所示。
dy c,或 x=±i/S。
dx
=0,如果 ax2 當然我們只需考慮
正。
解: =+竺
—十X4
2c
茜 >0,對於所有x>0.・.可取局部最

小値於 x= b

• 92 •


本 ax2+bx+ c

-x
\ 0 商品數量
-b.
2c. c-土
4a )

dx
2ax+ b
平均成本, 成本。

M
y ax+ b+
(0,b)'

-x
0 商品敏量

圖37
-93 -
我們注意到當 x=/S 時,平均成本和M成本相等,因
y=a +b+c 2V ac +b
dy = 2a|/-£-4-b = 2-/-iF~ + b

[例】 依上例討論下面的成本曲綫:
總成本:y = ax3+bx2 + cx+d,其中 a> 0, b < 0, c> 0,
d>0, b2<3ac
平均成本:y = y = ax24-bx + c ■+
x
dy = 3ax? + 2bx + c
M成本: dx

M平均成本: dy = 2ax+b
dx
【例】 依上例討論下面的成本函數:
總成本:y = axb + c;其中 a>0, b>l, e>0

平均成本:亍=¥ axb・i*

M成本: dy =ab
dx
dy =a(b-l)xb-2-
M平均成本:孚
dx

• 94 •
習題12

1) .如設某商品的總成本爲:
• T=0.4S2+3.8S+38.4
試求最低平均成本的値。
2) .某公司出版的錄音帶的總利潤爲:
P=-l.4x2+18.2x-38.4
求該公司的商品的最大總利潤。
3) . 求函效y=x3—9x2+15x—29在最大値和最小値時,x、y的數値。
4) ・某廠商每天生產某種產品一千件的總成本爲:
*x2+36x + 9800元,而這種產品的價格函數爲:

-0.01x2+0.3x4-260 元。
那末,對應於轉向點的每日產虽爲若干?並求這情况下的總成本、價
格、收入。
5) . •考慮下列各函败:
a ) X3—3x2+3x—8
b ) x3—6x2+7
c ) 3x4—4x3-12x2+5
d ) x5—8+ x4+26 x3—38x2+24 x—7
求這四函败的平均値,並檢驗它們是否有極大値或極小値。求出一切
拐點。
6) .某廠商每月出產某產品x公噸的總成本爲每月:
#x2+8x+4900 元。

據其公司職員作市况研究,指出在實際價格域範圍內,以價格爲每公
• 95 -
噸m元而言,每月可銷售号(628-m)公噸。假設該公司能够將全部產品
銷淸,試以(甲)最低平均成本,(乙)最大收入,(丙)最大總利潤爲基礎,
計算:
一) 每月產量,
二) 每公噸平均成本,
三) 每月總成本,
四) 商品價格>
五) 每月收入,
六) 每月總利潤。
7).某商行將以往紀錄作出週詳的調查研究和分析,找出僱用職員的人
數和新客生意訂單平均成本的關係 。假如x爲聘用職員的人蜘 則平均成
本爲:
+24x
x取什麼賊値才會使y變爲極小?試解釋這項結果。

5.不定式(Indeterminate form)的計算

在這節裏,我們應用導數來計算下列的極限:
妙5當f(a)吧或芝
計算這些不定式的基本規則就是洛必達•定律(L'HospitaPs
Rule)。定理如下:

• 96 •
f'(x)
如果f(a) = g(a) = O或f(a)=g(a) = 8及聖云 xj存在的

話,則XlimUM lim f'(x)


g'(X)
O不論a是有限或無限,這個
—> £■ g(X)

f'(a)
定理都是對的o如果lim
->a X—> I g'(a)
本身也是不定式,則我們再把
f'(x) Hm丄〃\飞,,如此類推。
定理應用-次,得出挡■咼
我們應注意到下面兩點:
,、i・ f'(x) , r d / f(x)
lim 一
1) x-*a
hm lim —.—I )£一
,\ 4■乂 x->a
g'(X)
hm dX \ g(X)
,卽是說,導數的商
不等於商的導數。
lim f'(x)
2) lim lim
lim
x—H 土涪"只是當
g (x) Ix->a g'(x)
本身
是一不定式,反之亦然。卽是說洛必達定理只適用於不定式,當
我們逐次的應用這定理的時候,第一個有定的答案是對的。一般
來說,在我們得到了有定的答案後還繼續使用這定律時,答案是
不對的。
如果其它的不定式可被還原至螺•及時,我們也可同樣的
使用羅比堵定理。試舉例如下:
1) 8。型的不定式
如果 lim f(x)g(x) = 8 ,其中 limf(x) = 8, lim g(x)=O,
x—> a x—> a x —> a

-97 •
f(x)
則lim
1 便是 oo
oo 類的不等式了。
2) 18型的不定式
如果 lim f(x)“x)=l 00 ,其中 lim f(x) = l, limg(x) = 8
X—> a x—> a "X—

則 lim log f(x) lim g(x) log f(x)=8.0,及


x— a

lim log f(x)


X—> c 便是音型的不定式了。
3) 0。型的不定式:
如果 lim f(x)g(x)= 0。,其中 lim f(x) = O, lim g(x) = 0,則
x—►a x—> a X—

lim g(x)
C—> 0 1 便是号型的不定式了。
log f(x)
4) 8。型的不定式:
如果 Um f(x)“x) =co*。,其中 lim f(x) = 8, limg(x) = o,
x—> a x~> a x —> q •


lim g(x)
0
1 )便是另型的不定式
0

log f(x)
5) 8 — 8型的不定式:
如果 lim〔f(x)—g(x)〕= 8 —8,其中 lim f(x) = 8, lim
x—■ x—► a x—> a

g(X)= 8,則

-98 -
1 1 1
lim
1
g(x) f(x)—g(x),

便是¥型的不定式了。
1/ 4 + h-2
【例】計算挡) O這是号型的不定式,故
lim V/4-rh-2 1電 K4 + B 丄
h— 0 1 4*
【例】 計算Hm XlnX。這是0-8型的不定式,故1血
x —► 0 >

lim limi—
hm —x
lim *(
XlnX X—>0 )=0
zv.

lim (*。
【例】計算 x—>OO 這是】8型。故
2
m(i+須
lim
x—>oo (T exp lim
x—> OO T~
X

2
mQ + 詞
lim
x—>oo i
T
2_
x2

lim
(i+4)
=2
—* oo
所以, lim (1+*) J。
x—>oo

【例】計算Xlim
—>0+
Xxo這是0。型,故

lim x: exp lim In x


X—>0+ >0+ ~T~

但 :」In x
:lim lim x lim xUl。
»—>o+ 丄 >0+ T =0,故 x->0+

【例】計算xHm I )
—0 x

這是8 — 8型,故lim ex —1—x


lim -
x—o x(e'_l)

lim
ex-l He -
x—o ex+xex —1 x—o (ex4-ex4-xex)

習題13
求下列各極限;
n.x-lim 3X—2X lim
3).】f x—sin
x3~
X

lim
2). x—>oo 渺e-A 4). x哓(号-海X)

•100-
e^ —1 lim f sin
5). xlim
—0
15). x-*o I
x
lim x+3
6)•凌limn log (cos 3戈)
(cos 2x)
16). X~*8 x log (
x-3
)

lim l+cos7rx 17). x聖)0(1 3


7). X—1 x2—2x+l
lim 3x2—x+5 lim (x+Z+e2*)}
8). x—>oo 5x2+6x 一 18).: 8
—3
lim
lim 19). x->0 (1
9). x-8 x2 e-x x2 tan2 x )
lim
lim
10) .X—>0+ X2 log X 20). x—>oo (l+2x)蓋

(cos x)& lim


11).x絶 21)・x—o+ (sin x)宓'
—>0

12).若 f(x) = (e3x-5x)~^-, 求 lim f(x)和 lim f(x)


X—8 X—0

lim [ lim e2*—2e*+l


13). x—>o ( sin2 x 22)
x->0 cos 3x~2 cos 2x+cos x

14). xlim
—o X^in x

6.中値定理(The Law of The Mean)

在這節裏,我們討論兩個很有用的定理,它們就是羅爾定理
(Rolle's theorem)和中値定理(Law of the mean)o 這些定理都
有蔑何的解釋,如果我們應用座標圖的代表函數的曲綫去理解它
•101-
們,這些定理都是頗顯淺的。
a) 羅爾定理
設一函數3 (x)在區間a£ x £ b內每點都是連續的,並設
此函數在區間的端點都等於0;卽
(P (a)= <p (b) = 0
我們再設0 (x)在開區間a<x<b內每點都有導數,則在這開區
間內,在最低限數一點處這函數的第一導數是等於零:
卽當avXVb時,(又)=0
證明:如果函數取常數値。時,這個定理當然是對的。如果
伊(x)不是常數的時候,則除。外,<^(x) 一定取其它的函數値。
從連續函數的性質,我們知道。(x)在此區間內取一最大値(M)
和一最小値m而其二者之中必最少有一個不是零。設這個非零之
最大或最小値出現在X,區間內的一點,則依據上面討論過的最
大和最小問題,0'(X) = O,因X是函數寧(x)的臨界値。
利用這個定理,我們可得出一個更强的結果:
b) 中値定理。
設一函蛟f(x)在區間內a£x £ b的每一點都是連續的並且
有一導收存在。刖有一變败値X存在;a<X<b,使
f(b) — f(a) = (b—a)f,(X)
我們可用幾何來解釋此定理。從斜率的定義,我們知道下面
的式子
f(b) — f(a)
b —a =f'(x)
是圖38中RS的斜率。上面的等式說RS是平行於在X點的切
綫。
•102 -
y

圖38

I
一 X
o b

證明:考慮下面的函數
f(b) — f(a)
V(x) (x-a)+f(a)—f(b)
b—a
它滿足了羅爾定律的所有條件,因爲9(a)=0(b)=e,所以,
它的導败
f(b)—f(a)
<P '(x) = f'(x)
b —a
一定在區間中的一點X等於Q。所以
f(b)—f(a)Z-P(X)
0= b —a 、

[例】 若給出 logio 291.40 = 2.46449o 求一個對"gio


291.35的近似計算。

•103 -
logioe 0.434294
設 f(x) = logio X;則 f'(x) =
讓我們取a = 291.35, b = 291.40,並利用上面證明了的中値定
理。結果如下:
0.434294
log10 291.40- log10291.35 = 0.05 X -»其中
291.35VXV291.40。所以,
0.0217147
logio 291.40—logio 291.35 = -x-
因爲X滿足了不等式:291.35VXV291.40,所以,上式右邊的
分數値是處於
與 。•。酔 = 0.000072
0.0^1 = 0.000075 與
之間,所以,logio 291.35和log】。291.40之差是小於0.0008但
大於0.0007,卽
2.46441 V logio 291.35<2.46442

習題14
1).若:(a) f(x)=x2, a=l, b=3,求 X
(b) f(x)=x3, a=0, b=3,求 X
f(x)=sin x, a=0, b =
(c) f(x)=sm =f 亠 X
— ,求
z
2).若函數0(x) = l —lxl,並定義在區間一 1匕x£l之間。如果取a=l,
b=l,討論這個函數是否滿足羅爾定理的條件。
•104 -
3) .利用中値定理來證明在一給定的區間內,若在此區間內任何一點f'(x)
>0,則f(x)是一遞增的函數。
4) .證明而一/~或是處於糸-與备之間。證明(1.00)务一(0.98)蓍
是處於0.03和0.03X(0.98)*之間。
5) .證明如果 log 10=2.3026,則 2.322< log 10.2<2.3226

7•弧長(Arc Length)

在初等幾何學裏,我們記得圓的周長被視爲內接這圖的一正
多邊形的邊長的極限。女n果邊的數目趨於很大時,則多邊形的邊
長接近圖的周長c這辦法也可用來計算任何連續曲綫的弧長,如
圖39所示。
設C爲一給出的曲綫,端點爲A與B。若A與B重合,則此
曲綫是閉合的。但以後我們不會一給定的曲綫是否閉合的,爲了
便利起見,我們稱A爲曲綫的起點,B爲曲綫的終點。在CJt,
依下列的次序Po Pi - Pn截出n+1點,並使A = P°, B = Pn;用
直綫把這些點連結起來:Pi Pi+1。於是我們便得到一折綫,起自
A終自B,並且頂角都在曲綫C上。在一給定的弧內,我們可作
無限多個這樣的折綫,因我們只需每次改變n的値罷了,卽使
n=i, 2, o在選定了一個口之後,折綫的長度便可被寫出;
L=P° P],----- Pn-l Pn
在幾何的直觀上來說,當n是充分地大的時候,綫段PiR+1便會
•105 -
圖39

變得對應地短,從而我們斷論,當口變大時,L的値不會有很大
的改變,而且,當口趨近無限大時,L接近一極限S。這個數虽
S,我們稱爲曲綫AB的弧長。當我們討論積分時,我們便會更
詳細的討論這個槪念,並指出S可用一定的積分來表示出。總結
來說,孤長PP'和弦長PP'的比例趨於1如果P'點依着曲綫向P
點移動的話。
現在我們用微分導數來把這個槪念表示出來。設有一函數
f(x),並設此函數本身和它的第一導數都是連續的。在座標圖
上,此函數的圖象是一連續的曲綫並有連續改變的切綫。使A爲
曲綫上的一定點,P爲A附近的一點,AP的弧長我們用S來表

•106 -
示。如果P的座標爲(x, y),其中y=f(x),則S是x的函數,
並且我們可研究隨着x改變時,s作爲對x的函數的改變。
現再考慮P附近的一點P《x+Ax, y+Ay),如圖40所示。
y

P'/

Ay

Ax
翻40

I
-x
0 x x+ Ax

若爲從P到P'的弧長,並選擇依曲綫從P向卩是S =
S(x)爲正方向。從定義:
ds
lim △s
AX—>0

As . pp7
不過 △x pp- Ax

•107・
及 (萱)七(里): 1,當△x->0
' PP,• \ PP,/

所以, lim
z—o (喪)*(喪) lim
ax—>0
/ PF \2
\ Ax /

怛從圖中,• pp> 2=(Ax)2+(Ay)2


所以, (罢)项+(瓮)J+⑵ ;當△x—O。

卽, (喪)'+僵
1 + )2
這個結果,我們又可用微分來寫出
(ds)2=(dx)2 + (dy)2
這個是往後在研究弧長最基本的公式。這公式可作多變數的
推廣。
像X—樣,S通常都被視作一代數量。一些問題如從那個起
點來測量S和依着曲綫的那一個方向S才是正的,都是任意决定
的。當然,我們首先要决定了那一個是正的方向才能斷定下面公
式中的那個符號才是對的;
ds dy
=± 1 +
dx dx
ds dx 2
~dy
=± 1 +
dy
如果在曲綫上給出的一點(X, y), s是隨着X而增加,則在
上面的第一個公式中,我們應取正號。若s是隨着X而減小,則
我們應取負號,對y的討論亦相同。
•108-
【例】 在一抛物綫上y=x2,决定s是從原點起最,並且,
若動點P(x, y)在第一象限時,s是正;動點移至第二象限時,
ds .
S是負。所以, ~dF >0o卽在這種情形下,
ds
=>Z 1 +4x2
dx
ds
但 的符號則依附於動點P的位置而定。如果xvo,則隨着y
ds ds
的增加,S代數地減小,所以, <0o但當x>0時, >0,

例如
ds
dy 1 +# 2 當》在(一,+1)時,
ds
及 W ]/ 1 +-4^2- = l/2_ 當 P 在(土,土)時。
我們可能給dx、dy、as一個很有用的幾何表示。對曲綫y=
f(x)作一切綫,使其切點在P。並依照測量S的正的方向在這切
綫上也取一正的方向,若切綫和正X-軸所成的角是刖
dy = tan (p
dx
如圖41所示,因爲
ds2 = dx2+dy2
所以,dx、dy、ds可被解釋爲一直角三角形的兩邊及斜邊。卽是
說,我們可用下面的方程式來表示它們之間的關係:
dx • ” dy
COS(P
dT;;sin(P —

•109-
y

dx
A

圖41

x
0

如果我們使△ x=dx,則dy及ds是Ay和As的近似,如果Ax
是充分地小的話。

習題15

若X爲已知,求下題各函效的 dx dy
dF 和-5ds~°
1) . 12y=33x—7x2, (x=2)
2) . y=x* , (x=l)

3). xy = 12, (x=3)


.、 1 . 一一一

4). y=^3 (x-3)Z~x~, (x=16)

5). y = tan x, ( x
6 )

•110-
6). 1
y=sin 7tx, (x =专)
3
dx
用x表達下題的二厂和小dy
ds、
7) y=-—%—
7)・ y=H
7 cos x
8) . y=x2
9) . y=#(x—3)/ x

10) . 6xy=x4+3 9
11) . a)若 4x2—y2=64,證明 ds=±y j/ 5x2—16 dx
2
b)若 3x2+4y2=36,證明 ds = ± 3x / 27 + y2 dy

8.質點在曲綫上的運動、速度的矢量性
(Motion of a particle in a curve
vector nature of velocity)

考慮一質點在x-y面一曲綫上運動,在曲綫上選擇一原點A
及一正的方向。用S來表示A和動點P間的代數距離。則s的算
術値是AP的弧長;並且,若從A走至P時,質點是依曲綫的正
的方向在運動的話,刖S爲正。第一導數 ds
dt 給予我們此質點在
曲綫上的速度。並且,導數的符號指示出運動的方向。這些和直
綫運動都很相似,但對曲綫運動來說,速度和加速度的槪念是比
•111-
較困難一點,所以,對此應多加解釋。動點P的速度是一有方向
的綫段,或矢量,用上面的方法得出。在P點上,,我們作曲綫的
ds
切綫,並在此切綫上,沿着運動的方向截出一綫段等長於 ~dT °
這樣得來的綫段我們稱爲P的矢量速度;並且用▽來表示。這矢
置的長度便表示速度的大小。卽
|研=|」 |
在解析幾何裏,我們已經定義過一矢最爲一有方向的綫段。
並且,矢髭在任何方向的分支是矢量在這個方向的投映 。我們可
以用矢最於三個互相垂直的方向的投映來描述矢虽本身。矢虽寸
在X-軸和y-軸的投映通常都被寫成V、及Vy,如圖42所示。
y

圆42

x
0

•112-
假設切於p的切綫的正方向(卽S代數地遞增的方向)求正
X-軸所成的角爲0,則速度矢景寸在軸上的投映爲
ds
Vx dFCOS ^,
cos(p V y ds--sin <p
Vy=K
因爲 ds2 = dx2 + dy2;所以,

(*)%(*)'(詞
爲了方便起見,使
ds
V = dt~ ,
則 V2 = Vx24-Vy
I例1 一質點的運動是依照規律x=2t\ y = t\其中t是.
運動的時間,求質點在t=l時的速度。
dx
解: Vx dT = 4t; Vy dy dt = 3t2
所以,|V|=i/i6t2 + 9t4
當t=l時,Vx = 4, Vy = 3, |V| = 5;運動的軸道是用方程式
X3 = 8y2
來表示。
[例】 一質點在圓周x2 + y2= 169上反時針方向地運行。
它的速率爲26尺/秒。當問質點在(12,5)時,V、及Vy是多
少?
解:從圖的方程式出發,得
2x dx dy
dT 4-2y dF =0

•113-
或 xVx+yVy = 0
我們知道V = 26 (我們已在此假設了反時針方向爲S的正方
向)
所以, Vx2 = v/=(26)2
所以,我們有兩方程式可讓我們來把Vx及Vy表示成x、y
的函效。得出
_2LV ・ V、2 +
Vy 七了=(26)2
y x'
V 2 -(26/y2 =(26/y2
-7x2-+y2) f69~
當y = 5時,Vx= —10尺/秒,所以,
x xr — 一 12,
一*( — 10) = 24 尺/秒。
y 5

習題16

1). 令s單位爲公尺,t爲秒,且若有
s=t3—9t2+24t,
求在t=5時的速度。假如質點瞬息間停止,求這時t的數値。
2). 設向上空抛擲石子的速度爲每小時60哩,小石在開始運動後t秒的
高度爲h呎,而
h=-i6t2+88
求2秒後、4秒後小石的高度。小石可升高達多少尺?
3). 質點P沿直綫運動,某個時間之後t秒,P離綫上一固定點s呎。若
s=t(10—t),那末,t=2至t=2+At這段時間內,P的平均速度
•114 -
爲何?從而推導在t=2時P的速度。
4) .某物體在t秒時離某固定點O的距離爲x公尺,而
x=-16t2+96t
a) .求t=2時的速度。
b) .該體離。的極大距雜°
5) .某物體沿直綫運動,選從下列公式:
x=i/ t + 4

距離以公尺爲單位,時間以秒爲單位。試求
物體在t=9時的速度和x的極小値。
6) .向上抛阕小石,小石運動離地面x公尺的時間爲抛擲後t秒,而且
x、t有下列關係:
x=-4.9t2+39.2t
求小石最高能升達多少公尺?小石朝上運動的速度爲14.7公尺/秒
時,發生在抛射後何時?又小石朝下以速度14.7公尺/秒下降時,是
開始抛擲後幾秒鐘?

9.加速度的矢量性
(Vector nature of acceleration)

在直綫運動中,我們定義加速度爲速度在一秒鐘內的改變。
在曲綫運動中,加速度是一矢量,對於一給出的笛卡兒座標系,
它的支量可以被定義爲在同一座標系中速度矢量的支量對時間的
第一導數。所以,如果用a來表示加速度,則
•115・
dvx d2x dv.
ax

W=B■;; ay
ay=-dt
通常來說,矢量在動點P的加速度矢量並不和曲綫正切。它的標
最値是:
d2x 2 2
a I +
爲了求Z在正切上和一垂直方向的分支,我, an, 我們在圖
43表示出這些分支和ax及ayo
y
ay a

ax

圖43 an \ I

ax

首先我們把a投射於切綫上,再次在法綫(Normal line)上,
便得到下面的關係:
at=ax cos cp +ay sin (p
an= —ax sin (p 4-ay cos

•116-
怛我們記得;
dx sin cp
cos <p,
~dT
所以,得出
d2x
~d^~ .
Ads
d2y
E・
d2x .dy d2y ____
an 一^d? , ds
+ dt2 . ds
第一方程式可被化簡:
首先 (告)2=(晋乒啓)
計至對t微分,便得
ds d2s ,dx d2x
2
顽----- d? 顽----- dP
_1_ dt
用牙 去乘全式,則得
d2s dx d2x (1纣
~^ = ds "dP
和原來的式子比較,、刖得
d2s
dP
所以,加速度沿着正切的支量是s對t的第二導數。
【例】 一質點的運動方程式爲x=a cos cot, y = a sin a)t,
其中/是一常數,研究這個運動。
解:運動的軌道是一圖,x2+y2 = a2。所以,
= —叼 sin cat
=_叼 cat, ao) cos a)t
dt ' dt

•117-
(静) =a2 u)2 sin2 a)t 4-a2 to2 cos2 a)t = a2 a)2

所以,它在軌跡的速度是一常數;v = aco,加速度的支虽爲
d?x
_ = —aa)2 cos a)t=d^v
—a)2
— 2 x; y=——a)a)2y2yo o
ds
沿着正切的支覺是零,因爲 常數。所以an是加速度本dt 7E'

身。它的方向是沿着半徑走向圓心。它的標匿値是
an = i/ a)4 x2 + a)4 y2 =cos2 a
v2
這也可被寫成a「= a 或v = aa),如圖44所示。

p
an

圖44
5 = Wt
0

•118-
我們應該察覺到,P在x-軸和y-軸的投射在進行簡諧運動。
特別注意到加速度的性質。當一質點用均速v進行圓周運動時,
V2
它的加速度是向圓心的,並且,它的標虽値是 ,其中a是這
回的半徑。
[例】 一抛射體沿着抛物綫y=x—瘟以每秒401/t尺的
均速來運動。
求v和at的一般公式°
解: 祟=(-高) dx dx
"dt" 401/2

所以, 僧)2=(蚩)*的
2 2

=〔】+("思)项受)
=3,2。0( 2—50
杀 + 示氣 )
--------------- -T-----------------

10,000
――

8
=赤(20,000 — 200x + X2)
所以,再進行微分時便得:
ds
2 dt
d 2s JA (-200 + 2X) dx
瑚= 25 dT
現知, V = ds (20,000-200x + x2井
dt ~5-
dx
d2s 8 (x-100) ^dT
卽, at =
dt^ _ 25v

•119-
= ______
(20,000-200x + x2)i
觀察一下,當x=100時,抛物體到達最高點。在此處, V = VX,
at = 0o並且,當抛物體在上升時(卽當XV10。時)如 是負的,
當它下降時(x>100), %是正的。

習題17
就以下各題,用s表示質點P離某定點。的距離,P、O均在一直綫
上,t表示某固定時間後t秒。按照給出的t値,求P的速度和加速度。當
P瞬時間停止運動時,求對應的時間t:
1)・ s=2 sin 3t, (t=0)
2) . s=3t3—9t2+12t, (t=3)
3) . s=t( —1+3)2, (t=2)
4). s=(2 cos ?rt) + 5 (t=土)
6
5) . s=t2(4—12), (t=l)
6) . s=6t+3 cos 2t, (t=0)
7) . 若s=t3-6t2+42t,求P最小的速度。
8). 物理學、力學常見到的公式:若 s = ut+ 1 gt2
2
_ds_
ML 這裏U、y爲常蛟,證明
v2 - U2 = 2gs.
若 s=4t + 2 cos 2t, v dv
9). E V = df證明
•120-
v2 =32v—4v2,
10).若s=a sin tot, a與s爲常數,證明
ds丄=±0)/ 一s2 ,
■df
d% —a)%
■d^

•121-
第5章積分槪念的引導
上一章總結了我們對微分的討論 ,從這一章起,我們開始討
論積分法。首先,我們把積分看成是微分的相反,繼之我們便簡
單的提出一些積分方法(有系統的積分公式留待以後才討論)和-
應用。在此我們引出微分方程。

1.反導數(Anti-deriVative)

如果y是一給出的函數,尋求■的過程叫做微分。現在假
想一下,如果是給出的話,我們可否求出y=f(x)呢?在很
普遍的情形之下,對這問題的囘答是肯定的,其過程我們稱作積
分(Integration)0
一函數F(x)在X-軸的某一區間上滿足下列的條件:
F,(x) = f(x)
被稱作f(x)在這區間上的反導數(Antiderivative, primitive) o
在下面的表中,我們舉出一連串的例子把函數和它的反導蚊並
•122 -
列。讀者應把右列的函數微分,看看是否得出左邊的函數。
f(x) f(x)的反導數
}x3 + c(c是任何常數)
5x6 -^-x77+
yx +cc
sin x —cos X + C

cos 2x -ysin
— sin 2x + c

1 -yin (2x+l) + c
2x4-1
從上面看出,一個函數f(x)沒有一個唯一 (Unique)的反導
收。如果F(x)是f(x)的反導數,刖F(x)及G(x)=F(x)+C都
滿足f(x)的反導數的定義,卽是說,F(x) = G,(x) = f(x),其中
C是一任意的常數。但是,如果我們能在X-軸的某一區間內找出
f(x)的一個反導數f(x),其它的反導數都全部確定了,因爲它們
都取F(x).+C的形式;卽和F(x)的差只是一任意常數。隨着C
的改變,我們便會得到全部的反導數了。所以,我們得出下面
的結論:
如果兩可微函數F(x)及G(x)在區間a<x<b內满足F(x)
= G,(x),刖我們可找出一常數C,使得在同樣的區間內,此兩
函數滿足F(x) = G(x) + C。
我們首要的任務是要能對積分的操作純熟。就是說,給出一
反導败,我們必須能馬上找出它的函數。爲此起見,我們用下面
的符號來表示反微分,或積分;

•123-
若 F,(x) = f(x),

則 f(x)dx=F(x)+C

在上面,我們把微分的公式用積分的符號寫出;
I). kdu = ku+C, C是任意常數;k是一給出的常數。
un+1 FC, a尹一1
H). undu =
n+1
du
HI a). In u + C,若 u>0
u
du
ni b).
u =ln( — u) + C:若 uVO
IV). .J cos u du = sin u + C

V). sin u du= — cos u + C

VI). J sec2 u au = tan u + C

VH). J cosec2 u du= — cot u + C

VIII). sec u tan u du=sec u + C

IX). J cosec u cot u du=—cosec u4-C

X a). au du 点a"+c; a>0, a尹 1

X b). J eu du = eu 4-c
u
XI). =sin-i u + c或 — cos -1 u + </
— u2

•124-
XII). J 1+U2 = tan-i.u+c 或 一cot" u+(/

如果要證明上述的任何一公式,我們只需微分公式右邊的函
蛟,看看是否能得囘積分符號裏面的式子。除了這些公式之外,
我們還有下述一些較普遍的定律。
XIH).如果F(x)是f(x)的一反導數,K是一給定的常數,則
kF(x)是kf(x)・的一反導數,用符號來表示:
Jkf(x) dx = kJf(x) dx
XIV).在一定的區間內,兩函數之和的反導數是等於函數
在同樣的區間內的反導數之和;卽
〔fi(x)+f2(x)〕dx =J fi(x) dx + J f2(x) dx
這個公式可被推廣至任何有任意函數之和 :
/ CScifj(x))dx=iciJfi(x)dx

【例】 用上述的公式,並設x>0,則我們得:
3 、卜
J ( 5x2 +x* + 7一宀 k)dx

= Mx,+ Mx3 + 7x—21n x —M + c

【例】依上例,說明:
) 3x4 dx= 4-c

b). | 7x-,dx=-gx-2 + c

•125 -
J 2x4志dx = 2 ■bC

d). (3 sin t—12 sec2 t)dt= —3 cost—12 tan t+c

_1 5 ^dy = -i-sin-1 y —5 ton"1 y + c


41/1 -y2 l+y2

2.代換法(Method of substitution)

這是積分運算最主要的技巧之一。它的基礎是複合函數的微
分法則。我們記得,如果①是一u的可微函數,而u又是一x的
可微函蛟,則複合函數的微分法給予我們;
d — ,、 d(D (u) du
①(u)
舉例來說,若<D(u) = i/Ryu=i+x2,則
1 1 +X2
■dF1/ 1/ 14-x2
xdx
所以, J izw+r =1Z 14-x2 4-c

現在我們用代換法來解釋這個結果:
在上式的左端,我們把 u=l+x2;而計算微分du = 2x dxo
則代換而得
xdx du
1+x, 21/=

•126-
再用公式II可得
U*
du 打u"du・=*・ +。=1/习 4-c
最後,我們再把U換成原來的式1+X2從而獲得上面的結果了。
在下面,我們把這個代換法寫成公式IV;
IV.現設需求一函數f(x)的反導數,並設。(U)爲一有已知
的反導數e(u)的函敕,若一x的可微函數g(x)存在並滿足下
列的條件;
如果 u = g(x)則 o(u)du=f(x)dx,

則(D〔g(x)〕是f(x)的一反導數。卽是說,我們首先計算J (P (a)dn,

在獲得結果之後再把u寫成g(x),這就是所需的Jf(x)dx了。

dx
【例】計算J l/16—x2
解:用代換x=4u;則dx=4du,及
f dx J 4du du = sin"x u4-c
1/16—X2 "16(1—u2) 1/1—112

在最後一步,我們用了公式XI。再把u換成专,我們則得到
dx
1^16-x2 =,卜信) + c
這就是要求的結果。
從上面這個例,我們可得到比公式XI和XII稍爲普遍一點的結
果;
•127 -
XI,) =sin
再令 = sin-i
-喘)+c 或-
— cos-@
cos )+</
成了要推導這公式,•我們只需用代換u=av,再用公式XI。同樣
地,我們可推廣公式XII:
du
XIT) a tan-1 (¥)+C 或—cot・i任)+</
a2-f-u2 —
【例】推導下列公式;
r du 1 ECri
a (告)+c 若 u>a
ui/ u2—a2
1
a cos广】(S)+c若u<~a
——
dx
解:我們用代換u § 則du=-a
首先,假設a>0,上面的代換給出
du __ f—
—22^1 dx
u a2

ax 「1 一 X2 J
從定義得出:/M=x若x>0; ”#=_x若XVO。所以,
如果x>0的話,用公式XI,我們便得出反辱數如下;
1 f dx 1 … , 5咔+ c
a 打力―— ’
a cos 1 ------
x+c
如果x<0的話,我們只需把最後的答案的符號改變掉。因爲ux=
u>0,所以,u和x的符號是一樣的。怛是,我們一定要限制
u>0或uv—a,這是因爲我們要使u2-a2是正定的。上述的公
式的推導便完成了。
【例】計算J 1 dx— 4x2
•128-
x
解: dx
5-4x2 "A — x2
-sin"1
2
4-c

2x…
1Z5
在上面的第一個等式裏,我們是用了公式XI,。
在這一節裏,我們只用了最簡單的代換,往後我們還會很詳
盡地和有系統地討論積分操作的技巧。
綫性的代換往往是非常有用的:
【例】求[(3-7x)2 dx
解:使 u=3-7x 則 du,-7dx
所以
du = — 2 |+ c
3
(3 —7x)2 dx
35
2 .
=一衣(3 - 7x)
35、

【例】依上例, 討論J e" *。

3.代換法的進一步練習
在計算反導收(或積分)中,我們必須很快的就能找出一個
適當的代換。因爲在高級的科學研究中,計算積分往往只是一個
極其複雜中的問題的一小節。所以,在學習初等微積分時,我們
•129 -
必須演算大量的例題,從而觀察到什麼樣的代換適合什麽樣的問
題。在這一節裏,我們說明一些常用的代換法。
現我們要計算下面類型的積分,
3+x2)n 乂火 J(X2—a2)n xdx
其中n不須是整數。爲此,我們只需使u=a2+x2或u=x2—a2。
在上面的積分裏,組合xdx是很重要的,如果只是dx出現的話,
問題就變得困難得多了。
【例】 計算 JxiZi6-x2dx
解:使 u=16—x2;則 du=—2xdx,
x/16—x2dx= ―yi/ u du,
3

並且[x 1/16—x々dx= — 勻—+c

=(16—x2+c)^
我們也可使U-1/16-X2,或讨=16~x2,則得2udu= -2xdx
1/16—
J x 1/16—x dx= — J u2du=—
x22dx= u3 + c
3 ~T

(16-x2)2+co

如果問題是計算 xdx
(x2-25)n ,我們可使u=x2—25或u2=x2-25

如果n是成的奇倍數,則第二個代換比較適合一些。
•130-
在某些常遇到的問題中,u = sin x是一個很有用處的代換。
【例】 J sin” x cos xdx
解: 使 u = sinx, du = cos xdx,則得
f sin^ x cos xdx= f u2du=4~u34- c =—(sin x),+c
3 3
很明顯的,這個代換對sinnxcosxdx類型的積分也適用,這當

然是不論n是否整數的。同樣地,類型如J cosn x sin x dx的積分


可用代換u = cosx, du= —sin xdx來計算。這些普遍的結果給
我們重要的特別情形:
tan xdx= —In cos x + c

J cot xdx=ln sin x+c


要推導上面的第一式,我們只需使u = cosx,則得,
tan xdx= sin xdx
cos x - J平
—In u+c= —In cosx +
C第二式的推導也是一樣。富然,只有當COS x>0時,上面的結
果才是對的。如果COS XV0的話,則我們得In( —cos X)。所以,
我們把上面二式統一地寫成:
tan xdx —ln|cos x| +c

J cot xdx=ln|sin x| +c

從此,我們把這二式看作標準的參考公式。在上面二式,我們加
•131-
了絕對値的符號,所以,它們不論cos X和sin x取任何符號都是
對的。在下面。我們舉出更多的例:
【例】a). (l+x3)^x2dx 代換 U=(l+x2)
ex dx
b).
l+ex
代換u = e' + l

J • *2
xdx
cos xdx
代換 U=-XZ

d).
(2 + 3 sin x尹 代換 u = 2 + 3 sin x
Ln xdx
e).
x 代換u = ln x
從上面的例,我們很容易的看出,只要小心的觀察符號
/< )dx裏的式子,就不難得到所需的代換了。
以下我們列出幾種常遇到的/分的類型。通常用下表的代換
式都很奏功效的。有時,同一數式可用超過一種代換。下表中用
粗字體的代換式,更較通用:
稍分類型 代换式
1). 含有1/%2 —乂2者 x=ain^, a cos ot a tan h。,
a sech Q
2). 含有/a2+x2者 x=a sinh 0, a cosech 0,
a tan 0, a cot^
3). 含有 1/\2—a?者 ―> x=a sec 0, a cosh 0,
a oosh 0t a cosh 0,
4). 含有分數乘方x者一 , p是分式指數分母的最低
•132 -
公倍數。
5). X f(x2)型 x2==z

6). X和i/ax+b的函败 u = l/ ax + b ,卽 ax-|-b = u2o


7). x 和 /(x—a)(x—b)的函败—> u 叱x — b
1/x—a
8). x 和 l/(x-a)(b-x)的函蛟—> u 1/b—x
— o
1/x—a
9). x和1/ x^ + bx+c 的函數 u mx+i/ x「+ bx+c

習題18
計算下列反導歧—
—並用微分來證明答案正確無誤:
1). j*x2 dx 8). [3 泌2 de

2.) / x dx 9).
『矗 dx

3).;> 10). f 2 dx

4). ax5 dx
】l).牌
5). x dx
12) . dx
6). x3dx
13) . J/ 2ax dx
7).
14) . J (2x3—5x2—3x+4)dx

•133-
15). ((8/京-_ 1
J

16). 务)3dx 25). sin3 ax cos axdx

J (a2—y2)3/ y dy 26). xdx


17). /a?_x2
18). f (dt 27). J 歹 l+x2 dx

19). 1/a2 + b2 X2 . Xdx f 5 adx


28). i (b—x)6
20).

21).
J >/a2—x2dx
29).
j (3ax2+4bx3)"2ax+4bx2)dx
修 xdx

22). 30). xdx


x2-l
23). 31). 5 bx dx
8a—6bx2
24). cos5 x sin xdx 32). x3 dx
x+1

用代換方法計算下列積分:
33). •..93、) T*(2x+3x2) dx
J (x24-x

34). J x(x2+a2)n 37). Jx2(a+bxs)*dx

35)・ J 2x i/x2—1 dx dx
38). (1一3x)3

36). J x j/a2—X2 dx 39). dx


ax + 订

•134*
f dx
40). 51).
J /W J / 1—x2
Qc + l)dx 52).
C dx
41). /x2+2x-i J 7 1 —a2x2

42)
xdx 53). J i/ 4—x2 dx
(2x+1F
xdx 54).
cosO da
43). / sin 0
j/x—1
dx f cos3° d,
44). J 7 a2—x2
55). J ~sin纽~

45). J X(X -1)5 dx 56) J (l+siW)2 do


f x2 dx J (cos^+2 sine)2 do
46). J (X-l)4 57).

47).
r 4x2+1,
58). (cos^+cos

48). __ dx 59) . j* x 1—x2* dx


/ 4—x2
xdx 60) . J x2p^.2—x2 dx
49).
7 4—x2
50).
x2 dx
/ a2—x2

4. 微分方程(Differential equations)

積分法的最基本問題可如下所述;如果孚是一已知X的函
dx
數,找出y=y(x)。若我們寫
•135-
幾=咫) ,或
,- = f(x), 或 dy = f(x)dx

在討論微分(Differential)時,我們知道dx和dy有單獨的败
學意義,其中f(x)是一已知的函數,y=y(x)是一個未知的函
數,則我們得到一最簡單的微分方程了(卽是說,一個包含導败
或微分的方程式)。對這方程式求解,就是要找出所有滿足這方
程式的函數y=y(x)。如果我們能找出f(x)的一個反導敗,則
這個反導數加上一任意常败便是這方程式最普遍的解了。所以,
這方程式的解不是單一的,而是一依附於一任意常數的函數族。
我們把這種函吸族叫做單參數函數族(。眼parameter family of
functions)0把函數族裹的函數描在座標圖上,我們便得到一單
參數的曲綫族了。在xy面上的一點(xq, y0),只通過曲綫族中
的一條曲綫。在此我們當然是假設反導數的定義域是包含xo在
內。在曲綫的方程式內,把x=x。,y=y。,我們便可算出常政
Co這點我們在下面會舉例加以說明。
另一種很簡單的微分方程式可用下面的式子表示出來:
= f(x),
其中f(x)是一已知的函數,y是未知的函數。這個方程式含有第
二導效,故稱二次的微分方程。在上次討論的例子裏,式子只含
有第一導數,故稱一次方程式。
要解一二次的方程式,我們需要做反微分(或積分)二次,
每一次的積分給予我們一個任意常蛟,所以,一二次方程式的普
遍解含有二個任意的常數,或者說,此方程式的普遍解是一二參
數的函數族。在座標圖中,我們便得到一二參數的曲綫族了。如
•136 -
果我們有兩個已知的條件時,我們便可把曲綫族中的任何一曲綫
找出來,這很明顯地是因爲這裏我們有兩個常數要計算。〔記得
在討論一次方程式時,我們有一個給出的條件,那就是曲綫通過
一已知的點(Xo, yo)〕。舉例來說,我們可找通過某一定點並
在此點有一定的斜率的曲綫,或者我們可從曲綫族中找出一通過
兩給出點的曲綫。當然,當這兩參數的曲綫族被找出後,計算這
些常政(叫作積分常數)只是很小的代數問題吧了!
1纣
[例】解方程式 (dx2
= 6x-x2,並且給出當x=l時,y=l
dy
dx
解:第—次積分給予我們
dy = 3x2一> X3 + C1
dx 3
第二次積分給予我們
y = X3 —-^rx4 + Cl X + C2

這個便是方程式的最普遍的解了,也就是上面所提到的兩參數函
數族了。爲了計算C1,我們把X=l,来-=一2代入幾的普遍式
dy
dx
-2

子, 得
-2 = 3—+ + Ci,卽 Cl — 孕
n J

要計算第二個常數,我們第二個條件,卽當x = 1 時,y=1,
所以, 1 = 1一&_¥ +FC?,
C2, C2
C2==&
t-
12 3

•137-
故從函數族中,我們選取下面的狹義解(Particular solution)

y=x‘一亙 X, 虬
3x+
x4
十 丑
12
[例】解方程式:
d%
= 12x2 — 6x + 2
dx2
已知當x=l時,y = 0;當x = 2時,y=10.
解:連繽積分二次,便得最一般的解,
y = x*—x3-t-x2-f-Cx x + C2
用上面已知的條件,得
C[ + C2 = — 1
2Ci + C2= -2
解這二個簡單的代數式,、則得C】=-l, C2 = 0o所以,這方程
式的狹義解是;
y = x4—x34-x2—x
一更普遍的一次方程式可用下列的式子來表示;
8(x)
■w
其中f(x)與g(x)都是已知的函數,y是所求的函敏。這方程式
最主要的特點就是式子裏面的變數可被分出來:
f(y)dy = g(x) dx
卽x只出現於右端,y只出現於左端。故在解方程式時,我們只
需計算兩個反導败: Jf(y)dy 及 Jg(x)dx。
【例】討論下面方程式的解:
•138 -
dy dy = (I—x)dx
= (l-x)y =>
dx y
dy
b) * xe eydy=xdx
dx
dv =2v dv 2dt
dT v
dv vdv ==sds
d) ds v
【例】 一曲綫在(x, y)的斜率爲2xy2。若曲綫通過點
(2,-1),求曲綫的方程式。
dy = 2xy:
解因給出 dx
用上面的法子把變數分開,則得:
dy 2xdx
y2
兩邊積分,則得:
』孚=“xdx
一丄 = X2 + C
y
這便是方程式的最普遍解,也代表一參歟的函數族。要找出通過
(2,-1)的曲綫,我們使x=2, y=-l,得出
1=4 + C, C= —3O
故所求的曲綫是
1
-y=x2-3 或 y 3 —x2°

•139-
習 題19

解下列微分方程式:
10). dy sec2 x
1). dx
2). =ay 11). sec2 y

3). 12). x

4).
5>
<
==x2—1 13).
y4^=i/y2-1
• =y2 — 1 14). x dy=2y dx

6) dy = (x-2)(xi-l)
• 15) =4y
dx

7
• dy =(y—2)(y+l) 16). (x—a)dx+(y—b)dy=0
8)
dx
• dy =sin nx 17). xdx+ydy=O
9D
dr
>
• =sin ny 18). xdy+ydx=O

19). xdy=(y—2) dx
利用下列已知條件,解下列各微分方程式:
20) . y,+y = 0(X = O 時,y = 2)
21) . y,+ y = 3 (x = 0 時,y = 0)
22) . y,=sin x (x=0 時,y=0)

23). y,一y2 = () (x = 0 時,y =

•140-
24) ・ 某物體以每秒20呎下降,其連動方程爲:
祭+gt+c
s代表距離,t代表時間,g代表重力加速度,c爲常政(g=32呎/每
秒/每秒)。6秒鐘後其速度是多少?
25) . 一小石從停留在空中的汽球擲下,擊着地面時的速度爲384呎/秒。
問小石下降所花時間若干?

5.微分方程的應用
在很多門自然科學裏,一變量y常常依照下面的規律而改
變;
事=ky,
其中k是一常數,t是籍間變數,並且,k可以是正或負的。卽
是說,y對時間的改變率是和y,本身成正比。這個情况出現於化
學、力學和其他的工程科目。
首先解此方程式(用分變數法)得
dy kdt
y
In y = kt + C
這個一參數的函數族是方程式的最普遍解。在這裏,我們已假定了
y>0o如果k是已知的話,則若給出在某一時間y的値,我們便
可計算積分常數c 了。若k是不知的話,上面的解便變成一二
參政的函數族。怛注意,k並不是積分常數,只和問題有關。這
•141-
時,我們需要知道在兩個給定的時刻y的値,才可把方程式全部
解出來。
【例】 求k和c,若已知當t=0時,y=10 0及當t=10時,
y=25。
解:把這些已知的條件放進式子內,便得下面二代數式;
In 100 = 0 + c
10k=ln 25-ln 100

解之則得 c=ln 100, k=—*ln4

1
結果,我們得到 lny=-^ln 4t + ln 100,或
In y = ln(lOOXe"-io In 4t)

卽 y=100xe-4

【例】(化學)一放射性的物體的質暈的迅時改變率(Instan­
taneous rate)是和物體在某一刻的質量成正比例,其比例常數是

負的。如果在第一天內,土原來的質量消失掉,問共需多少時間
物體的質量:才等於它原來的質量的!?
解:設m。是原來的質最(卽在t=0時的質量),若m是
在時間等於t時的質最,並用日爲時間的單位。則
dm =km
"dT
解之得 In m = kt + c
再用給出的條件:
•142 -
3
m = m( 當 t=0; m=气■m。,當 t=lo
3
所以得出, In m0=c, k=ln—
m
原來的式子便變成 In = kt。

m
我們要知道當 時,t的値是什麽?從上面的式子,我們得
,1 1 1 Jn 2
t=wy= In
(1)
卽是t=2.409天。
下面我們舉出微分方程在力學上的應用。設有一質點在直綫
上運動。若我們取它運動的軌道爲X-軸的話,則質點的速度V
_ d 2x
爲, 它的加速度是 a
»它的加速度是 =商 ,又可被寫成如下,
dv dv dx _ dv
~dt di dF =
dx , ~dt dx
【例1 一球在一光滑面上滾動,它的初速爲25尺/秒。如
果空氣的阻力使球受到8尺/秒2的減速,問球在受了起初的冲撃
後能滾多遠?
解:現已知球的減速,卽
d 2x
-8

此乃一二次微分方式。解之則得
x = —4修 +t + C?
—,n dx
已知當 t = 0 時,x = O,-j丁 =25, 所以,得出下面的代數式;
•143-
25 = 0 + Ci, 0 = 0 + 04-C2
卽是說,球的運動方程式是
x==4t2 + 25t
現在我們問,當球停止滾動時,x的値是甚麽?爲了囘答這問題,
我們首先找出當v = 0時t的値,然後再求x的値。
把上面的式子微分一次,得
dx
v 一 8t+25
dF
所以,當 t 滿足 0=-8t + 25 時,v=0;卽 t=25/8。
再把這個t的値代入x的式子,便得出
xfE(斜 =39編尺了。
在物理學裏,我們知道,若一質點在X-軸上的區間一b<x
<b內用和x成正比的加速來運動,我們便稱這運動爲簡諧振動。
運動的方程式可被寫成
x=b cos (aJt+c)
其中常數3决定運動的周期T(T+2时3):常數C質點在t=0時
的位置和速度。用微分導數來表示則得
dx
—。b sin (eot + C)
"dF
d 2x
=—o>2 b cos(o)t + C)
"dt^
d2x — a)2 x
所以, E
任何以x = 0爲起點、2时s爲周期的簡諧振動都滿足這個方程式。
反過來說,我們可以證明滿足微分方程的運動都是周期爲2兀但的
•144-
簡諧運動。
d2x dv
首先,用 ~dt2- V
dx
把原有的方程式改寫成
dv
—co2 x,或 vdv= —a2 xdx
v~dT
v2
解此式則得 — 0)2
2
FC

或可改寫成 (A)
2c
我捫看出,因V2不能負的,所以,X々V 歹 。 爲了方便起見,我
們用b來代表這個黄;
b= 2c

故得 (祭)=必(屮-乂2)
卽若X永遠處於區間一 b<x<b內的話,這個方程式就被滿
足了。如果xVb2時,我們可把上式抽平方根並且把變數分出
來,
dx
± =/ dt
l/b2-x2
積分全式則得
dx
±
lZb2~x2
x
或 cosf ±( Ot + c)〕

•145-
旣然cos X是偶函數,所以,我們可把上面的式子寫成
x=b cos (効+ b)

這便證畢。

習題20

1). 解方程式:
y〃+6y'+4y=0 (y'=^-, y〃
yo=16 而 y'=24
2). 解下列微分方程式:
')湛
a) +3祟s亠…
整+3-#+2y+5=°
dy 10
yo="2O, ~dt
b)影
d2y
+3 孚 dy +5=0
dt2 dt
yo=2O, 4t_=10°
3). 某國人口 50年內増至兩倍,假設其人口增長率與居民數目成比例,
那末在若干年內,該國人口增至三倍?
4). 某辦公室體頂爲15G'X5O'X2O',據檢驗查出室內空氣含二氧化碳
0.2%,後來開動通風風扇,將含0.05%二氧化碳的新鮮空氣引進。
新鮮空氣引進的速率爲每分鍾9,000立方呎。求20分鐘後室內空氣
含若干二氣化碳。
5). 錯分解的速率與其存在質嚴成正比。若原有質虽之半在1600年內消

•146-
失,那末,100年內損失的百分率爲何?
6) .某公司將不擬投資的剩余利潤存放於某銀行。若以年息五厘祺利計
算。若干年後,該欵會增至兩倍?
dx
(提示:利用 ■dF =0.05x,其中x爲t年後的存欵額。)
7) .運動員和獨木舟共重320磅。若划槳沿運動方向的力爲16磅水對運
動的阻力(磅)相當於速度(呎/秒)的兩倍。從靜止開始後15秒,獨木
舟的速率爲何?

•147 -
第6章積分的技巧
在上一章裏,我們指出用代換法往往可用一適當的代換
X 0(u)把瑙分f f(x)dx化成一極其簡單的積分。但是,當一
個問題給出後,並沒有一定的法則引導我們去找所需的代換;問
題的解决就决定於讀者的智力和對代換法的熟練情度了。有一點
我們應注意到,那就是在進行代换x=3(u)時,我們已假設了式
子乂=(P(U)有單一的倒逆,這就等於說,我們已假設了 3'(x)在所
考慮的區間內處處不等於零,其中(P(X)是x=(P (u)的逆函數。如
果這個假設不被滿足的話,往往就會得出錯誤的結果。這是一個
非常重要的地方,在一般的敎科書中都沒有很明顯的被指出,往
往就算是很熟練的科學工作者也會把這點忽視。如果在區間內,
0'(x)=o只出現於某些孤立點,我們必須把區間再行分割,伊
得這些孤立點都變成區間的端點,在每一新的區間內,我們再蠢
行代換。在下面,我們將逐類函數的積分加以詳細的討論。

•148 -
1. 一般較困難一點的例

1).
dx
a2 sin2 x + b2 cos2x =制 b2
]
tan2x+1
dx
cos 2x

因爲 sec2x dx = d(tanx),所以,我們用代換 tan x=y,


a
則得 _a2 dy;再進行綫性代換, Z=by>
-b2»-y2+i
a h
便得 & = Bdy,或dy=—dz,而積分也可被寫成
dz
z2+l
C dz
再用公式 J z2+l snf(z),我們便可把所求的結果
寫下,卽 I dx
sin2x + b2 cos2 x
tan"1
(5)
1
=疝 5 (4 tan x )了。

2).同樣地,我們能得出:
-^arctanh(Atanx)
f ___
J a2 sin2 x—b2 cos2 x 或
—茄arc coth (衆 tanx)
dx
3).計算 1= sin x
•149・
解:首先觀察到sin x=2 sin与cos
~2
=2 tani cos2 兰
2 並
用代換 u = tan x 則du=》sec2成dx;所需之積分
29 2
可被寫成。
dx du
u
2 tan—

=ln I tan^-|,

其中的絕對値符號已在上面類似的例討論過了。若我們將x
換成x+专時,則上述的公式變成:•
dx
cos x
In I tan(奇+?)[
4).計算Ii cos2 x dx, I2= sin2 xdx
解:用三角公式,2,cos2x=l+cos2x,使I】變成
1! = — f (1 4-cos 2x) d x =-y(x + sin xcos x)

同樣地,我們可用公式2sin2x=l — cos2x來計算 项 得
12=~ [ (1 —cos 2x)dx=!(x—sin xcos x)
Z J £

5).計算 I】=;/(a2-x2) dx; 12= i/ (x2-a2) dx


解:用代換x = a cosu; (a尹0),使虹變成
Ii=a2 讨 1 — cos2 u( —sin u)du

•150・
—a2 sin2 u du
再用例4的L,得
Ii=— sin u cos u);因 U = COS~xi

所以,L= 一T音)+奇仍E有
2 cos
現用代換x = a cosh u來計算【2; dx=a sin hu du
I2 = a2 i/cosh2 u—1 sinh u du = a2 sin h2u du,現用公式

—2 sinh2 u= 1 — cosh 2u,得


a2
【2 = J (1—
—cos
cos h 2u)du ——- (u—sin hu cos hu)
h 2u)du=
~2
cosh-1】(告)+奇1/(x2一旳
夺cosh-

同樣地,用代換x = a sinh u,我們可計算類似的積分。


&= j V (a2+x2) dx = 詩sin或(言)+专讨3+a2)
a
6) .用代換,u=W,dx =
u2
du,可引至下列的積分公式;
dx
x l/3c2^a2~ KG)
dx —4~sin h-1
】2 =
x i/x2 + a2 ~
dx
L= x i/a2 —x2-— a
—os h-1

7) .計算下列類型的積分
•151-
sin m x sin n x dx; I2 = sin m x cos n x dx;

cos m xcos n xdx,其中m與n是正整數。

解:用三角公式:
sin m x sin nx 土{ cos(m — n)x - cos(m + n)x

1
sin m xcos nx =-^-{sin(m + n)x + sin(m—n)x}
2
cos m x cos nx =土{ cos(m + n)x+cos(m
cos(m + n)x+cos(m— n)x }
—n)x

再用代換,u = (m+n)x或u = (m-n)x;則可得到下面的結果:


'1『sin(m n)x sin(m + ii)xl.
m尹n
m —n m+n
Ii = sin m x sin nxdx
/ sin 2mx\
2 (x - 2m 2m ); m = n
丄! cos(m+n)x 十 cos(m—n)x]
2 L m+n m —n )
乂n
I2= sin mx cos n xdx =

T cos 2mx
~~2m-
m=n

,丄!sin(m + n)x [ sin(m-n)x[. m ,


• 2 t m+n m—n )9 干
& = cos mx cos nx dx=
x 4- sin2 mx
2 ~2m~

•152・
習題21
計算下列各暗分,並微分所得的結果:
(6x—5)dx
1). 13).
1v/3x2-5x+6
dx 14).
2). j/x2—a2dx
1 + /x+1
dx (x + l)dx
3). 4/3+1 15).
xZ~x-2
/3Fdx J / 4 — 1 dx
4).
/£-3/五 16) ・
dx f x+1
5). 17) J xQ+xtfQdx
X/x2_[
In2 x dx f cos /W,dx
6). 18).
J "VT
cos (In x) (1
7). x 19).
J e^+l dx
8). (14-sin x)5 cos x dx C sin2 x cos x
20). J 14-sin2 x
9).
dx_______ f x5 dx
a2 sin~2 x+b2 cos2 x 21).
J / a3—x3
10). r dx f v/a2—x2
J W+a2)2 22).
J-
f x2 dx C dx
11). J jZ a2—x2 23). J /(x2-汲)3
12). f /x2—a2 dx 5x—2___
24).
3x2+8x+10

•153・
12 dx dx
25). 27).
x4+x3—x —1
x2+l dx
26). -dx 28).
X4 + X2-F1 /一 2x2+x—3

2.部份積分法(Integration by Parts)

這個計算積分的技巧是基於下面的微分公式;
(fg), = fzg + fg,
用積分把公式寫出,我們則得
f(x) g(x)= J g(x)f,(x)dx+ ( f(x)g,(x)dx

或 J f(x)*(x)dx = f(x)g(x) — jg(x)f《x)dx

這就是部份積分法或換部積分法的公式了。一個積分的計算被換
成另一積分的計算。如果我們把一積分裏的被積函數 (w(x) dx
寫成二函數的乘積:w(x)=f(x) 0 (x),並且,如果下面的積分
是很容易計算的話;
g(x) = J 9 (x)dx,
則用上面的公式,我們便把需要計算的積分
J Q(x)dx= J f(x) (p (x)dx= J f(x)*(x)變成 J g(x)依x)dx,

而這個積分通常都是比原來的積分容易計算的。因爲一個給出的
•154 -
被積函數/(x)可以被分成很多個不同的乘積f(x)o(x)= f(x)
g'(x),故此,這公式給予我們一個很有用的工具。
[例】 下面的例子可幫助我們了解部份積分法。
如果我們使 f(x)=x,g,(x)=ex,則得f'(x) = i, g(x)=e\ 故
xexdx = ex(x— 1)

同樣地,我們可推導下面二個公式;
x sin xdx= —x cos x+sin x

x cos x dx=x sin x+cos x


又,若f(x) = lnx, g,(x)=x,,刖我們用部份積分法得
Xa+1 1
xa In xdx= -—
—(Inx-
(Inx —r-7)
a 4-1、 a+“
在這裏,我們假設a乂一1。若a=-l,則上式左端的積分變成
J 4 In xdx= (In x)2— J In x dx
x
其中右端的式子,是用部份積分法的公式來得到的。爲了要計
算式子右端的第二項,我們用代換,
dx
In x = u; =du
x
dx u du = -^-u2 =-y(ln x)2
刖, In x
x 2
二式並合起來,便得
1 (ln x)2
1 (In x)2=;
J -^-In xdx=(ln x)2一亏
2 2

•155 -
在計算積分1= sin-i xdx時,很明顯,我們應避免用困難
的函數,故使f(x)sin-1 xg,(x) = lo所以,得出
xdx
1 = J sin-1 xdx=x sin-1 x— 1/一1-X2

要計算右端第二項,用代換x2=u,得出
I=x sin* x+ •/ 1 —X2
同樣地,我們可計算積分
[tan-1 (x)dx=x tan"1 x—-yin (14-x2)

和其他同類型的很多積分了。
下面的積分類型稍爲不同。在這裏,我們連續的把部份積分
法的公式應用二次而把積分變囘原來的樣子。結果我們得出一代
數式,解之則得所要計算之積分o例如下:
eax sin b x dx -^-eax cosbx+-g-J
—-g-e cos bx+ eox cos b x dx

—-g-eax cos bx4-嘉e a2


sin

eax sin bx dx

這是兩次應用部分積粉法的公式的結果。觀察到,式子右端
末項所含之積分和所需的積分一樣,所以,這是一代數式,其中
積分Je" sinbxdx是未知數。把這代數式解出來,便得所需的
結果:
J eax sin b x dx 1
a2 + b2 eax(a sin bx—b cos bx)

•156-
同樣地,我們可推導下面的公式,
eax cos b xdx= eax(a cosbx+b sin bx)
a24-b2

3.遞推公式(Recurrence Formulas)

很多時,並積函數不只是變數的函效,它常常還依附於一正
整的指數&當我們用部份積分公式時,我們便得到一相同的.式
于只是n在新的式子裏取一較小的値。所以r若干步驟之後,所
要計算的積分便變成所熟知的結果了。這樣計算積分的方法我們
稱爲避推法 Qecurrence process)。
[例】計算類型爲:
【1= cosn xdx; I2= I sinn xdx; I3= I sinm xcosn xdx

的積分。其中m和n都是正整數。
解:首先用部份積分法計算一次,得
cosn xdx=cosn-1 xsinx+(n—1) cos11"2 xsin2 xdx

式子的右端可被寫成
cosn-1 xsin x-F (n — 1) cos dx— (n— 1) Jcosn x dx

所以,我們得到所需的遞推公式;

cosn xdx= n cos n-l xsin x+ n—1 cos11~2 dx
~n~

•157-
如果我們明顯的指示出積分是口的函數,則可把上式寫成:
Ii (n) cos11-1 xsin XT a—1 Ii(n-2)o
n
將此公式再應用一次時,我們便得到
Ii(n-2)= 一A_ cos。“ xsin x-F n—2 L(u—3)
n -1 n-1
卽是說,只需把口改成n-lo如此類推,公式一直把我們引至
11(1)或11(0),可是這些都是很熟習的積分;
1丄(1);= cos xdx = sin x; Ii(O)= dx = xo

同樣地,應用部份積分法的公式一次,我們得到下面的二個遞推
公式:
sinn xdx n—^in
1 n-1 x cos
sinn-2 x 4-
xdx
n

sinm x cosn x dx sipm+l xcosn-lx 斗 n—1
m+n m+n
sinm xcosn~2 xdx

這些公式也可被寫成:
—1
— sin11"1 x cos x 4 n了
L(n) = L (n-2)
Isgn)
sinm+1 xcosn~1x n—1
m+n m+n & (min-2)
注意下面的例孑:
1). (lnx)mdx=x(lnx)m m J (In x)m・i dx

•158-
2). xm ex dx = xm ex —m xm,

3). xmsin xdx= —xm cos x+m xm-1 cos xdx

4). xm cos xdx=xm §in x—m xm-1 sin xdx

5). xM(ln x)m dx = x・+i(ln x)m m


aTl a+1
— dx; a^-1

習 題22

用部份積分法,求下列各積分:
1). x sin x dx 7). e* cos xdx

2). x2 sin x dx 8). In xdx

3). x sin x cos xdx 9). In xdx

4). x2 cos2 xdx 10). ln2x dx

5). 11). J/3F in,xdx

6). ax dx 12). x tan2 x dx

•159-
13). In In x 18). dx

14). dx 19). dx

15). cos In xdx 20).

b (l+sin x) 21).
16). (x2+3x+5) cos 2xdx
1+cos x~ dx
dx
17) (x2+a2)n 22).計算下列各積分米:
X 8S xdx g)・ J cos6 xdx
~sin2 x~

b). (1 一 X4)2 h). J x<Z 1—x2 dx


* cos xdx i). x2e xdx

d) .J x» e-Z dx j).J野dx; n、l


e) . J x8 cos x2 dx Jxm Inxdx
k).

J sin4 xdx J x2 (In x)2 dx


1).
第22題可參閱下面的提示進行計算:
用部份積分法;使&'(x)= 黑号 olU X
因 g(x) = — sin
用代換U = X4
用部份積分法;使g'(x)=cosx
用代換U = X2
先用代換X2 = U;再用部份瑣分法。

,160*
⑴ 用過推公式,或連續二次應用三角公式2sWx=l — cos 2x
®
用遞推公式
ch 用代換x=sin0,再用遮推公式
ci) 用部份積分法,使g'1x)=3
(j)® 用遞推公式
® 用遞推公式
用部份積分法,使x2=g,(x)

4.有理函數的積分
(Integration of Rational Functions)

在初等函數裏最重要的函數類便是有理函數了。這種函數取
下面的型式;
f(x)
R(x) = g(x/
其中 f(x)=amxn,4-am_1xm-1+...+a0,
g(x) = bn xn+bn.x xn-i+ ••• +b0; b0y=0o
我們記起,多項式的積分是非常容易的,而且,積分後仍得
一多項式。所以,我們只需研究分母不是常數的有理函數;因爲
分母爲常數的有理函數只不過是一個普通的多項式吧了。並且,
我們可假說分子多項式的次數比分母多項式的次數低 ,否則,我
們可用g(x)去除f(x)—直至到它們的次數有上述的關係。當然,
函數f(x)/g(x)可被寫成若干多型如an xn/g(x)的函數之和,所

•161・
以,我們只需考慮到類型如xVg(X)的函數就夠了。
a).基本類型
我們不會馬上便考慮最普遍的有理函數,反之,我們首先考
慮g(x)是特別簡單的例子:如
g(X)=X; g(x) = l+x2;
或者普遍一點,
g(X)=Xn; g(X)= (l+x2)n,
其中n是正整數。這個情形是重要的,因爲如果分母g(x)是一綫
性函數和一定正或負的二次式的幕時;
g(x) = (ax+月)n; a尹0 g(x) = (ax2+2bx+c)a
我們可用下列的代換把分母函數變成g(x)=x\
或 g(X)= (l+x2)n 的型式,
1).如果g(x)=ax+夕;a尹。時,則代 f=ax+n; df
dx

a, x 3 —月) 因爲這個代換使一個綫性函數變成
ao

另一綫性函數,所以,多項式 f(x)=f (直亏£)變成另


一多項式0(f),並且,新的多項式和舊的多項式的次
數相同。所以,我們得,
f f(x)
fW ,dx = ± 0(f)a f
J (ax+ py a -F5
這便是我們要討論分母爲xn型的原因了。
2).若分母g(x) = ax2 + 2bx+c是一正定或負定的二項式時,
則不論變數取任何數値,g(x)永遠不等取零。我們可把
•162 -
分母寫成:
丄 d2
a (ax+b/f a -;(d2=ac—b2)
ax2+2bx-Fc=—

在初等代數學中,我們知道正定或負定的二次式的判斷正
的,而且,a尹0, a的符號便决定這二次式的正或負了。我們可
用下面的代換,
ax + b

使得被積函數的分母變成 〔告(心矿
所以,爲了要積分一些分母爲一綫性函數或一正定或負定的
二次式的幕的有理函數,我們只需要學會積分下面的函數;
1 X2x X2X+1
(1+X2)n; (1+乂2),

下面我們看到,就算類型如上述的函數也不需普遍地去考
慮,因爲如果我們懂得怎樣去積分v = 0的情形就足夠了,這是
因爲我們可用遞推公式或部份積分法來計算這些比較複雜一點的
積分。所以,我們現在只考慮類型如;
丄. I. X
xn ' (x?+l)n,(l+x2)n
的函數。
b).基本類型的積分
函數 * 的積分馬上給予我們下列的結果:
J马!-火=卜I X I ;如果n = l
•163・
一 (n-1) x11-1 ;如果n>l
這些都是初等函數。第三類的函數可用代換§=1+乂2來計
算,得出2xdx=df

xdx
4hi(l+x2);若 n = l
(l+x2 戸 2 1
一 2(n—1)3+1) n-il 若 n>lo
然後,我們來計算積分
dx
(1+X2)n
如果n=l,這個是習知的結果,卽I1=tan-\x)o故可假設
n>l,使用遞推的方法去計算。
因爲 1 1
8+1尸—(2x2)5 (1+乂2)。

dx dx x2 dx
所以, j (1+乂2)。 (1+X2) n-1 3+丁
右端的第二項可被化簡如下:
x
用部份積分法,使f(x)=x, g,(x) = (1+乂2)。
並用習知的代換求用
g(x) = TI" (n-l)(x2+l) n-1
故得一遞推公式;
dx =_______ x_______ , 2^-3 dx
_______
In=J (l+x2)n = 2(H-l)(x2+l)n-1 2(n-l) J 3+1) »-1
=___________ X________
n—10
~2(n-l)(x2+l) n-i 2(n-l)
•164-
連續的使用此公式,直到我們被引至
I1 = tan-1(x)
這個積分還可用別的方法來計算 。例如,我們可代x=tant,
得到
[ dx
=J COS tdt
J (1+X2)n
從此出發,我們便可用習知的遞推公式了。

5.部份分式(分項分式)(Partial Fractions)

從初等代數學中,我們知道每一有理函數都可被寫成若干函
數之和,這叫做把一給出的有理函數分解成分項分數 。這裏,我
們把一有理函數寫成有限個有理函數之和,其中每項的分子是一
常數,分母是一綫性函數的幕,或者分子是綫性函數的幕而分母
是一正定二次式的幕。利用此法,我們可以積分最普遍的有理函
數了。因爲從上面所得的解釋中,這些分出來的有理函數的分母
都可被化成型式如X。或(1+乂2)七 所以,這些分數都是上述的
基本型式的組合。
對下面所述出的結果我們都不加以證明。因爲在初等代數學
中,我們都遇見過這種證明了,並且,在實例中,要分項的有理
數都是比較簡單的。
從初級代敷裏,我們知道每一多項式g(x)可被寫成
g(x) =a(x- a D& (x- a 2)坛・・(乂2十2 戶 1 x+ T i) pi
• 165*
(X々+2 P 2X+ T 2)P2…
這裏,«1«2•-是方程式g(X)=O的實數和不重合的解,正整蛟
与/2…表示這些根出現的次數;因子 3+2彳1X+T1)是表示
正定的和不重合的二次式。從這裏我們得到方程式的複根。整數
与功是這些複根出現的次數。所以,只要計算被積函數的分母的
實和虛根就可把它寫成上述的因子分解了。現在我們並假設分子
多項式的次數是小於分母多項式的次數,則我們可把分解一有理
函數的成分項分數的定理講述如下;對於每個因子(x—a)。一
其中a是多項这g(x)的一個實數根,並且,Z是這個根出現的次
數,我們可决定下面的一個式子:
A】 A2 A1
(X— a )
4- (x—a
(x_ a)2
.
-••• + (x— a 广
對於每個因子Q'(x)=x2+2 0x+7,我們决定下面的一個式子
B1 + C1 x ,I B2+C2
B2 + C2 x Brr +C
X +, ... +- B +Crf X
X,
Q(X)—十 Q2(x) 十…十 Q「(x)
用來决定這些常數的條件就是:上面所述的二種式子之和是
等於原來的函數:f(x)/g(x)。就是說,被積函數f(x)/g(x)可被
寫成若千個有理函數的和,而這些有理函數都屬於我們已經積分
過的類型。
【例】若g(x)=x2 — 1,則
x2 — 1 _ 2 2 x+1

故 J X2-1 2 ■In x+1


更普遍一點,如果g(x) = (x-a)(X-尸),卽是說,如果g(x)

•166-
不是一正定的函數,則有,
(x-a)(x-戶) a — p x— a a — p
r dx x— a
所以, J (x-a)(x — p) = I
a — p In x- P 當然,只
有當x不在區間(a】/?)內,這個結果才是有意義的。
現設g(x)取下面的形式;
g(x) = (x—ai)(x—a2)・・・(x—aQ,並且,這些根都是非
重合的。甬上面的定理,我們可寫:
] = + -2 ------ p 端
g(x) x— ax x— a 2 x— a n
在方程式的兩邊我們用x—ai去乘,把分子裏和分母裏的公因子
(x-a D約掉,並且處處代x=alo這樣,我們便得
ai =
___________ 1___________
«3) — («1- «n)

{冽】一個典型的例子就是當g(X)有重根的時候,如g(x) =
I 從上述的定理,我們得到,
X2(X—1) °
1 一 a__ b
x2(x—1) x-1
把這個方程式左端和右端相乘x4(x_l),則得一方程式。
l = (a+b) X2— (b—c) x—c
對於所有變數的實數値,這方程式都是對的,從此,我們可用未
定係數法來决定a. b、c的値了。所得出的代數式是
a4-D = 0, b—c = 0; c-f-l=0,

•167-
卽 a=l, b= —I, c 一 1。

所需的分解便是
X2(X—1)—X—1
從而得到所要計算的積分,
C dx- In ] x-l | -In J x I +土
J x2(x—l厂'
現在我們再分解函數岸口y (我們不討論複數,所以,不
IB*皿伍毎■厶切"輙一1

需考慮到這函數有複根)。從定理得
1 aa[ bx+c
bx+c
x3+l) — x x2 +1
用未定係數法得a+b=c=a—l=O,故
1
X(X2+1) x2+l
所以 dx
x(x24-1)
In J x I —yin 3+1)

x4 + l 。
作爲第三個例子,讓我們來考慮函數淆丁 ° (微積分始創
人之一萊布尼茲(Leibnilz)也覺得這是一個困難的積分)。先把
分母分解成二次的因子;
X4 + 1 = (乂2+ 1 + 讨云)(X2+ 1 一 1/TX)
上述的定理給予我們下列的分解形式;
1 _ cx+d
ax+ ___ ___________
x4 + l~ — x2 + i/~yx +1 〒 x2 — ■/乏x+1
我們再用下面的條件來决定未知的係數,
•168-
(a+c)x3+ (b + d—a-/-!" H-ci/^Jx2 + (a+c—+di/'^_)x
+ (b+d-l)=O
從而得出
—1__ . b= 1 . 1
a b =—
~7T; d=S
2
21/T* 2' 2t/T'
於是我們便得函數的全部分解,
1 _ 1_. x+-/2~ 1 . x--/T
x4-fi~ ~''~2t/T • x2+72x+i 2/T X2—yTx+l
用習知的公式,(對數形和反正切型)便可得到下列的結果,
(dx _ 1 In x2 4- i/2^x + l In
J x4+l 41/T 41/T
x2— i/y X4-1 4- 1_ r
ITT Lan-1 (i/Tx+l) 4
ITT
tan-i(讨了x—1)
從此看到,計算這些積分,只需要做一些冗長的代數計算,
而並沒有槪念上的困難。讀者學習稍分法時,必頊牢記這點。
【例】下面是幾個分項分數的例子。讀者應嘗試把這些結果推
導出來;
7X-4 _ 2 . 1________ 2
(X——2(x+2) — %—1 (x—I)2 — (x-*2)
X2+6X—1 _1____ . 13
b)
(x-1)(x-3)2 - 2一一 1 2 . x—3 (x—3)2
5 1 x+1
c)
徉 1)3+4) 一 x-1~x2+4
—x3 + 6x2 + x + 2 =____ 1_ 2_______ x—2
d) 3—1)(x2+1)2 x— 1 1 •gi 3+1)2

・169・
習題23

計算下面的積分,並微分所得的結果:
x5+x4—8 dx
1). X3—4x
dx 8).J X4+1
x—5 / x@+l
2). dx 9).
x3—3x2+4 x4+x2+l
_____ dx_____
3).
4x4-3 dx
(x—宓 10). J (X+1)2(X—2)S
x3 —2x2 + 4 3x4+x,+4x2+l
4). x3(x —2)2 11). ~x5+2x3+x dx
3 dx x2+3x—2
一(x-l)(x2+x+l)2 dx
5). 12)・
x2+7x+16
5x-2 5x2+6x+9
6). 3x2+8x+i0 13). (x-3)2(x+1)2 dx

7). J 3x+5
x2+4x+5
dx

6. 一些包含三角函數和雙曲函數的代換
有些函數的積分可被化成對有理函數的積分,爲此,我們只
x
需用一些很簡單的E角學公式。若使t=tan 万 , 則初等三角學給
予我們
sin x 2t .
cos x = 1-馋
1+。' 1+馋

•170*
1
因爲 1+t2
cos2 和 l+t2
sin2
万'
並且,用下列的公式,
X
sm x = 2 cos2— tan
X 及 COS X = COS2 壬― sin1 x
2 2 万
從而得出所需的公式。所以,我們斷論,我們可以用t=tan奇這
個量來有理的表示sin x和cosx。再微分t = tan手這個定義,便

得,
dt _ I l+t2 dx
^dT = 2 cos2
x 2 ■dt 1+伊
~2
dx
所以, 這個導數也是t的有理函數。
從雙曲函
忱5E cosh
Jfe - … x==
一2 —(ex和 —
甲一丿仲 sinh x= T SY")
的定義,我們很明顯的看到對應於t的量應該是砧所以,我們
得到下列的有理表示式;
cpsh x •y G+W)sinhx □•yC—f); r=e'

dx
這裏, dF 4■也是一個r的有理數。爲了使這個代換和三角函
數的代換t = tan(专)更加相像,我們在這裏定義t = tanh・

使得 sinh x 2t ..;coshK-
ghx=(S (E)
(17),C (if。

並且,這個定義的微分給出下面的有理式;
•171・
dx 2
"dT (1^)
在下面,我們討論這二個代換對各種有理函數的應用。這些
計算並沒有任何槪念上的困難,讀者只需耐性地把所有的代數計
算了解,便可掌握一個很廣泛的計算技巧了。
a)被積函數是取R (cos x, sin x),其中R是一有理函數
我們考慮被積函數形如R (cos x, sin x)的積分,其中R
(cos x, sin x)是sin x和cos x的有理函數,卽R是由sin x和cosx
和常數來組成的有理函數,例如
3 sir^ x+cos x
R=
3 cos2 x+sin x
如果我們利用代換t = tan吝,則積分
J R(cos x, sin x) dx

可被寫成
l-t2 2t \ 2dt
l+t2,l+t2 ) 14-t2
新的被積函數是t的有理函數。應用上述的方法,便可把積
分計算出來。
b)形如R (cosh x, sinh x)的被積函數
同樣地,如果R是一由sinh x和cosh x和常數組成的有理
dx _ 2
函數,我們則可用代換t=tanh吝。並且,記得 dt 一 (If '

從而可把所需的積分改寫成

•172-
JR (cosh x, sinh x) dx= j R( ‘ 2t \ 2 dt

新的被積函數也是t的有理函數了。
c)形如R (x, V 1 —X2 )的被積函數
如果被積函數是一由XX和和常數組成的有理函數
則可用下列的代換
x=cos u, 仍([一乂2) =sin u; dx= —sin udu
用這個代換,我們可得到
JR(x,i/ 1—x2 )dx= — Jr (cos u, sin u) sin udu
=J R (cos u, sin u) du
這個比積函數在a)節裏曾經被考慮過,所以,我們可再用代
換t = tanA而使被積函數變成t的有理函數。
一 2 %—。

c1)形如r(x, 的被積函數
我們可用代換X = cosh U把被積函數寫成如在節b)裏
所討論過的一樣O
d)形如R(x, i/ET)的被積函數
如果被積函數是一由X, t/ET和常數而組成的有理函數
則可用下列的代換。
x = sinh u
如此,便把被積函數變成在B節裏所討論過的類形了。
e)形如R(x, i/ax24-2bx + c)的被積函數

•173-
首先我們把二次式改寫:
ax2’ + 2bx+c=«- 1 (ax+b)2.
/ ,. ac—b2
a z a
如果判別式ac-b2>0時,引進一變數定義成爲
五 ax + b
l/ac—b2
用這代換,便把平方這二次式的平方根寫成。
』ac —b2 (宀1)
於是便把被積函數變成在d)節裏所討論過的類型了。在這
裏,常數a—定是正的,否則平方根給我們引進複數。
如果判別式爲零,卽ac—b2 = o, >0,則得
V (ax2 + 2bx+c) == l/a7(x+?)
a
這時,被積函數只是X的有理函數。
如果ac-b2<0,我們用代換£ = ax + b
1/ b2—ac 從而把方
根裏的二次式寫成 b2—ac (f2_i)。如果a是正的話,則被
a
積函數屬於節c)的類型;原來a是負的話,我們可把二次式的
平方根寫成/牛三3丄讨?=■戸,卽是說,被積函數屬於節c)的
類型。
f)形如R(x, i/ax + b , i/ax+ 的被積函數
在這裏,我們用代換f =/a x+们使得

•174 -
x=(/—尸). dx 2±
2 , a
所需的積分便變改寫成,
& R(x, i/ ax + b > Vax+ g ) dx
2_
="( 气]/*{a 宀(ajba)}‘f) df
~a a
這便是在e)節裏所討論過的積分了。
g)形如 R(x, “ ax + b )的被積函數
ax+ p
在這裏,我們引起下面的代換,
n // ax + b \
V \ ax+ P )
ax + b 一 ”f n + b dx
便得f n a x+ R'
X
~ a n-a
a” 一ba
(a £ n_>)2
n e "-I

從而得到
jR(x,再財
J R (二心"b a B —ba
a E n—a (a & n —a)2
這是一有理函數的積分。
【例】計算積分 1 = dx
1 +sin x
解:用代換sinx = 2t •, dx = 2 dt,得
14-t2 l+t2

•175*
l+t2
l+t2+2t
2dt
l+t2
=2
J 0+^ dt
2
1+t
—2 cos 2^

sin兰 x
cos 4- sin
1 4- 2 2 T
cos
~2

習 題24
計算下列各積分:
sin6x co^x dx dx
1). 8).
sin~4x
siiP x dx
2). cos2 x -dx 9).
sin-4x co/x
dx dx
3). cos3 x 10).
J 3+5 cosx
dx dx
4).
J sm~3x cos x ID. 1+3 cos2x

5). J sin4x cos2x dx dx


1+2 tan x

6). sin~6x dx sin x—cos x


13).
sin x+cos x
7). J sin4x cos*x dx _____ dx_____
14). a sin x+b cos x

•176*
x+sin x
15). J /tan x dx
sin x cos x
22)
J 1+cos x
dx

COS? X X2-1
16). J sin4 x
23). X2+" l+x4
dx

r 2又,一4
17) Jv/, sin" 3 2Sdx
cos7 X
24). /
(X2+I)(x+1)2 dx

/ (l-x)Z'2x-x« dx
18). J /讣dx+e*+l 25)

xe51 dx
19). (X+l)2 26). dx
x+sinh x ,
20). J cosh x—sinh x
dx
27).
J
dx_____
21). 1+sinx+cosx

讀者在計算道些積分時, 應該首先找尋快捷的代換,而不應盲目地應
用上節所討論過的公式。

•177-
第7章 定積分(Definite Integal)

當我們去量度一邊界爲曲綫的區域的面積時,我們便遇到定
積分的槪念。更精細的研究使我們能把這個積分的槪念和直觀的
面積問題分開來。並且,使我們能不依賴幾何學的幫助而把它分
析地用數目的觀念來表達。但首先讓我們從直觀的出發點來討論
定積分。

1.用面積來表達一定積分
現假設我們有一函數f(x),在一含有a、b (a<b)二値的區
間內,此函數是連旗的和只取正値。我們考慮此函數的圖象,並
要求計算邊界爲直綫x=a, x=b的X-軸及f(x)的曲綫的區域
的面積,如圖45所示。
必須注意到,我們只假設對上述區域求面積是有一定的義
意。這個假設是基於直觀的,不過,以後讀者在較高級一些的硏
究中就會發現對這個問題可能要作出嚴格的處理 。
我們稱這區域的面積Fba爲函數f(x)在極限a與b內的定
•178 -
y

E045

0 b

積分。當我們眞眞正正的用數字去量度這個面積時,我們發現,
普遍的來說,若區域有曲綫邊界時,這裏測量是不可能的。可
是,我們却很容易的去量度一多邊形的面積,因爲我們只需把它
劃分成若干個三角形和長方形。一般的來說,我們也不可以用這
方法來劃分有曲綫邊界的區域,我們只可能用下面所解釋的辦
法,把所要求的面積看作若干個長方形的面積的和的極限。
首先,把區間(a, b)分作n個子區間(Subintervals),爲了
方便起見,我們可使每個子區間都是等長的。在每個分割點上,
作一直綫平行於y-軸,並和f(x)的曲綫相接。在第v個子區間
內,我們假設函數f(x)取最大値Mv和最小値mv並作二平行綫
y = mv, y=Mv,使得區域處於四邊形之間,如圖46所示。
從圖 46 看出,x=a, x = b, y = 0, y = mv, (v=l,2・・・n)爲

•179 -
y

圆46

y= Mz

x
0 a | b
第二個最間

界的多邊形的面積是小於所求的面積;而以x=a, x=b, y = 0
y=Mv, (v=l, 2-n)爲界的多邊形的面積是大於要求的面積。
如果我們用鸟來表示第一個多邊形的面積,並用瓦來表示第二
個多邊形的面積,則我們得到下面的不等式:
Fn <Fab<FH

如果我們使子區間的長度縮短,或者讓n無限比增大,直觀
告訴我們,瓦和%會互相接近,並二者趨於F?爲極限。所以,
我們可把我們的定積分考慮成下面的極限値;
F」 lim Fn= Um 京
n—>OO n—>00

在上面的解釋中,我們把子區間看作是等長的,這當然是多餘
•180-
的,需要的只是:當n接近無限時,子區間的長度一定要趨於
零。

2.定積分的分析定義
現在我們把上面的推理反過來。我們並不允許從直觀裏導出
的面積槪念,只是從一個純粹分析的觀點出發,首先推導二個多
項式的和,然後再證明這些和都趨於有意義的極限,然後再用這
個極限來定義我們的定積分。
現設一函數f(x),在區間a<x<b內是連續的和只取正値
的,在區間內任何n-1點,Xi、X2・・・Xn.i,把區間分割成口個子區
間,並使x°=a, Xn = b。在每個子區間內,取一任意點(這點可
能是子區間的端點),假設我們在第一區間內取了 fi,第二區間
內選了旧2,……在第n個區間內選了
個區間內選了f 現在不再考慮連續函
數f(x),只考慮下面的非連續函數,階梯函(Step function)
仞(x),
0(x) = f(fi); a£x£xi
= f(?2); Xi 匕 X£X2

= f(£n) Xn.i 匕x£Xn

如圖47所示,這個階梯函數給予我們一系列的長方形,它們面
積之和是用下列式子表示出,
Fn = (Xi —X。)f( £ 1)+(X2 —Xi)f( f 2) + + (Xn—Xn-1)

•181*
y

f(fv)

圖47

I I I; 1
0
7n
(Xv-1) fv Xv

f( f n)或者用求和符號£來表示:
Fn ▼郭-乂心顼八)
引進符號,
△ Xv=Xv—Xv-i
上面的和則變成:
Fn vV( V)
我們主要的定理可叙述如下:
如果分割區間的點的數目無限地增加時(並且,最大的子區
間的長度同時趨於零),則上述的和一定趨於一有意義的極限。
這個極限是不依附於分割點X]...Xn和2區間裏的輔助點E 1.・・f n
•182-
的選擇的。這個極限,我們定義它爲被積函數f(x)在極限&與b
間的定積分。所以,我們可把定理重述如下:如果函數f(x),在
區間a<x<b內是連續的,則在這區間內,函數的定積分存在。
這個定理的證明需要更詳細的分析,這點我們從畧。

3.定積分的基本性質
在圖47中,我們假設了 1)函數f(x)在整個區間內都是正
的,2)b>ao可是,在把定積分定義爲一多項式的和的公式中,
這些假設是多餘的。因爲如果f(x)在區間的一部份或全部是負
的話,我們只需把有關的因子f(f v),改成負的。所以,在X-
軸下面的區域的面積必須看作成負的,這和解析幾何裏我們所熟
習的慣例相同。所以,一般的來說,曲綫下面的面積便是數項分
面積之和,其中有些項是負的。
如果把avb的條件删掉,改用a>b的假設,刖上面定積分
的算術定義仍可通用;因這個改變只使我們從a到b時,子區間
的長度都是負的。所以,我們便得到下面的關係:
f(x)dx = 一 J f(x)dx
不論a與b取任何値(a尹b),這關係都成立。當a=b時,我們
定義
f(x) = O

•183 -
還有,下面的關係也是很容易從定積分的定義裏導出來;
J f(x) dx + J^f(x)dx= J f(x)dx; aVbVc,

如圖48所示。
y

圖48

0 a b c

下面的兩個公式也同等地重要
cf(x)dx=cJ a f(x)dx, c 是常數

b)如果 f(x) =(P (x) + 弦(x),

則 Jbf(x)dx= |C(P (x)dx +


(P (x)dx + jc#(x)dx

最後一點很容易了解到的便是一函數在一一定的區間內的定
積分是一個數字,完全給被積函數和區間决定,它和積分變數
(Integration variable)的選用無關,卽是說,

•184-
J%(x)dt= Jb f(n) dv= J f(x)dx
I例】 用上述的兩種不同方法(幾何的和分析的)計算
f(x)=x在區間(a, b)內的定積分。
解:首先,幾何的觀點給予我們: f(x)dx= fb xdx,只
是圖49中的梯形的面積,所以,用初等的面積公式得出:
y

<=X

圖49

0 a b
x

f(x)dx =—
J A Z
(b—a)(b + a) =—
z
(b2—a2)
現在,我們來證明,定積分的分析定義也給予我們同樣的結果。
首先,我們選下列的點爲區間(a, b)的分割點:
a + h. a + 2h, • • •a+^n—1) h
其中h = (h-a)/no所需的積分便是下列變項式的和的極限:
•185-
h {a + (a + h)+ (a + 2h) H----- F (a+n—lh)}
= h{na + h + 2h----- (n—l)h}
用初等的求和公式(算術級數):
1+2----- (n —l)=*n(n—1)

便可把上面的多項式和寫成:
ah(a+.hZm)= (b—a)(a-h 气兰-旦扌)
如果n無限地增加時,碧趨於1,所以,上式的右端趨
(b2—a2),如我們想像的一樣。
[例】用上面求和的方法,計算積分。乂2火。
解:依上例,我們把區間(。,圮分作n等份,每份之長爲
h=b/no則我們要求下面的和的極限;
h(h2+22h2+ —+n2h2) = h3(l24-224- •••4-n2)
b,(12+22-------瑚)

再用下面的求和公式:
1
124-22+ + n2=—n(n4- l)(2n +1)
得出上面的和爲:
1+普))((22 ++ 普
3~((1+商 2)
當n無限的增加時,我們得到所需的積分公式:
fbx2dx 丄b3
3 °

•186 -
4.計算定積分的方法
上面所述的方法,是很難被用於實際的計算的,因爲只要被
積函數稍爲複雜一點,則一切求和的計算和往後的極限便會變得
過份的冗長,或者不可能了。後者是因爲我們不是常常都能對任
何多項式的和找到一閉合的公式(closed form expression),所以,
極限的運算就變成非常困難。
現在,我們來證明下面的定理:
如果函數f(x)在區間a£x£b內是連續的,並且F(x)是
一任意函數在區間內每一點都滿足條件
F,(x) = f(x)
fb
則 f(x)dx=F(b)-F(a)。

證:我們知道,一函數在一給定的區間內的定積分是被這區
間的端點而决定。現在,我們把這積分的下限(lower Umit of inte­
gration) 保持不變,而使積分的上限(upper limit of integration)
變動,則我們得到這定積分是决定於上限的位置。如果上限本身
是一變數・x,則我們的結論是:在給出了被積函數和積分的下限
時, f(t)dt是-乂的函數。我們用9(X)來表示這個函數;
0(x)= J: f(t)dt
現求這函數的微分導數。用定義,我們得
"(x)
lim (x+ △心 <p (x)
△jo Ax

•187 -
lim
ax—o
{L f(t) dt + Ax

lim
AX—o
{L△ xf(t)dt}
我們引用下面的定理(積分的中値定理):如果一函數f(x)在積
分區間內是連續的,•則
[b f(x)dx=f( x)(b—a)

其中X是區間內的某一點。這一定理的證明很容易,因爲對課文
的發展沒有很大的用處,故我們留給讀者自己去嘗試。
應用這個定理,我們得到
0 '(x)= lim f(X) lim f(X)
>o A x— o

因爲X是區間(X, X+AX)內的一點,故當厶乂一。時,X
所以得到”.(x) = f(x)。由此可見,在定理裏面的函數F(x)是
f(x)的定積分,其中積分上限是變數x本身,卽;
F(x) = J J(t)dt + c, a< x < bo
式子右端含有一未知的常數。現使x=a,再次使x=b,則得
F(b)-「f(t)+c
F(a)= f(t) + c=c

所以,得出下面很有用的公式,
•188 -
b
F(b)-F(a)= jb f(t)dt=F(x)
其中F(x)是f(x)的一反導數。
E-
【例】用上面的定理,計算:丄二备
解:用上一章求反専政(或積分)的技巧,我們知道F(x)
tan-1(x),所以

匚 dx = tan-1(x)
14-x2
tan-i(l) — tan-i(一1丿=手
(
7T n
五。
上面的定理告訴我們,假若我們知道被積函數的一個反導
政,刖這函數在任何區間內的積分很容易求得了只需計算此反導
數在積分的上限和下限,二者之差便是所求的結果了。因此,上
面兩章對反導數(或積分)的計算和定積分的計算是有很密切關係
的,從此,我們把f(x)的反導數J f(x) dx叫做函數f(x)的不定積
分。我們應該分別它們的差別,f(x)的不定積分是一新的函數,
並且取原來的函數f(x)爲導數。定積分fb f(x)dx則是一數字,
它的値只决定於積分的區間,而且,這個積分的定義和微分沒有
關係,因爲原則上來說,我們可以永遠不利用反導數來計算定積
分。爲了操作的方便起見,我們證明了上面的定理,利用了反導
數而不需每次在計算一定積分時,都要經過冗長的極限計算。
【例】計算f (2 sin x +cos x)dx

•189-
解:用上面定義過的符號,F(x)'=F(b)_F(a),得
7T
0^/2
(2 sin x+cosx) dx= —2 cos x + sin x
o
= (0 + 1) —(一2 + 0) = 3。

習 題25

計算下列各定積分:
,p pdx
x2 dx 9).
o v/p2—x2
,i dx
2). f: sin xdx 10).
o / 3-2x
f» dx
3). Jo^+xz ID. I:令。2-旳^
12) .
4). Io- sec4^ d0
Jo x+l

5). 13) . 2a J*: (2+2 cosB)*d。

14) . Jo
f S. dx
6). jf"o (a2x—x3)dx
,2-3x2
7). 15). 啓 sin3 a cos3a d«

8). :(x2—2x+2) (x—1) dx


0
16) •財 dy
o y2—y + 1

•190-
5 cos。d。
I + sin祐 皿£ cos2 -^-dt
2
t dt adx
mJ:j '
a/2
⑻・』;-l+t2 (x—a) (x—2a)
dx
19)•匚 (11+5x)3

5.定積分的代換
我們用一個例子來說明怎樣利用代換來計算定積分。
【例】 計算「X — 4+5xdx
J0
解:用下列代換u=i/4 + 5x,或u2=4 + 5x,則2udu
5dx,所需計算的積分可被寫成:
J x/4+5x dx = J -J-(U2 - 4) u-y-udu

=妾 J (u*-4u2)du
我們觀察到,當u=2時x=0,當u = 3時,x=lo所以,我們

x/ 4 + 5xdx=表
Jo x/4 5xdx = 3 (u4 —4u2) du

=妾(土5 _訓): 506


'375
其中最重要的地方當然就是對應於u變數的積分極限和對應於X
變數的極限是不一樣的。這一小節,在計算不定積分時是遭遇不
•191・
到的。現在我們把從上例題得到的結果一般化起來:
設一函數f(x)在區間a〈x<b內是連續的。現引進一新的
變數u,並使x依附於此變數。再設u的定義域爲«<u<A,
並且下面的條件是滿足的;
dx 在區間a<u<"內是連續的;
(i) du
(ii) 當u處於區間a<u<0內時,不處於區間a<x<b內;
(iii) 當 u= a 時,x=a;當 u= P 時,x=b。
則如果代換x=x(u)使f(x)dx變成cp (u)du,下面的結果是成立
的;
<p (u)du
a
我們不對這個定理加以證明。

習題26

用適當的代換,計算下列的定積分
、fTT/2 dx
1). Jo XL dx 5) . jo - 2 cos x+3

2). J 1/2
dx
x/1—x2 6) .户 / 1+X2

xdx 7) " cosn xdx


3土 i+r
dx
4)•冬 x/1+lnx 8) ・ J: sin 豎dx

•192-
dx / 16—x2 位
9)・ Jo (x2+16)2
1。) j: x

6.用積分來計算面積
設現要計算邊界爲曲綫y = f(x),直綫x=a, x=b, y=0的
區域的面積,並且,如果在區間a<x<b內f(x)是連續和只取
正値的話,則從上面的討論中,我們知道所需的面積是用一定、積
分來表示:
A b f(x)dx
現在我們把這個問題稍爲推廣一下;假設需求邊界爲二曲綫
y = fi(x), y = f2(x)和二直綫x=a與x=b的區域的面積,其中
fi(x)與f2(x)都是連續函數。我們假設此二曲綫不相接,除非
在區間的端點(a, b),如圖50所不。
讓區間(a, b)被分割成子區間,其中分割點xo, 乂丄…今滿
足 a=X<J<XX<X2<---<xn = bo 並使 △xi=x]_-X2, Ax2=x2—
xr--o則平行綫x=Xi, i = 0, 1, •••□把區域分畫成n個平 行的
子區域,其寬度爲每一個子區域和一長方形相像,
只不過兩端的邊界都是曲綫。現考慮一典型的子區域,令其邊界
爲X = Xi_i及X = Xj。如果Xi滿足Xi.1<xi,<xi,,則平行於y-軸
的直綫X = Xj是處於這子區域內,並且,此直綫和曲綫相交於
X = x/, y = f2(x/); x=Xi,,y =fi(x/)o 現在此子區域內作一長
•193*
y

y = fz(x)

圖50
y=fi(x)
Axi
0 x=a x b

方形,使其邊爲直綫y=f2(x《),y=f2(Xi'); X=X」1, X=Xi。此


長方形的面積爲
如果我們用同樣的辦法在每一子區域內都畫分出類似的長方形,.
則這n個長方形的面積之和可被看作爲對整個區域的面積的近似。
準確的面積當然應該是這個和的極限,卽是當n一8並且當最大
的厶犯也趨於零時的極限。像上面所討論過的一樣,我們得出所
需的面積爲:
A=「 g) — fi(X)〕dx
【例】 計算邊界爲抛物綫y=x2和直綫3y=4x的區域的
面積。
•解:直綫3y = 4x和抛物綫相交於二點;(0, 0),(y,普)。
•194-
4
我們作平行於y-軸的子區域,每一子區域的長度爲yx-x2o 一典
4
型的長方形(如圖51所示的)的面積爲(yx-x2) Axo所以,要
求的面積是用下面的積分表示出:
y

圖51

-x
0 舍

A= 铮-乂2)吹=(* 弓 X,):”
32 64 32
一~27" _ ~8l 8F
【例】 依上例,計算邊界爲二抛物綫y2=-4(x—l);
y2=-2(x-2)的區域的面積。

•195*
習題27

求在X軸以上,抛物綫y2=4x以下,夾於縦標x=4, x=6間的面
積。
2) . 求囿於抛物綫y=4-x2及x軸間的面積。
3) . 求囿於x軸、y軸、及抛物綫xi+yi=ai間的面積。
4) . 利用積分方法,求由直綫y=5x, x軸、直綫x=2所組成的三角形
的面積。利用初等數學(例如算術)方法驗證計算的結果。
求給出曲綫、x軸及縦座標包住的面積:
it
5). y=sin x, x=0, x 2~
6). y,= —x2+9, x=—3, x=3

7). y= l+x x ,x=0, x=8o


2
8) . y=x2+x+l, x=2, x=3
9) . y=x3+3x2+2x> x=0, x=3
10) . y2=—4x, x= — l, x=0
11) . xy = a2
12) . 求兩抛物綫y2=4x和x2=4y間夾住的面積。
13) .用積分方法求由直綫5x-y-15=0, x-y + l=0, x+3y—3=0所
造成的三角形的面積。
14). 求囿於曲綫y= 及直綫y=*之間的面積。
15). 求囿於曲綫y=cosx、x、y軸,及直綫乂=告的面積。
--—… 6 -

16). 求囿於曲綫y2=2x+l和直綫x-y—1=0的面積。

•196*
—X—2
17).求囿於x軸和曲綫y= X2对+8 在其與X軸兩交點間的面積。

18).求糧圖形* + 苦一=1的面積,倘若a=b,求圆形的面積。
19) .求囿於x軸和擺綫。
x=a(t—sin t), y=a(l — cos t)
內一弓形(卽0<t<2^)的面積。
20) .求囿於閉封曲綫y2=x2-x4的面積。
21) .求包含於曲綫x = t2-l, y=t3-t的環內的面積。

7.旋轉體的體積計算
考慮一平面區域,設其邊界爲X-軸,直綫x=a, x=b及一平面
曲綫y = f(x),其中函數在區間(a, b)內是連續的。如果我們把
這平面面積沿着X-軸來旋轉,這樣我們便得到一旋轉體了。習
知的旋轉體有正的圓柱,正的圖錐體(right circular cone)等等。
又例如,我們可把球體看作是一半圖沿着它的直徑被旋轉而成的
立體,如圖52所示。
爲了要求這種立體的體積,我們也採用面積問題裏用過的極
限辦法。首先,把區間(a, b)分成n等份,並設Ax爲一典型的
子區間的長度;y = f(x)是曲綫在子區間左端的縦軸。刖對應於
x = Xj, i=0, 1, •••!!的平面把此立體分爲n片,如圖53。
像圖53的虛綫所指出的,在一典型的小片中,我們可劃出
一底半徑爲y = f(x),高爲AX的圓柱片。此圓柱的體積爲7ry2Ax9
•197-
像在面積問題裏一樣,這些圓柱片可被看作對原來的小片的近
似。所以,旋轉體的體積是所有的圓柱片的體積的和的極限;
n (y02 △xi+・・・+yI1_i2 Axn)
這當然指當n趨於,並同時最大的AXi也趨於零時的極限,可是,
這個和都是一定積分的定義:
V=7T P y2dx
[例】計算球體的一節的體積:
解:設球的半徑爲a,節的高度爲h,如圖54所示,我們可
把球看作爲圖x2+y2=a2被沿着X-軸旋轉而成的旋轉體。所以,
y2=a2-x2t 及 (a2-x2) (3a-h)
習題28

i). 求繞着x軸旋轉的糧回形。告+善 =1生成的立膻的體積。


2). 球體是由圓形繞着圓形的某直徑旋轉而生成的,今球體由回形X2+
y2=V繞着某一直徑旋轉而生成,求它的體積。
3) . 求由圆形x2+(y-b)2=a2橈着OX軸旋轉生成的環體的體積。
4) . 求繞着OX軸旋轉的曲綫:
(x—4a)y2=ax(x—3a)
而囿於x=0和.x=3a間生成立體的體積。
下列各題,面積均由不同的軌跡包成。求這些面積圍繞OX軸旋轉生
成立體的體積:
5) . 抛物綫 x*+y^=a*, x=0, y=0o
6) . 抛物綫 y2=4x, x=4o
7) . y2=9x, y=3x。
8) ・ y=xex, x=l, y=0o
9) . 圓內旋輪綫x务+y^=a务。
10) . 兩個相等回柱半徑爲a,其軸正交—
—相互垂直相交。
求這兩柱體相同部分的體積:
11) .求囿於y=x2+sin x, y=0, x=0, x=-^-範圍內面積繞OX軌旋
轉生成的體積。
12) .求囿於攏綫x=a(t—sin t), y=a(l—cos t)—弧和x軸的面積橈
x軸旋轉體的體積。
13) ,抛物綫xi+yi=2和直綫x+y=4包成的面積,繞着直綫y=x旋
轉。求這樣生成旋轉體的體積。

•200-
14).囿於曲綫y x2+l 、X軸及兩直綫X=L x=2之間的面積,圍
繞X軸旋轉,求這旋轉體的體積。
15).某樋回柱體,爲沿短軸分割其底的平面所截,求圖55所示的槌形立
體的體積。

h
2b
圓55

•201-
第8章無窮級數

1.級數的槪念(Infinite Serie)

在開始討論無窮級數之前,我們首先温習一下一些熟悉的槪
念,
考慮下述幾何級數(Geometric series)o這個級數是個序列;
a, ar, ar2, ar3---arn••-
其中a與r爲任何常數。觀察到邳「是級數中的第(n + 1)項,第
□項是 arn-io
以下是幾何級數的例子:
— 1 1 1
1111.,~. . . 1、
,力》 8',T?
百 16 “(a=l,,=顶)
2 9
b)苗 99 9 ...(a=2
•••(a r=丄)
10,Too_, 1000,10000 , k 10, 10?
C) 2, 6, 18, 54, 162, --(a = 2, r=3)
現設我們需求起初n項的和,用Sn來表示這個和;
Sn =a + ar + ar2----- arn-1
如果當n趨於無限大時,Sn趨於一有意義的極限,則
•202*
S = a+ar + ar2----- arn-1H—
的點表示S不是把有限多個項加起來,這個寫法
在上式,arn-i
純然只是符號性而已,它的數學意義應解作
S= lim (a-f-ard- ••• 4-arn-1)
n—>8

[例】 再察看上面的幾個幾何級數。在(a)裏,如果我們
把很多個項加起來,它的和趨近於2,所以,用求和的公式寫出
來,我們得
1 1
2=1 +"5"+
2 _7_----- +
在(b)裏,觀察到

10 十 100 = 0.99; 誑高+彘 = 0.999

所以,起初n項的和是
Sn = 0.999・・・9 (n個小數位)
並且,我們得到S=l,因爲
1 —Sn .00-01
= 若 n->8的話。
所以,我們得下面的表示;
1 2+里 9 9
10口-1
10 100 1000
在(C)裏,和Sn可寫成
SnF + 2(3) + 2(3)2+... + 2(3)n-i
當n趨於無限大時,Sn-8,故Sn不接近有意義的極限。

•203-
在初級代數中,我們已學過一些初等的公式,
Sn=na,如果 r=l
a (1-L) ,如果17^1 o
(It)
從這些公式,我們看到
S= lim Sn=0,如果 a = 0

S= lim Sn a ,如果 一 IV r VI
1— r
又如果a尹。及|r|〉l的話,則當n趨於無限大時,S。不趨於無
何有意義的極限。所以,如果一IVrVI,則得
=a+ar+ar

這些對幾何級數的考慮,很自然地讓我們引進一般無窮級败
的研究。設ui, 口2, -uQ,爲一序列的數字並且假若給出了
我府有一有着意義的規律使我們能寫下Ug設
Sn=Ui + U2------- %
爲起初n項的和。如果當n趨近無限大時,Sn趨近於一有義意的
極限,則我們得
S=U] + U2------- F un + • • •
上式右端的式子一—如果我們不知道當n趨近8時,Sn是否有
有意義的極限的話 —
—是純粹符號性的式子。這式子叫無窮級數
(Infinite Series)。數字金,S2> S3,…叫作級數的部份和(Partial
Sum):
定義:如果當n趨於無限時,Sn接近一極限S的話,上面

•204*
的級敷是收歛的(Convergent), S叫作級數的和,或級數的値。
發散的無窮級數是個不收歛的級數。
定理:以ar11-】爲第n項的幾何級數是收歛的,如果|r|<lo
如果a尹0與|r|>l,則此級數是發散的。
【例】考慮0.373737…可寫成
37/100 + 37/(100)2 +37/(100)3 +…
所以,在這裏a = 37/100, 1=1/100,故級數是收歛的,並且其
和是
37 37
Too 1
1 99
100
一個不重覆的不盡小數是常可寫作一個收歛無窮級興,但這
個級數並不是幾何級敷。設小數爲0.ax a2 a3 -,其中a」是小數
的第n個位,例如在
n -3=0.14159…
ai=l, a2 = 4, a3=l, a4=5 o對應的級數則是
ai +-a? d-----
a2-+ a3f+・・T F +…
10 1Q22
10 103 10n
此級數是收歛的,並且,它的和就是用這個不重覆而又無盡小數
來表示的實數。其實,通常我們都用上面的級數來定義我們的小
數。順便提出,在別的應用中,如電子計算機等等,10這個基數
(Base)常常被2所代換。
如果S是一收歛的級數的和,用這個級數,我們可把S的値
計算到任意準確程度。因爲如果Sn-一S,則我們可用部份和
U1 + U2 + …+un

•205 -
作出對S的近似,而且,如果n是充分大的時候,這個近似値是
無限地準確的。
在實際的數學計算,我們當然希望隨着n的增大,很快
地便會減少,所以,我們只需幾個項的和就可得到一個對S很好
的近似。當這個情形出現的時候,我們說這級數收歛得很快。往
後在這章裏我們會學到怎樣來决定一級數的收歛和發散 。下面是
一個很簡單但很重要的定理:
定理:如果一級數收歛,則當及趨近8時,它的第n項1^一
定趨於零。換句話說,當n無限地增大時,如果Un不接近零的話,
則級數一定不收歛。
證明:我們知道,Un=Sn-Sn.i,並且,如果級數是收歛
的話,則部份和Sn與Sn・i都隨着口的增大而趨於S,所以,
lim un = lim Sn — lim Sn-1=S —S = 0
n―> oo n—> oo n―> oo

我們應該小心,這個定理的逆定理是不對的,就是說,如果我們
從Un— 0當n— 8而結論一級蛟是收歛的話,那就是很不應
該是錯誤了。

2.對數函數和反正切函數的級數
在這一節裏,我們要推導下面的公式:
In (1 4-x) = x — ~x2 4--t-x3 — ••• 4-( — l)n-1 "z~xn4- (1)

•206 -
1 1+x 1 3 +-|-x5 +…4
x 4—J-X .x2n-l + ...(2)
—In 2n—1
2 1 —x 3
丄 (一 1)~ x2n-i
tan"1 (x)= xz—j-x3 +
5 -y x74----- F 2n—1
H-- (3)
當一 1VX<1時,公式(1)是眞確的;當一 1VXV1時,公式(2)
是眞確的當一l<x< 1時,公式(3)是對的。所以,如果在公
式(1)和(3)裏,我們使x=l的話,則得到下面有趣的公式:
In + —+・・・+
In 2=1—-y+-| ----- (_1)~

7T | 1 , 1 1 +
. … .[(
( _1 )~ 卜…⑸
— IT
石=11 —
-丁丁+
+M - !"+•••+ 2D-厂
現在我們考慮公式(1)的推導。從幾何級數開始
1—t + t2—t3d-
1+t
tn
…+ ( — l)nTtnr+(-l)n
r+t •••(6)
現設一 1<X<1,求公式(7)兩端從0到X的定積分,得出
ln(l +x) =x—yx2 +-yx3 —
3
•・・+(-1)~卜+(一1)“ x tn dt
o 1+t
對於一預定的x,讓我們下定義:
Sn(X)=X —--X2 +§X3. .• + ( — 1 )E 1 (7)
3 Tx
x tn dt
Rn (X)= (-l)n (8)
o 1+t

•207-
則ln(l+x)=Sn(x) + Rn(x),並且,我們必須證明
lim Rn(x) = 0
n—>oo

公式(9)是對的,如果一IV x< 1的話。證明如下:


i)如果x=0,則對於任何的n, Rn(x) = 0,所以,公式
(9)成立。
ii)現在我們分別考慮0Vx< 1和一l<x<0兩個可能
性:
a)如果0 Vx< 1的話,則0<t,所以,
tn
<tn
(1+t)
xn+1
故 |Rn(x)|< £ tn
TTt dt<
Jo n+1
n+ 1 n+1 '(10)
在這裏,我們應用了一個很著名的不等式,那就是(a+b)<|a| +
|b|其中a, b是二任意實數(其實,這不等式也適用於複數)。
在上一章裏,我們定義一定積分爲一多項式之和的極限,卽
n
s f(xD A Xj
「f(x)dx= lim i=l
J a n—>oo

所以, n
I J:f(x)dx 卜 lim
n—>oo
§if(Xi) A Xi I

< lim s
i=L f(Xi) A Xi
n—>oo

在這裏,我們用了上面的不等式的簡單推廣。再用定積分的定
•208 -
義,便得到
J:f(x)dx |< J: |f(x)| dx

這就是公式(10)裏第一個不等式的來源。
現在,我們反過來討論公式(10)o旣然
RS |
Um RJx) = 0的條件是滿足了。
所以,n-^oo
b)如果一 1VXV0的話,則用代換t=-u,則
un du
RnOO=—
1—u
現在我們有0 <u<-x<l,所以
------- 1___
・1+X
OX" £ 1
及 ®(X)IV 厂斋 、(n + l)(l+x) (11)
(n + l)(l+x)<
如果n趨於無限大的話,公式(H)末端的式子很顯明的趨於零
To所以,公式(9)是成立的。
公式(2)的級數的推導很容易,我們讓讀者自己嘗試一下,
作爲練習。
現在我們考慮反正切的級數。在公式(6)裏,我們把t換成
t%刖得
1
l + t2 l-t2 + t*----- 4-(-l)n"1t2n-24- (_l)n
l+t2
求這公式兩端從t=o到t=x的定積分,得出
•209-
tan~1(x) = x—|-x34- -yx5 «- + (_1)〜 X2n-,
2n-l

+ (—1尸
在這裏,如果我們有一 1 Vx〈l,則
(一1)「
o 1+t2
dt < t2°小=蜡〈由 |x|2n+l
2n + l
其中第二個不等式是不難得出的,讀者稍爲思量一下,便會
覺得|X|是對的積分上限了。
所以,如果我們使
Sn(X)=
n /_、 —X-gx3
1 - +&C5...
1 + I X2n-1
Jc — 3 人 丁 5 2n-^T~
我們觀察到,若一 1<X<1的話,則
|X]2E _1
|tan・i(x)-Sn(x)| < 2n+l <
<211+1
所以,Sn(x)->tan-i(x)當n趨於無限大的時候。這便證明
了公式(3)是對的,並且,公式(12)說明了如果用Sn(x)作爲
一對tanT(x)的近似的話,則所引進的錯誤不會大過無窮級數裏的
第n+l項。
利用公式(2),我們可把任何正數的對數寫成一無窮級數。
例如,如果y>o,使
—19- l+x
x= y__
y+i ,卽y= l-X
刖一 ivxvi,所以,
In y=ln l+x
l-X

2 (x-F-yx3 +~yx5 ■
3
)

•210-
如果y是很大的話, x接近1,所以,級數的收歛很慢。如果y
和1接近,則x=0,這時,級數收歛得非常迅速,並且,可被
利用爲計實際的計算工具。

3.重要的定理
我們討論一下研究無窮級數的三個重要定理,其中上文已經
提及過的。
a)若級數,Un爲收歛,則lim un = 0o這定理的逆定理
n—>oo
是不成立的,就是說,給出一項級數,已知n—Hm
>oo
心=0。
刖,Un的收歛性不是必然的。
【例】考慮級蛟£普= 1 +*+}+++£+…。
當 n—>oo % ± n

但 是個發散的級數。
從這個定理,我們可以得到一個單邊判別法(Qne-side Test):若
級數的n項不趨于0,則該級數爲發散級數Q
b)級數各項乘以非零的常數,則發散性或收歛性不受影
響。理由很顯然。我們知道,若級數各項乘以k,
則 Sk Un = k£Un。 (k尹 0)
C)從無窮級數中,除掉有限多個數項,或增加上有限多個
•211-
項,亦不影響級數的收歛性或發散性。

4.怎樣判別正項級數的收歛或發散
每一項都是非負的級數,稱爲正項級數。未進行討論判別
方法前,首先引入一項等別的級數,叫做P級數。P級數的定義

底「萨=lp + 2P+3。+妒+…
這裏p爲一個常敷。若p>i,這項級數收歛,若p£i,級數則
爲發散的。當p=i時,我們得到級數
1+§+}+*■+}+…
就是發散的調和級數(Harmonic series)o
a)比較判別法(Comparison Test)
這判別法適用於各項爲非負的級數。
1)收歛性的判別:考慮兩項級數2L和SVno若,Vn爲
收歛,而當一切n>N時,令vn <0o若果一切n>N及。〈七
<vn,則,七也是收歛的。留意我們說n>N,表示當n大於一
定的數量N的情况,卽指級數由某一項開始的意思。通常我們用
N=lo
【例】 首先證明£寺爲收歛級數:

•212 -
8 1 1 1
2> 5厂 22 23 24
開首n項的和是
2n
,而且nHm
—>oo
(1 A=i,故這級數是
收歛的,而其和爲S=l,現在考慮到:
I
8, 2n + l
旣然2 +爲收歛,利.用比較判別法,卽知道£ 井T 亦爲收歛。
厶 厶 i" JL

2)發散性的判別:若級數Svn發散令n>N的一切n値都
有vn>0o若對一切n>N的n値有un>vn,那麽級數:Sl也是
發散的。
【例】因爲菖并發散,又因爲 [ 丄 ,故級數
>云
In n

OO

s - In[ n 亦爲發散。
n=2

3)比較判別法的證明:比較判別法是個定理:若0<un<vn
n=l, 2, 3, 4,……而且,若2vn收歛,那麽Sun也是收歛
的。
vr + Vv2 4+
證明:令 Sn = Ui + U2+・・・+、n>,Tna = V --- v no
- •••+V 由於
爲收歛,那麽n—lim
>oo
Tn存在,同時等於一個數量,叫這數
量做T,就是說nUm
—>oo
Tn = T。

•213-
又因vn>0, Tn<T,於是:
Sn = Ui + U2 ------- Un<Vi + V2--------- Vn<T
或作O〈Sn〈T
sn爲有界的單調遞增數列,它必有個極限,所以Sun是收
歛的。
【例】證明+ 3
丄 爲發散級數。
解:我們知道
1 > T
1 1 1
t+t
3 > —4--—:
- =T
4十Z

4+
丄 十丄
5 丄+丄 8+
A 丄
6+7 > +丄8+
+丄8+
+丄-丄
8"2
丄 +
+丄丄丄
9 +To +・.
丄丄丄 +…+讦=万(共八項)
8 十 * + 15 >讦+话+毎
k k
'一

如此類推,因此,不論寫到第幾項,都有
1+(*+*)+(+ + 丄
++++*-
5 ) +・・・>,+*+4-+…
如果項數越多,那麽上列不等式的右方,可以任意增大。取足夠
多個項時,右方的和可以大於任何數員。因此,級數,§是發散
的。
b)商數判別法(Quotient Test)
這個判別法,實是比較判別法的變相,不過,它自身的形式
往往比比較判別法更便利。
•214-
1)考慮兩個級數,%和2vn,設un>0, vn>0而lim
n—>8

Un
=好0或 8, 那麽如gVn收歛(或發散),則有亦爲
Vn
收歛(或發散)。
2)如1=0(卽nlim 宰=0),又若,Vn收歛,那麼
—>oo Vn

是收歛的。
3)如1=8 (卽、虬 #- = Z=8)而Zl發散,那麼
ZU”也是發散的。
我們證明第一種情况,旣已知I爲学
vn
的極限,那麽已知
€ >0,我們可取整數N,按一切n>N的n値令 %
Vn

於是,n=N+l, N+2, ---- € < Un l<e


Vn
或寫作(A — € ) vn<un<(A+ e ) vn...................... CD
求從N+i至8的和,說得更確切一點,求從N+1至M的和,
再令M—8。
oo oo oo
(A - € ),Vn< S Un<(A+ e ) S Vn (2)
n+i n+i n+i

令A— €>0也不會導致失至一般性。從不等式(2)的右方可
知,,Vn收歛時,也收歛,從(2)不等式的左邊得出:若
,Vn爲發散,那麼,%也是發散的。

•215 -
In n
【例】級數, ,是發散的,因爲我們取極限
In n
n =8。
(n +1)*

n
【例】考慮級數, 4n3-2
是否收歛。
n 1
利用商敏判別法,取極限lim n2. n—>OO 4n3-2 4°

我們知道汁旣不等於0,也不是無限大,所以要試驗的級數
是收歛的。
In n In n
留意上兩例中, n
'(n+1)*
•可寫作 (n+l)*.
J
而 . ・n
可寫作 n2
'O 可知這是應用商數判別
4n3-2
法。 n2
I例】試判別下列三項級數的收歛性和發散性:
oo
⑴ S柴彳,(ii) S 黑牛(団)S -
(ii) M
In n 4n2—n4-3
n=i ,n=£ n2 + 3
2n° — 1 ~~n3 + 2n
n=i n4+ 3 n=i

n + lZ五 n
(i)若n充足大的話, 2n3_]
與・
2^ 2n2 很接近。令
n+iZ~n~ 1
2n3-l f
Vn
2n2 考慮極限
Un
Vn
1,因爲級Svn = yS^--收歛,所以級數

•216-
oo
n+iz云
s
n=l 2n3-l
是收歛的。
(ii)不妨先將商數判別法寫作一項定理:令Um npun = Ko
甲)如果P>1而K爲有限,那麼2七收歛。
乙)如果P<1而K尹0(K可爲無窮大或無窮小),那
麽$Un發散。現在再囘看例題。我們寫
lim
n—>oo
n2
3
(皋)<地旳毋)= lim
n—>oo
In n
=0

3
(應用洛必達規律)。那麼,p=亍
2 ,所以可知這級數是收
歛的。
4n2—n + 3
(iii)像(i)那樣,當n充份大的時候, n3+2n~ 接近
4n2 4 4ii2-n + 3
n3 n 令土 = ~~n3 + 2n
而令vn = n
那麽如1 - 了 -
n—>oo Vn

1,因爲机= 42*發散,所以£七也發散。
【例】試判別下列兩級數的收歛性或發散性:
S sin,(\§
(i) n=j ), n=i
u /
(ii) S e-址
(i)當n很大的時候,sin (号)就很接近 這樣導致我們
sin
考慮 lim n3 sin3(§) lim
n —> oo \u / n-r> oo

•217-
這一來,用上一例(ii)的定理,p = 3,表示給出的級數是
收歛的。
(ii)利用洛必達規則,nlim
—>OO
n2. e-n2=0o再利用上一例(ii)
所述的定理,P=2,那未,級數是收歛的。
c)積分判別法
此法適用於只有非負項的級數,設函數f(x)在n大於某一
定數N時,就是x>N, f(x)爲正、連續而單調遞減函,並且
f(n) = un, n=N, N+l, N+2,……,那麽,級數會隨積
OO M
分 *•N f(x)dx=M—lim8 N f(x)dx是否收歛或發散而收歛或發
散。特殊情形是N=l,那是時常眞確的。
OO
[例】試判別級數n=i £ 的收歛性q

考慮 lim
M—8
.m dx _ lim
1 M—8 ■-净 1,就是說,乘分
•收歛,(因上述極限存在)。因此2 n12 是收歛的。
2 OO
n 的收歛性或發散性,就
【例】考慮和 ,i n2+l
oo 2
第一個級數,考慮積分 i xe-' dx是否存在,卽看看
roo j
L xl dx是收歛還是發散。現在
M 2 2
dx = lim xe lim — 2
M—8 M—8

•218-
lim
M—8
仕/_多財}=多疽,顯然
oo oo
xe-x dx爲收歛。所以級數Sne-n2是收歛的。

至於第二個級數,我們寫出積分:
8
xdx ,M xdx
X2+1
lim
M-*8 1 X2+1
lim
M—8
*m(x2+i)|:

=MHm曷 In (M2+l)-—
—8 I z 2
In 2

故這個積分是發散的。就是說,其極限不是有限的。所以級數
oo
n
si n2+T 是發散的。
d)交錯級數的判別
級數有正負項相間的,叫做交錯級數(Alternating series)。
交錯級數如滿足下列兩個條件:
(1) K+i| < |un| n> 1
(2) Um un = 0 (或作 lim |uJ=O)則爲收歛。

【例】級數1一 *+1+1+ …项 (一1)~


2 3 4 5 6 n=i n
(一1" 1
現寫第n項爲Un n ,I % I 8,
於是當 n>l 時,|un+1|<|un|o
I Un*l I = n + 1'
此外,Hm |un| = 0o所以這交錯級數是收歛的。
•219-
OO
(一 I"
【例】試證明級數n=l
S 2n-l 爲收歛。
1
這級數是1 一 }+} 1
1 —*■+*-
3 5 9
I" ,那末這第n項的絕對値是an = |un|
令% ( —2n-l
I
2n-l ,第0+()項的絕對値是an+1=|un+1| = 2n + l ,由於
I ]
-1—< 2n-l
2n+l ' 及由於nlim
—>oo 2n-l
=0,這級數的收歛性便獲
證實了。
e)絕對的收歛和像件收歛
給出一項級數SUno若S)Un |收歛,刖£土稱爲絕對收歛
(Absolute convergence)^ 若,七 收歛,但 2 |un| 發散,就
稱爲條件收歛(Conditional convergence)。
顯然,若S|un|收歛,Sun當然也收歛。換句話說,絕對收
歛的級數也是收歛級數。
【例】級數T~+ 寿
32
+ + —
是絕對收歛的,因而也是收歛的。這是因爲絕對値級數

14- *
1丄+ 321丄 +是收歛之故。
-p-+~^-+ + 42

【例】考慮級數1 一土+ 丄一土


3 +…。利用交錯級數的
1丄1丄4- 1 +…爲發散級
判別,可知它是收歛的。但是,1+五+
一 +至+习

•220-
數。因此1一~ +}—++••.是個條件收歛級數。-
sini/万
【例】試硏究級數 siniZlT
的收歛性。
這級败的各項的絕對値,都小於或等於級數:
崂 + 23 + 3| +
的各對應項。但這是個收歛的P級數。因此第一個級數絕對收
歛。
[例】試判別下列兩級數的收歛性和絕對收歛性:
OO co
(一 1)~ n (一 1)5 2n
(1) 2 (2) 2
n=l n2+l fl2

OO
n
(1)利用積分判別方法,可知絕對値級數"+S1 n2+l •,爲發
OO
散(見交錯級數的第2例)。因此,級數n=i
S ( — IL n
Q2+1 不可能絕
n 及 an+l= |Un+l| _ n+1
對收歛。可是,令an = |un| =
n2+l — (n+l)2 + i
lim n
於是有an+1<an (n>l時),而且lim an 0,
a—>oo n2+l

故已知級數是收歛的。這級數收歛,但不是絕對收歛,叫做條件
收歛。
(2)用單邊判別法,求這級數第n項的極限。
-221-
(_[)n“ 2n
lim un
n—8
lim
u—>oo n2
尹0。故給出的級數不可能收
2n
歛。要證明Hm
• n—>00
&尹0,只需證lim i |un|= lim
n—>oo n—>oo m2 *0就
行。說得具體一點,我們用洛必達規則運算如下:
lim 2L= lim 22 In 2 In 2 lim 2n
n—>8 n2 乙竖 2n
n^>8 2n 2 n—>oo n
In 2 lim
~2~ ,一> 00 1 2
f)達朗貝ss判別法
達朗貝爾判別法(D,A13mbert,s Test),又稱比値判別法
(Ratio Test)。已知級數Sun,若極限
lim
n—>oo Un I
(i) .若“VI, 2土收歛(絕對收歛)
(ii) 若 A >1, 2Un 發散。
【例】 試判別下列級數的發散性或收歛性:
oo
⑴豎»=1
* e-2 ,(2) S
n=l
-(書
(一1)『n
址+1

DL^+l
♦, ⑶
,(3)”ZS 日)广2
n=1
=l H2

(1)這級數的第n項是*-吃第(n + i)項是(n+l)4e-(n+D2o
(n + l)4e -3+1)2
lim 41
n—>OO %

lim (n + I)4e-(n2+2n+i)
n,
n—>oo
n4 e-n2

•222-
lim
n—>oo
(玲1)切2卜1
lim /n+l\4 lim -2n-l
=n—*OQ I ~~~) n—>8 D = 1-0=0

這裏0V1,故已;知級數爲收歛。
(2)這裏第n項L是 (_1广七 爪2+1

n—>oo
凫中叫
lim I un4-l n2+l
lim ]( — l)n(n+l) _____
■" I (n+l)2+r ( — 1)
lim (n+1)3+1)
=lo
=n—>oo
(n2+2n+2)n
顯然,這判別法不能决定級數是否收歛。就是說,對於這級
數,達朗貝爾判別法不合用,讀者可自行試用交錯級數判別法,
證明它是收歛的。
(3)這裏地= n2
那末,
. n2
lim Jl lim |(_1T
n—>8 % n—>oo I 8+1)2 一 (-DFn
2n2
lim =2
n—>oo (n+152

現在有2>1,故已知級數發散。
g)柯西判別法
柯西判別法(Cauchy's Test),又稱n根判別法(N^ root

丁颜)。令口竖丿同=“,那麽,

•223-
(i) 若〃V1, 2. Un收歛(絕對收歛)
(ii) 若“>1, £七發散。
若“ =1,柯西判別法就無法判別級數的收歛性或發散
性了。
h)扯比判別法(Raabe's Test)
令 lim n(l —

(i) 若“>1, £ Un收歛(絕對收歛)


(ii) 若A<1, £七發散或條件收歛。
若4=1,拉比判別法無效。達朗貝爾判別法無效時,往往
宜採用拉比判別法。
i)高斯判別法(Causs* Test)
£ +
令級數以2七表示。若 Ji
%
1 n 這裏
一切n>N時,|cn|<P,那末,又如
(i) 若時,級數,J收歛(絕對收歛)。
(ii) 若級數£七散發或條件收歛。用拉比判別法無
效時,往往宜用高斯這個判別法。
j)有關絕對收飲級數定理
本節我們簡述一下兩項有關絕對收歛級數的定理。由於本書
水平和篇幅所限,對這些定理不擬加以證明,僅作個簡略的介紹
而已。
定理一:絕對收歛級數的項的次序可以任意作重新排列,重
新排列的級敷收歛於相同的和。條件收歛級數的項如作適當的重
新安排,所得的新級數可發散,也可以收歛於任意和。
•224-
定理二:兩絕對收歛級败的和、差、積也是絕對收歛,加.
減、乘的運算可應用於絕對收歛級數。
【例】 已知一級數 l+2r+r2+2r3+r4 + 2r5 + •••
7 2 4
(1)當, (2),當
—至,(2) ,當 r=—■p (3)
1=一 (釘當 當r=亍時 .試判別
3 :,二:二
這級數的收歛性或發散性。
用達朗貝爾判別法是行不通的 ,因爲 Un+1
=2 | r |或亏
2
|r|,這隨n是奇或是偶有所不同。用柯西判別法,則有:
^nz |u | _ I2 |rn| = n/ 2 |r| 若 a 爲奇數

'1 “―1气/下叮=|[| 若a爲偶數


____ I
l/luJ'=M1 (應用
取極限 limn—n>oo n—>oo
lim 2~ = 1)

由此可知,若|r|<l.»級數是收歛的;若M>1,級數是發
散的。讀者不饒找出(1), (2)兩種情形導致級數收歛,(3)則使
級數發散8
【例】 已知-級數爲时+(豈戶(塁),+…
1 ■4*7,,,(3n — 2) 2
+ 3-6-9 -(30) +…試判別其收歛性或發散性。
如果我們用達朗貝爾方法,就判別不出級數是否收歛或發
地+1 3n+l 2
散:因爲lim
n— 8 Un
lim
u—> oo 3n + 3 1,判別無效。怛
如用拉比判別法,則有極限,

•225 -
lim
—> 00 n(T*D二竖 NT甥)?
lim (3n + 3)2-(3n+l)2 lim 12n + 8
n—>oo n- n—>oo n
(3a + 3)2 .9(n + l)2
8
12n2+8n 12 + T
lim y>i»
n—> oo 9n2+i8n + 9 18 . 9
9+云+語
所以知道給出的級數收歛。
【例】 試判別下開級數的收歛性或發散性:
閑+(尚M瑟泸•••+( l・3・5・・・(2n-l)\ 2
2,4* 6,•,(2n)丿
如果用達朗貝爾法,那末得到極限
2n+l \2
lim
u—>cxo
lim
u—>oo 2u + 2
)=1,不能决定級數是
收歛還是發散。
用拉比判別方法,則有
lim可1一|星屮 n—lim*OO n<I 1 _/\2n
2n+l
+2
1

同樣不能决定級數的收歛或發散。用高斯判別方法:
__4
u) + l =(典 + 1)2= 4n24-4n4-l 5 —n
\ 2n + 2 丿 1 n 血+ 8n + 4
% 4n2+8n + 4
=1一§ + 静,這裏 |Cn|VP
可知因此,這級數發散。

•226 •
習題29

1). 試判別下列各級數的收斂性或發散性:
(a)魯4,((56) 宀-,(。£~,i(b+1)/
)%头澎
n+2
ii+3'

OO
(d) S - 311
n=l n-5n n=l 5h—3.

2). 利用比較判別法證明:
岑•資
3)n=l "4H 收斂,(當P〉1時),而當P<1時,級數發散。
8 oo n2
(b) 2 - tarr~i n 發.散 (C) s 收斂。
n=l n n=l 2°
3). 試判別下列級數的收斂性或發散性:
罗卫写也
(a)n=l (In n)2
a'
p 9 (b)塁n=l n sin2 (—)
n
4). 試用積分判別法判別下開級數的收歛性或發散性:
OO OO
(a、S1 n2 I
n=i 2n3—1
(b) 2
n=2 n(ln n)3 ©糾
5). 下列交錯級數,哪個發散,哪個收斂?
OO

n=l n=i n2+2n+2 (C)岑建 U1


3n—
n—1 •

6). 試求出下列級數中,哪個屬絕對收斂?哪個屬條件收斂?
OO
(a) S - (-l)nT
n=l n2+l
(b) S
OO

n=l
(二"当
n2+l
塁qm
(c)n=l 1 n3
3+1)
(- 』

•227-
(d) Tn—1 2n-lL

7).試用達朗貝爾判別法,判別下列各級數的收斂性或發散性:
a)n=l
(… M 訶'
a。 ⑴
n=l 3血 (0 n=l n2+l
8).下列兩級數,哪個收斂,哪個發散?
z X 1 ,+ P4
1・4 . 1-4-7 +
w 1十3・6为
3-6
z. s 2 , 2-5
2・5丄2・5・8 + •••
(b) £9 +
(b) +

•228-
習題答案
習題2
1). 5 2)・-8 3).2 4). 1 5). 2 6). -6

7). 12 8). 5 9). 一 1* 10).奇 11). 4 12). 3a2

n—m
13) ・我 14). n 15). n a°-i 16). na m~ 17). 一3

18). 3 19). +6 20). 一y- 21). 4 22) 1


・2
23)
3
,10

習題3
1) . (a) 一切 x>-10 的値。
(b) 一切xM 的値。
(c) 除x=±l外的一切數値。
(d) 一切x的値均可。
(e) 一切x的値。
2) .因 xlim
—>2
f(x)=f(2)=4,故 f(x)在 x=2 爲連續。

•229 -
另一說法是:對任何e >0,都有8>0,令
I f(x)-f(2) | = | x2 -4 Ke,當 I x-2 I < b
3).因f(e)不存在,故知f(x)在x = l非連續。倒若我們給f(x)重
新下一定義,令f(l)=lim f(x) = -8,那未函數f(x)在x=l變
x->4
爲連續了。
4)・ 對。我們若能對任何e >0,都能找到b >0o每當I x-xo |<b都
有| f(x) — f(xo) | = | x-xo I < e便行,當然我們可選b = e卽

習題4
1)・ 6x 2). 10 3). 一2 4). 16x 5).

6). 2a x+b 7). 一寺 8). 一 1/2x2 9). -l/(x+2)2

11). 一a/(ax+b)2 12).


5
10). —4/(4x—3)2
3
13).—2x/(x2+a2)2 14
. 2/ 3x-l

習題5
1). 2x-4 5). 5—2x4-9x2

6)・
4
2). 9x2+14x-9

3).
5 +4. 7). x+m
1
X2 5
4)— X2 十 4 - —-法-
2
X3 8). 2x

•230-
1 1 —]__2_
9) 10).
, 27^ (1 X
) 3 3j/x2 X2

11) . (2x—5)(2x2+7)+4x(x2—5x+2)
1
12) .負2 x2+2x +寺 14). 8x3+10x
13) . 24x3+6x2-22x2 15). 4x3
16) . 18x2+26x+9
17) . (px+q)(2ax+b)+p(ax2+bx+c)

18)・ 1)(x2+xT 2) +2/M3+x+ 2) + i/TX2x-1)


(2x+l)
19).
一3
(X—I)2 25). _二 L
/M (.\7~~x. —I)2
-6x2 -2x2+2
20). * 一 1)2 26). (x2—x+1)2

21). 飞亠x4x2+4x+2
2-3x+2)2
5x2—10x
27). (3x2+x-l)2
-8x 4x3 (2a2_x2)
* (x2—4)2 28), (a2—x2)2
1—X —(2x+3/ x )
23). 2v6H:x+1)2 29).
2 x3
X-l
24).
2xf

習題6
2x x
1). 2)・
x2+a 3^=1

•231,
x x _ 2x
3)・ +
l+x2 十 1 一x2 一 1 一x4

.1 , 、
4).产 XN (专+10gX)
1 20 2

2 +i/ *2—5[8(2+/x2—5) +Iog (4叶7)


5). x(4x2—7)
I 4^7~ / x 2 —5 }
a
6). ax+b 17). ae
2/丁
4x 4x
7). 1—x4 18). 2x2-3
8). 3x2-2
x3—2x+5 19). 2 log 7 (x+ 1)7拓a
2+3x2 20).—2x log c-ea2~*2
9)・ logk e - 2x+x3
1
21). l+e*
10). log x+1
22). 0 (1 — 2x-x2)
3_
ID.与
x 23) , (e*+l)2
3 log2x
12) X 24). xe ax
ax (ax+2)
1 V
13).
7 ]+x2 25). (e + e 5
14). & 4
26). 3+6)2

15). 4 5 27). ax x11"1 (n+x log a)

16). lUeb2* 28). xyiogx+i)

•232-
X (1 — log*) 36)• y/~[+*2
29).
37). 0
30) ・ log x2 • x(log x一 1)

31) . 3 (log x+吉) 38). sin 2x

39).
cos/W
1-xlog X
32). x e*
cos x
40)
I * 2/ sin X
33). xlog X

41). ~3 sec2-?-
4
34)
.xlog X X x
42). csc 6
6 0 cot -T-
j &
35)・
] T
1—X2 43). 3 (cos 3x4-sin 3x)

dy 7
dx
(x±)x (丄 x〔)

dy cos

dx
dy
dr

D dy
dx

2) dy
3)
4) dx
1 d s n X

5)
6 dyd X

) = X X

) 2 s

n ) X
1

2 /
x 1
d s n
i

233
7) •
8)

dy =n (a.cos nx—b sin nx)
9)
10 .
dx —x

ll
12
1
dy / 1—X2
3
. dx '(l+*2)/l+xf _X_______

\
dy 4x8
J
X • dr '(1+x)矿

X dy-- ,2(x2—4x+6)
J •
dx / x2—4x4-8

• dy x cos x2
sin
dx =tan x +-^- (sin 2x)+sin 2x-^- (tan x)

dy
dx = (tan *) 2 cos 2x+sin 2x sec 2x
dyd =2 cos 2x tan x+sin 2x sec 标
r sin x
4-sin 2x ・
cos x
2 cos 2x sin x cos x+sin 2x
cos2 x
cos 2x sin 2x+sin 2x sin 2x (1+cos 2x)
COS2 X cos2x
cos2
= ($in 2x) (1+2COS2 Xx-l)

=2 sin 2x
dy —F *一
14).帑
dx 一 2^1
dy sin x
15).必
时= (l+C08X)2

•234・
2 sin x
16). Q+cos x)2
sec2 x
(1—tan x)2
sin x cos x(2+sin x)
18). (14-sin x)2

19)・ =sin x+cos x

2 sin x+x cos x


20).
27 sin x
_____ — L ■
21).
(l+x)/ 1—x2
[
4/x(i+/q
23).解答:令 u = l+sin2 x, y=ni,則有
dy_=du_=i
dx 一 du
HV Odx 、2
" 、
4

= 4-(1+sin2 x) *(2 sin

sin x cos x
/1 +sin2 x

24). -^-=3 (1—x)2(l—x) sec2 3x—tan 3x

=2x(x2+4)s (2x3-3)2 (17x3+36x-12)

・235・
習題8
= 12(2x-l) d2y 2 d 3y
1). 8).

2). =6! 9>・•&=姦.靄=焉


4!
3). io).靄=一- 】q
/ (2x+l)3 . ^xs

4). =0 _3
7 (2x+l)5
5). =a4 sin a^=a〈r
11). s =6 sec4, —4 sec2^
6). =6 d4y 6
dx4 (X+1)4
7). =6 cos x2—24

題9
__x_ 3b 务 xy*

1). dy 7) dy
dx 8)
9 dr
>
)
dy b2x y
2). dx a2y y-x

dy dy 2x+3
2a
3). dx ■y~ dr
> 2y+5

dy
dx 2a 10) • 2x+y
4). ll) < ~ x+2y
’3(l-y2) 2J)

dy
• —6x—7y
5). dx
> y •
X 7x+18y

dy
dx
3
• 2x(x2d-y2)—x
6). 2y(x2+y2)4-y

2 36
da = sin (8+ / )
d, 1 +siA(O+ 肢,
du k+u sin(uv)
14). dv =_v sin (uv)

習題1G
1).若 Ax=I, Ay=21, dy=14
若 Ax=O.l, Ay= 1.461, dy=1.4
若 AxhO.。,Ay ==0.140601, dy=0.14
2) . dy=4(5—2x+x6)s(—2+5x4) Jx
3) . dy=(5 一"旨] 2
m 5 •名-吉-iox) dx
4) . 0.4849
5) . 一0.8747
6) . 1.007
7) . 1.043
8) . 0.795
9) . 0.8665
10) . 1.1
11) . a) 2.25 b) 5.833 c) 10.9546
12) . a) 2.083 b) 2.9907 c) 1.938 d) 1.9954

習題11
1).當x=l, y=5爲極大値。
當x=3, y=l爲極小値。

•237-
2). 3 (極小値), 2
3)・—5-T-(.........
極小値),x 3
4) ・0(極直),x=0; 4 (極大彳直),x=2
5) . 25(極大値),x=2
6).
4 (極大値),x=2
7) . 2(極小値),x='0; —6(極大値),x=—4
8) . 12(極小値),x=2
. 7至
9)) ・ i (極大値), —2; 一 13(極小値),x=3
10) . 0.631 (極大値),x=-0.549; 一2.113 (極小値), 1.215 •
11) . (0, 一 6)極大値;(1, 一 7)極小値
12) . (-2, 20)極大値;(1, 一7)極小値。
13).(-3, 2)極小値; (土 20昔) 極大値
a+2b 4(b—a)。 •)極大値
14).(a, 0)極小値;(-~3~ f ~27~
15). (-1, 一2)極大値; (a<b)
(1, 2)極小値
習 題12
1) .最低平均成本約爲11.64
22).20扌
) .20*
3) ・最大値在(1, 一22),最小値在(5, — 54)。
4) . 5 萬 3 千件;13.112元;247.8 元;13.13 元

•238-
5) . (a)(—1,0), (2, -27)有極小値;(0.5)有極大値。(-0.549,
2.32)是拐點。
(b ) (0, 7)有極大値;(4, 一25)有極小値;(2, -9)是拐點
(c )(1, 一 7)是拐點
(d )極犬値在(0.6, -1.66);極小値在(2, -3), (0.95, 一245)
是拐點。
6) . ( a)(—)140 公噸,(二)78 元,(三)10,920 元,(四)52 元,
(五)7>280 元,(六)一 3,640 元。
(b ) (-)1略•公噸 : ,(二)80.98 元,(三)8476.1 元,
(四)78.5元,(五)8216.33元,(六)一259.8元
(c ) (一)74.5 公噸,(二)92.39 tc,(三)68,836 元,
(四)10详元 (五)7,533.8 元,(六)650.2元。
7) .據找尋平穩値法則,我們得

3 (土)
顯然該公司應僱用超過4人。當x趨近4時,(x-4)趨近0而吝
趨近無限大o X=5時,則成本再度上升。x=4時,成本趨向很高,原因是
職員人手太少,工作堆積。因此應僱用五人,或僱用4人而找聘散工協助。

習題13
1). log 32 2). 0 3).§ 4).0 5). 2 6).
T
3
8). 4-
7).
T 5 9). 0 10). 0 11).C 12). e3; L

•239-
1
13). 1 14). 1 15). e 16). 6 17) c-6 18). e2
J
2
19). 4 20). 1 21). e 22). -1
3

習 題14
1).2 2). 3). 50°28'

習 題15
12
1). IT,TT
丄 6). /兀2 +厂/兀2+ 4
7T

3 4. …
COS 2 X_ 2 sin x
2). 7). sin2x+F sin 2x+l
5' T
3_
3)・u
± ]
8) ・ /4x2 + l' / 4波+1
2x
5
8 15 x-l
4). 9). 2/T
TT,1F T+T x+1
代、3 4 2x2 X4 —I
5)・ T T 10). x*+T X4+l

習題16
1).速度 v=3t2-18t+24, t=5 時,v=9 公尺/秒
質點瞬息留滞時,t=2或4
2) .24呎/秒,--40呎/秒,121呎
3) . (6 — At)呎/秒,6 呎/秒
4) .32呎/秒,144公尺
5).备公R/秒, 另公尺
当公尺
•240-
6). 78.4公尺,2.5秒後,5.5秒後

習題17
1) .速度=6,加速度=0,r t=-|-, -g-»
2) .速度= 12,加速度= 18, t=l, 2
3) ・速度=一3,加速度=0, t=l, 3
4) .速度=一3.14,加速度= -17.1, t=0,1, 2, 3,…
5) .速度=4,加速度=一4, t=0, 1.414,…
妄也....
it
6) .速度=6,加速度= -12, t= 矛工'年, …
5〃 9兀

7) . 30

習題銘
1). T +c 8). a0s+c

2) 著: 3.
X2 + C 9)・一浸+c

3). 2x2
4-c 10)・ +c
ax6
4). 6 +c 11). 3b,孑 +c
5)・ 琴卜
4
c 12).(nx)*+c

3_ X3+c
6).
5 13). 32. x / 2ax +c
7). 21/ s +c
14). — 寻5x3—亲3X2+4X+C

•241・
32 旦 1
15). -=-xT—3x3 2x< +c
5
16) .a^x+寿a耳 X3Z —-
9 a3A xsA
,Q_____ __ 。。 — q
♦- _ e。
5
17) . 2yi (捉- 号淇
3 y2+击3
〒(■③55?4— 尚1
15 y^+c
S 旦3 2 ■—
18). a2t-2at2+談2七2一普豆
q
+c
1 一,_

5
3+b2x2)3+c
19). iW' 5
31).
1
20). 一!®—xD^+c
3
"12 In (8a—6bx2) + c
32). x -音+孕-ln(x+l) + c
; 2+4bxs)'3+c
21) . y(3;ax
,。, ,、 1
33). 2(x2+x3)*+c
22) . J~|(a2+x2)*+c
34). _1 (x2+a2)"l+c
2(n+l)
23) —2—1)2+c
35). 2- (x
24). j* sin x dx— ■|<x»®x+c

25). j* sin3 axdx ■sin4 a x+c


36).
4 (a2—x2)"t+c
37). 不
i
4b~(a+bx8)9+c
26). J—/ a2—x2 4-c
1
38)・ £(1 一3x/+c
27).•|-(1 + x2)1-4-
音(1+x?) 耳+cc 6
2
3
a 39)・ 戋(ax+b户+c
28). (b— x)5 +c
2
40) ."5^-(1+
3,一 - £ n+1
29). y (14-X2) 34-c
41) . Z x2+2x—1 +c
30).专 ln(x2 — l)+c 4x+l
8(2x+l)2 +c
•242-
43).专(*+2)/顽+c
"sin弋~+c
45). *(x-1X6x4- l)+c

13x2—3x+l
46). 一司(x—l)3 4-c
1 12x2—8x+23 丄。
47). rv ・- --- 7===== c
15 / 2x+l
• - x . 一
48). sin"1—;+c
y
49). —/4—x2 +c
50). 4- a2
a2 sin-i
sin"1 — x/a2—x2 +c
2 a 2
51). sin-i x+c

52). 4-sm-1(ax) + c
a
1 x
53). xZ 4-x2 + 2 siK奇 + c
2
54). 2/ sin。+c
55)・烏一
sin 0 3 sin3 0
+c
1 (30—4 cos^—sin。cos^)+c
56)
・2
1
57) . 4-(50 —2 cos 29 —寺3
sin绍)+c
2
厶 z2
58) . 6+戒功+丄 sin 20+4-sin 3。+*sin 4^+c
3 8

•243-
. 1 3.
一与(l
59) . -j (1 —
一 xz)2 + cC
X2)@ +

60) . §(2x8-如)石二^+§a4 siK告+c

習 題19
1). _L
y= 2 ax2+c 14). y=cx2
2). y=ceax 15). y=(x—c)2

3). y2 1
c—2x 16).(x—a)2+(y—b)2=c

4). 1 - x+c 17). x2+y2=c


2
5).
c+泸
y= c— e2* 18). xy=c

1 3一- 1 x2—2x+c
6). y=4-x
2 2
19). y=cx+2
2+c放
7). y= 1—c e3* 20). y=2 e~x
c—COS DX
8). y= n 21). y=3 (1 — ef

9). 1
tan(~^*iiy) =c e皿 22). y=l—cos
2

10). y=tan x+c 23) y=^


23)・〃=左
11) . y+sin y cos y=2x+c 24) .212呎/秒
12) . y=/x2—1 —sec-ix+c 25) . 12 秒
13) . y2=x2+2 cx+c2+1

•244・
習 題20
1) . yi=e-5,236t, y2=e"0,764t
y=-8.096 e-5-236t+24.096e-°,764t
2) . a )y=—32.5 e-2t+55e-t —2.5
b ) y= 35 e-3«
-3t_A
-4t t==215_
9 3
3) . 79年 6) . 13.9年
4) . 0.07% 7) . 7.6呎/秒
5) . 4.2%

習 題21
1) . / 3x2—5x+6+c
2) . 2Ci/x+l-ln (l + i/m)〕+c
3)
3). -qT-(3 产一4)
.-^r~(3 4) 4/ 1),
(e*+1)
4/(e*+ ,+c
+c
厶1

6 3
4). x+gy
5
x5 +音必
2
x2 + 2 /F+ 3 汐亏E+ 6 Y^r + 6 In | 之H
—11 +c
5). tanW x2— 1 +c 7) . sin (In x) + c
(In x)3 8) . |(l + sin x)6+c
6). ■—3~~ +c

9). 4" tan"1 tan x)+c

ax
10).鑫(也*■ + a2-*-x 2 )+c

•245-
11). x»/a2—X2 +c

12).;/x2—a2 —a tan-1

13). In (x+/a2+x2)+c

14). 2 xi/x2—a2 5告+ c
⑸.
-----2Zx =2+l/2- tan-】匕法^
----- E +c
16). — 1 —2也或/e* — 1+c
xe36 19) . 2 In cosh告+c
17). In I l + xex I +c 厶
18). 2 sin /* +c. 20) . sin x—tan'1 (sinx)+c

21)・-膏2 i^a3—x3 (2a3 + x3)+c


—i/a2—x2 —sin"1 +c 23) * 1_ x
22). 一-a2- • / x2-a2 ,+ c

26 • 3x+4 丄一c
24). 5. In (3x2 + 8x+10)—余卷 tan-i 訴賢 (====•+
6
x 2— 1
25). 2 In I(X+1)3 I 一 4 ZT tan-i 2x+l c

跖)-去 x2-l
4-c 27).

28).寿1丨 4x+l + /( 4x+l 2 -1 I +c


~5~ ~T~

・246・
習 題22
1). sin x—x cos x+c
2). —x2 cos x+2 x sin x+2 cos x+c.
3). —-^-x cos 2x + 2x+c

x2 i q<
飞-十玄 sin2x 4--|-x cos 2x 2x)+c
4).

5). e*(X,一3x2+6x—6)+c

6). 矗3 - In a + •)+c

7). (sin x+cos:x)+c


2 、
8). x Id x—x+c
2 4 旦
9). TX21nx — 三 X2+C
x^lnx-9

10). x ln2 x—2(x In x—x)+c.

11).
2 3 a 3 ic 3
16 ~ X2
x恐Q$x—令 X2 In2 X +专
29 旦
in x —• 32- x2+c
3 5 y9 27
mi

xa
12). x tan x +ln I cos x I +c
~2~

13). 专 *(cos In x+sin In x)+c

14)・ _±2 Z (x4+2xa+2)+c


15). In x+sin In x)+c

16)尸 tan-^- +c

•247・
17).當 n=l, Ii =— tan"1
音+。
當 n=2, I2 i・忐蒙■ +击tan-i^-
tan"i—+ l1

當n=3, «=矗+32二2)2+碁(翥・~\2畠2 + 由
tan-i 告)+ c
18). (x2—2x+2) ex4-c
19).十 e* (sin x—cos x)+c
2
20).
1 ex [(X2—2x4-1) sin x+(x2—1) cos x〕+c
2
21) .(寺+十) cos 2x 4-
(lx2+lx+f)
3 sin 2x+c

22) .見習題22末端的提示

習 題23
2^_ +.x2 +4x4-In j x2(X—2)5 4-c
1). T+~2 I (x+2)3
2 In x-2 + I +c
・T x+1 x-2
3)・
4 11 +c
x-2 2(x-2)2
11I «
4).-y-ln x1I __ 1 1__j_
4 2x2 *4 In I x—2 I — 2(x-2) +c
7_
x
ten-i 二 2 4■厂
5).

•248-
6).
5 In (3x2+8x+10) — 26 tan-i ^=4
丢 3x+4 - £
6 3Z14
3
)
2 (x2+4x+5)—tan-1 (x+2) + c
1 = Ax+B cx+D
8). 提示:令 x4+l x2+/,x+l 4-
x2—v/^2- x+1
答案: X24-xZ2"+l
In_______ _ _____ 士 ~T~tan"1
4Z2 X2—/云 + 1
j/V _
4--^—tan"1 (/2 x—l)+c

9). 为 taiK 关丄+c


7T /3x
月+c
10) .提示:令 x—b =t可有击( +-|-+31ii I t I -t)+c
x-2
最後再將t x+1 代換囘來,化簡後卽得答案。
11). In | x | 4-In (x2+1) + x2 dx
(X2+1)2
=ln I x 3+1) I +* tan-1 I X
X---- k +c

12)・ 5x + 2
3(x2+x+l)
+音皿 x—1 I —|-ln(x2+x+l) + 3Z~F
8 tan-1


/ 3 +c
^=-4-c

13). 5x+3 +c
(x-3)(x+l)

習 題24
1).令 sin x=t, cos x dx=dt;得* sin7 x— 刍 sin9 x
9
•249-
2). 令 cos xf 得?+t + c = 4-cos x+c

3). 2 + "(寺+令)I +c

4). 令 sin x=t,得 I + ln I tan x | 4-c


2 sin2x

5). 1 x4x— in3 2x4-c


16 48 . 64
5 x—*sin2x+ 3 sin 4x4-sin3 2x+c

6).
16 48 -W
7).
顽<3x—sin 4x +§sin 8x)+c

8). ] I +c
3 tan3 x tan x
I 3 + 3 tan x 4--j-tan3 x+c
9). 3 tan3 x taQx 3
x + 2+t
10).用 tan 答案是: 2-t I + c=/ln
2+tan-J
+c
2—tan-jr-
2
1 tan-i
11).令 tan x=t,得 百 +c 1 tan"1 ( tan x
2 )+c

12).普[x+2 In I cos x+2 sin x | ]+c


5-
13). —In I sin x+cos x | +c
_____ dx_____ dx
14). a sin x+b cos x J7=57b2 sin (x+t)
__1
得到 %2+b2 In tan x+4 +c

•250-
這裏sin t ,cos t
/a2+b2 Za2+b2
15).令 tan x=t, 得出 2/tan x +c

16).
4-cot3 x+cot x+c
2
17)・-^tan2 x tan x + c
5
18).令 ex=t. dx=dt;得出
x+ln [1+志 1/e2x+cx4- 1 ] + c

19).利用部份積分法,得出
xe*
_ 'x+T +e«+c= x+1
+c

4
X
21).令tan =t;得出In I 1+t I +c=ln I 1 + tan寺 | +c
X
22).令 W —t, x=tan-1 t.得出
~2~
2t tan-i t—In, (14-12) 4-In (l + t2) + c
x tan x +c
T
1+X4 +c
23). .n'ri
2x?
3
24). In (x2+l)—tan-1 x +73-7+0
x+1
1 x-x2)3+c
25). ?(2 ]
3 26). 3a2 x3 (a2—x2)3+c

•251-
3
27). WsinTx-*x(3+2x2)/i-x2 +c
8 8 —

(用上節所討論及的公式)
習 題25
n a4
1). 21 2). 2 3). * 4). 4/3 5). 1 6).

7). 4r
8
8).
-I 9).¥ 10). i/T- 1 11).
256兀甲
315 a4
12).导一 log 3 13). 8a 14) 4/3
7t 1 .、2^
3 15). •云7亏=
16)•3Q
n 7 n
18). 19). 20). 21). In 1.5
-4 T2- 2"

習 題26
1). 一2
e + 1(提示:用部份穆分法)

2).令 x=sin t, dx=cos t dt;得出


3+2ZJ
孕- In (2 —1/3~)= In

~~3-
3).令 l + x=t2, dx=2t dt.

得出-y-
¥
4). 2
x
5).用 tan ~2~ =t;得出
A tan'1

1

6). In (/6 + 2/J-/亨-2)

•252-
n Cn/2
7). 用代換x 土得出J。 sinn t dt;再用轉化公式(Reduction
Formulas).
8). 用t =
7tt2
寺得
9). ~512
10). 4 In (2+/J)-2/J

習題27
1
1)・肅岌
3 2). 6)・ ~a~
6 4).讀者自證之 5).1
6). 36 7). In/歯 5
8). 9芝 9). 56*
6 10). v
o

11). c2 In 身 12). 5| 3
13). 8 14). 一号
4 +ln 4 15)・*
2
16). 5y- 16
17). fin
3 7ZF:tanT3/l~
18)・利用參數方式變換:x=a cos t, y=b sin t,
得出答案:〃ab 圖形面積爲^a2
習題28
4?rab2 zra3 (15—16 In 2)
】)• -2-
3 2).夺W 3). Zr2a2b 4). 号
2

5).
7T a3
IT
32兀a。
6). 32兀 7).
16 a3
并 8).于每2—1)
9). H05 10).号 11). ”a3[穿Jn(2/万+3)-£]
3 8 6

•253-
12) . 5渺 a$ 13). 4/歹兀 56
14)孚渺 2 abh
15).音
3 3

習題29
1). (a)收歛 (b)發散 (c)發散 (d)收歛 (e)發散
3) . 3)收歛 (b)發散
4) ・(a)發散 (b)收歛 (c)發散
5) .(a)收歛 (b)・發散 (C)發散
6). (a)絕對收歛 (b)條件收歛 (c)發散 (d)絕對收歛
7) ・(a)發散 (b)收歛(絕對收歛) (c)發散
8) . (a)發散 (b)收歛

•254-
國立中央圖書館出版品預行編目資料
微積分初舉/孫文先編著•“六版•-臺
北市:九章.1994(民83]
面: 公分
ISBN 957-603-064-1(平裝)

I.微積分

314.1 83011364

微積分初季
孫文先編者
出版者/九章出版社
地址/台北市信義路3段147巷15弄5-1號7樓
登記證/局版臺業字第2327號
電話 / (02)27048405、27045243、23257970
傅真 / (02)27067353
e-mai1:ccmp@tptsl.seed.net. tw
發行人/孫文先
印刷者/九章文具印刷品有限公司
岀版日期/ 1999年11月六版二刷
定價/ 150元整
郵政劃撥/ 1053467-6 九章出版社
總經銷/成信文化事業股份有限公司
台北縣新店市中正路542-7號6棲
查詢本社圖書目錄/~一
http://www.hk.super.net/~phcheung/ccmp.html

You might also like